Vous êtes sur la page 1sur 186

Ejercicios Resueltos de Física General II (FS-0310)

Carlos Vargas Agüero


Índice general

1. Estática de Fluidos 4

2. Oscilador Armónico Simple 19

3. Movimiento Ondulatorio 34

4. Ondas Sonoras 43

5. Superposición de Ondas 47

6. Temperatura 57

7. La Primera Ley de la Termodinámica 67

8. Teoría Cinética de los Gases 81

9. La Segunda Ley de la Termodinámica 88

10.Campo Eléctrico 100

11.La Ley de Gauss 115

12.Potencial Eléctrico 128

13.Capacitancia 141

14.Corriente y Resistencia 155

15.Circuitos DC 165

1
ÍNDICE GENERAL ÍNDICE GENERAL

Ultima edición: I Ciclo 2018.


Carlos Vargas
carlos.vargasaguero@ucr.ac.cr

2
ÍNDICE GENERAL ÍNDICE GENERAL

Recuerden no solo leer, se aprende haciendo los ejercicios. También aplica para física.
Comic 353 “Warrior" de Abstruse Goose1

1
Más humor de este estilo en abstrusegoose.com

3
1 | Estática de Fluidos

[Resnick] Problema 15-13


Un cascarón esférico hueco de acero flota casi completamente sumergido en el agua. El radio externo mide
58.7 cm y la densidad del hierro es ρF e = 7.87 g/cm3 . Determine el radio interno del cascarón.

Figura 1.1: [Resnick] Problema 15-13

Solución
Sea R el radio externo y r el radio interno. Las 2 fuerzas que actúan sobre la masa de hierro son la fuerza
de gravedad y la fuerza boyante.

X
Fi = Fb − Fg = 0 ⇒ Fb = Fg
i

El volumen a usar en la fuerza boyante, es el volumen que se desplaza de fluido. El fluido no siente el
hueco interno que tiene la esfera, solo le importa el volumen desplazado. Por lo que el volumen a usar en
la fuerza boyante es

Fb = ρa V g = (4/3)πR3 ρa g
Para la fuerza de gravedad, es nada mas Fg = mg. Pero tenemos la densidad, no la masa. Para la masa si
hay que considerar que la esfera es hueca, por que en el hueco no hay masa. Hay que quitarle ese volumen
cuando usemos m = ρV
 
4 3 4 3 4
πR − πr = πρF e R3 − r3

Fg = mF e g = ρF e VF e = ρF e
3 3 3

4
CAPÍTULO 1. ESTÁTICA DE FLUIDOS

Con las 2 fuerzas expresadas en términos de la incognita y datos dados, ya podemos reescribir la ecuación
de suma de fuerzas y despejar r
4
Fb = Fg ⇒ (4/3)πR3 ρa g = πρF e R3 − r3

3

ρa 1/3
 
⇒r =R 1−
ρF e
Evaluando con los datos del problema

1 1/3
 
r = (58.7) 1 − = 56.10 cm
7.87

El radio interno es r = 56.10 cm

[Resnick] Problema 15-3


El agua se halla a una profundidad D detrás de la cara de la cara vertical de un dique. Supongamos que W
es el ancho del dique. (a) Calcule la fuerza horizontal resultante que sobre él ejerce la presión manométrica
del agua, y (b) la torca neta debida a la ejercida alrededor de una línea que cruza O paralelamente al
ancho del dique. (c) Donde se encuentra la línea de acción de la fuerza equivalente?

Figura 1.2: [Resnick] Problema 15-3

Solución
(a) La presión en función de y, con y positivo hacia abajo es P = ρgy + Patm . Pero están pidiendo
la fuerza/torque de la presión manométrica, entonces hay que restarle la Patm . Lo cual no da Pm =
P − Patm = ρgy
No podemos decir nada más que F = Pm (y)A porque tenemos una presión variable. Hay que obtener
la fuerza total de otra manera. La presión varía en la dirección vertical, a lo largo de la horizontal
es constante, entonces podemos proceder de esta manera. El diferencial de fuerza dF a lo largo de la
horizontal es dF = Pm (y)W dy = ρgyW dy
Ahora podemos encontrar toda la fuerza en el dique integrando sobre dF .
ˆ ˆ D
D2
F = dF = ρgyW dy = ρgW
0 2

5
CAPÍTULO 1. ESTÁTICA DE FLUIDOS

Figura 1.3: Presión manométrica en el dique


[Resnick] Problema 15-3

Donde los límites de integración van de 0 hasta D ya que la variable y recorre ese intervalo.
(b) Usemos otra coordenada, ya que el torque lo vamos a obtener respecto a O. Definamos z = D − y,
con z positivo para arriba. El torque, al igual que la fuerza, no puedo nada mas hacer τ = r × F, ya
que la fuerza en el dique es variable. Hay que obtener el dτ e integrar. El dτ es dτ = r × dF. El vector
r = z k̂ y dF = dF ĵ son perpendiculares, ya que es un productor cruz, podemos cambiarlo a una expresión
escalar, ya que el ángulo entre los vectores es 90, la expresión es dτ = zdF = zPm (z)W dz donde Pm (z)
lo obtenemos sustituyendo en y = D − z. La Pm (z) es Pm = ρgy = ρg(D − z). Ahora ya podemos integrar
ˆ ˆ D
D3
τ= dτ = ρgz(D − z) dz = ρgW
0 6

(c) La línea de acción es posicionar la fuerza que hace toda la distribución de fuerza, posicionarla en
un solo lugar y hacer que tenga el mismo torque que la distribución de fuerzas. Lo relacionamos con los
resultados obtenidos en para F y para τ . Sea req la distancia desde abajo hasta donde se ubica la línea
de fuerza equivalente
D3
 
ρgW
τ 6 D
τ = req F ⇒ req = = 2
=
F D 3
ρgW
2
La distribución de fuerzas actúa igual que una fuerza de magnitud F = ρgW D2 /2 actuando a una
distancia req = D/3 para efectos de la fuerza y torque generado.
[Resnick] Problema 15-14
Un bloque de madera tiene una masa de mw = 3.67 kg y una densidad de ρw =594 kg/m3 . Va a ser
cargado con plomo para que flote en el agua con 0.833 de su volumen sumergido. Qué masa de plomo mP b
se necesita (a) si se encuentra en la parte superior del bloque, y (b) si el plomo está sujeto a la madera
por debajo? La densidad del plomo es ρP b = 1.14 × 104 kg/m3 .

Sea f la fracción de volumen sumergido de la madera, donde va a tener un valor de f = 0.833 para ambos
casos, pero para hacerlo en general, usemos f hasta el final.
(a) Para el primer caso, tenemos el Pb afuera del agua. El agua no va a ejercer fuerza boyante sobre el

6
CAPÍTULO 1. ESTÁTICA DE FLUIDOS

Figura 1.4: Sistema equivalente


[Resnick] Problema 15-3

Figura 1.5: Diagrama de fuerzas


[Resnick] Problema 15-14

Pb, solo sobre la madera. Tenemos la fuerza de gravedad de la madera y el Pb hacia abajo y la fuerza
boyante hacia arriba. Estas dos fuerzas son iguales ya que el sistema está en equilibrio.
X
Fi = Fb − (mw + mP b )g = 0 ⇒ Fb = (mw + mP b )g ⇒ ρa f Vw g = (mw + mP b )g
i

Cambiemos el volumen Vm con Vm = mw /ρw


mw
⇒ ρa f g = (mw + mP b )g
ρw
De aquí podemos despejar la masa mP b que da
 
ρa
mP b = mw f −1
ρw

Evaluando con los datos  


1000
mP b = (3.67) (0.883) − 1 = 1.78 kg
594
Se necesita una masa de mP b = 1.78 kg.
(b) Para el segundo caso, tenemos una fuerza extra que considerar, la fuerza boyante ejercida sobre el

7
CAPÍTULO 1. ESTÁTICA DE FLUIDOS

Pb. El sistema está en equilibrio, hacemos sumatoria de fuerzas


X
Fi = Fb(w) + Fb(P b) − (mw + mP b )g = 0 ⇒ ρa f Vw g + ρa VP b g − (mw + mP b )g = 0
i

Cambiamos los volumenes respectivos, con Vw = mw /ρw y VP b = mP b /ρP b , la ecuacón queda como
mw mP b
ρa f g + ρa g − (mw + mP b )g = 0
ρw ρP b
Ya podemos despejar mP b , lo cual da
 
ρa
f −1
ρw
mP b = mw  
ρa
1−
ρw

Evaluando con los datos  


1000
(0.883) −1
594
mP b = (3.67)   = 1.95 kg
1000
1−
1.14 × 104
Se necesita una masa de mP b = 1.95 kg. Comprando con el resultado de (a), cuando estaba arriba, se
necesitaban 1.78 kg, una menor cantidad. Tiene sentido que se necesite má masa de Pb para la segunda
parte, ya que ahora hay mas fuerza boyante por que el Pb está sumergido, tiene que agregarse más masa
de Pb para cancelar esa fuerza y quede en equilibrio el sistema.

[Resnick] Problema 15-15


Un objeto que flota en mercurio tiene sumergido un cuarto de su volumen. Si se agrega suficiente agua
para cubrirlo, que parte de su volumen permanecerá sumergido en el mercurio?
Dato adicional: Densidad del Mercurio ρHg = 13600 kg/m3 .

Solución
Sea ρ0 y V0 la densidad y volumen del objeto
Las fuerzas presenten cuando solo está en mercurio es la fuerza boyante del mercurio y la fuerza de la
gravedad. X
Fi = Fb − mg = 0 ⇒ Fb = mg
i

Relacionamos la densidad del mercurio y la del objeto.


1 ρ0 1
ρHg V0 g = ρ0 V0 ⇒ =
4 ρHg 4

Con esta relación seguimos a la parte cuando agregamos el agua. Sea f la fracción del volumen sumergido
en el Hg. La fracción que está sumergida en el agua debe1 ser entonces 1 − f . El volumen desplazado en

8
CAPÍTULO 1. ESTÁTICA DE FLUIDOS

Figura 1.6: Diagrama fuerzas, primera parte


[Resnick] Problema 15-15

Figura 1.7: Diagrama de fuerzas, segunda parte


[Resnick] Problema 15-15

el Hg es f V0 y en el agua es (1 − f )V0 . Las fuerzas presentes cuando está el agua es la boyante del Hg, la
boyante del agua y la fuerza de la gravedad.

X
Fi = Fb(Hg) + Fb(w) − mg = 0 ⇒ Fb(Hg) + Fb(w) = mg ⇒ ρHg f V0 g + ρa (1 − f )V0 = ρ0 V0 g
i
Ahora usamos el resultado de la primera parte para cambiar ρ0 .
ρHg
ρHg f V0 g + ρa (1 − f )V0 = V0 g
4
De aquí podemos despejar cuando es f , que da
ρHg
− ρa
f= 4
ρHg − ρa
Evaluando con los datos
13600
− 1000
f= 4 = 0.1901
13600 − 1000
1
Las fracciones son porcentajes, todo el volumen del objeto tiene que sumar 1 (El % 100), la fracción en el Hg más la del
agua es f + (1 − f ) = 1

9
CAPÍTULO 1. ESTÁTICA DE FLUIDOS

La fracción de volumen en el Hg es f = 0.1901. Este resultado tiene sentido, ahora hay menos fracción
en el Hg, que inicialmente la fracción era 1/4. Ya que ahora el agua ayuda a empujar hacia arriba, el Hg
tiene que ejercer menos fuerza para soportar todo el peso del objeto.

[Sears] Problema 12-58


Un estrecho tubo de vidrio en forma de U, con extremos abiertos, se llena con 25.0 cm de aceite (de
gravedad específica 0.80) y 25.0 cm de agua en los lados opuestos con una barrera que separa los líquidos
como se muestra en la Fig. 1.8. (a) Suponga que los dos líquidos no se mezclan, y encuentre las alturas
finales de las columnas de líquido en cada lado del tubo después de que se retira la barrera. (b) En los
casos siguientes, obtenga por razonamiento físico y no por cálculos: i. Cuál sería la altura de cada lado si
el aceite y el agua tuvieran las densidades iguales? ii. Cuál sería la altura si la densidad del aceite fuera
mucho menor que la del agua?

Figura 1.8: [Sears] Problema 12-58

Solución
Sea d la altura de las columnas de agua/aceite, d = 25 cm. El problema pide luego evaluar casos límites,
si el aceite tuviera la misma densidad del agua o mucho menor. Voy a escribir la densidad del aceite de la
forma ρo = f ρa , donde ρo es la densidad del aceite. La f es que tan grande es la densidad con respecto a
la del agua. La gravedad especifica dice que es f = 0.8. Voy a mantenerlo así para los casos límites. 2
Una vez que se quita la barrera, el sistema no va a estar en equilibrio. El agua va a desplazar al aceite.
Digamos que el nivel del agua baja una distancia x. Ignorando la parte del fondo en forma de U, la
columna x de agua que se fue del lado izquierdo ahora está del lado derecho. La columna de aceite sube
una altura x tambien.
Ahora ocupamos ver como planteamos el problema. En diferentes fluidos no puedo asegurar que la presión
sea igual al mismo nivel, pero si me mantengo en el mismo si. En el Nivel de Referencia (N.R) como se
muestra en la figura, ya que puedo llegar del lado izquierdo al derecho manteniendome en el mismo fluido,
la presión a ese nivel va a ser la misma. La presión sacandolo del lado derecho es P = Patm + ρo gd =
Patm f ρa gd.
Por el lado izquierdo, la presión obtenida es P = Patm + ρa gh, donde h es la altura de la columna de agua
desde el nivel de referencia hasta arriba. El valor h no lo sabemos, pero podemos averiguarlo viendo que
d = 2x + h ⇒ h = d − 2x
2
Es mejor que rehacer todo el problema con un nuevo número. Trabajen con letras, no les de miedo.

10
CAPÍTULO 1. ESTÁTICA DE FLUIDOS

Figura 1.9: Dimensiones


[Sears] Problema 12-58

Reescribiendo la presión en el N.R., tenemos P = Patm + ρa g(d − 2x) Ahora igualando las presiones

P = Patm + ρa g(d − 2x) = Patm f ρa gd

De aquí ya podemos despejar x, que da


d
x= (1 − f )
2
(a) Evaluando para los datos d = 25 cm y f = 0.8 = 4/5
25 5
x= (1 − 0.8) = = 2.5 cm
2 2
La altura x es x = 2.5 cm.
(b)
(i)Si tuvieran las mismas densidades, el agua no desplazaría al aceite. Es razonable pensar que el sistema
se va a quedar como está. Corroborando con el resultado obtenido, haciendo f = 1, vemos que
d
x = lı́m (1 − f ) = 0
f →1 2

En el límite que tengan las mismas densidades, x = 0, esto significa que no se desplazo el sistema.
(ii) En el otro caso, dice que la densidad del aceite sea mucho menor a la del agua. Imaginemos que el
aceite tiene la densidad del aire, basicamente 0 comparado al agua, es un poco exagerado, pero pensemos
así en el caso extremo. Con una densidad mucho menor, es como si el aceite no estuviera ahi, se podría
esperar que el agua se acomode como si no estuviera y que en cada lado tenga la mitad del agua. Osea
que bajo x = d/2. Corroborando en el caso límite con el resultado obtenido

d d
x = lı́m (1 − f ) =
f →0 2 2

Como se esperaba.

[Serway] Problema 14-60

11
CAPÍTULO 1. ESTÁTICA DE FLUIDOS

Un balón lleno de Helio (cuya envoltura tiene una masa de mb = 0.250 kg) es atado a una cuerda uniforme
de longitud ` = 2.00 m y de masa m = 0.0500 kg. El balón tiene forma esférica con un radio de r = 0.400
m. Cuando se suelta en el aire a una temperatura de 20◦ C y una densidad de ρair = 1.20 kg/m3 , este
alza a una longitud h de la cuerda y se mantiene estacionario. Deseamos encontrar la longitud de la
cuesta levantada por el balón. (a) Cuando el balón se mantiene estacionario, cual es el modelo de analisis
apropiado que lo describe? (b) Escriba la ecuación de fuerza para este balón en este modelo en términos
de la fuerza boyante Fb , el peso del balón Fb al, el peso FHe del Helio y el peso Fs del segmento de cuerda
de longitud h. (c) Haga una apropiada substitución para cada una de estas fuerzas y resuelva de manera
simbólica para la masa ms del segmento de la cuerda de longitud h en términos de mb , r, ρair y la densidad
del Helio ρHe . (d) Encuentre la longitud de h.
Dato adicional: ρHe = 0.179 kg/m3 .

Figura 1.10: [Serway] Problema 14-60

Solución
(a,b,c) Hay un segmento de la cuerda que está en el suelo. La fuerza normal que ejerce el suelo es para
soportar el segmento de la cuerda en el suelo. El resto de la cuerda en el aire es soportado por la fuerza
boyante que hace el aire. Voy a considerar solo la parte en el aire. Las fuerzas que actúan sería la fuerza
boyante que hace el aire sobre el objeto (ignorando que la cuerda tenga volumen), que es Fb = ρair V g.

Recordatorio: La fuerza boyante es una fuerza que experimenta un objeto debijo al fluido que lo rodea.
El fluido que rodea al globo es el aire. En la fórmula de fuerza boyante Fb = ρgV , ese ρ es el fluido que
lo rodea, que es aire. Aunque el balon esté lleno de He, el He no es quien realiza la fuerza boyante, es el
fluido que rodea al sistema (El aire).

El resto de fuerzas son causadas por la gravedad, la masa del globo (El plástico que esta hecho) seria
mb g, el peso del He que es FHe mHe g, y el peso de la cuerda que es m0 g. Esta masa m0 no es la masa total
de la cuerda, si no solo la masa que está al aire. Podemos averiguarlo usando que la cuerda es uniforme.
La masa sería m0 = µh donde µ es la densidad lineal de la cuerda. Y ya que es una cuerda uniforme, la
densidad lineal es µ = m/L, entonces m0 es m0 = µh = m(h/L).
Ahora podemos plantear la sumatoria de fuerzas igual a 0 y queda

X h h
Fi = Fb − mb g − m g − mHe g = ρair V g − mb g − m g − mHe g = 0
L L
i

12
CAPÍTULO 1. ESTÁTICA DE FLUIDOS

Figura 1.11: Diagrama de fuerzas [Serway]


Problema 14-60

Cambiamos mHe = ρHe V = ρ(4/3)πr3

4 h 4
⇒ ρair πr3 g − mb g − m g − ρHe πr3 g = 0
3 L 3
De aquí ya podemos despejar h que da
 
L 4 3
h= (ρair − ρHe ) πr − mb
m 3

Evaluando con los datos


 
2 4 3
h= (1.20 − 0.179) π(0.4) − 0.25 = 0.948 m
0.05 3

El valor es h = 0.948 m

[Serway] Problema 14-71,72


Un beaker de masa mb = 1.00 kg contiene mo = 2.00 kg de aceite y una densidad ρo = 916.0 kg/m3 que
descansa en la balanza. Un bloque de hierro de masa mF e = 2.00 kg suspendido a una balanza de resorte
está completamente sumergido en el aceite. Determine la lectura de equilibrio de ambas balanzas.
Dato adicional: Densidad del Fe ρF e = 7860 kg/m3
Solución
Empecemos haciendo sumatoria de fuerzas sobre el bloque. Sobre el bloque actúa la fuerza de tensión T ,
el peso del bloque mF e g y la fuerza boyante del aceite sobre el hierro ρo VF e

X
Fi = T + ρo VF e g − mF e g = 0 ⇒ T = mF e g − ρo VF e g
i

Sustituyndo VF e = mF e /ρF e queda


 
mF e ρo
T = mF e g − ρo g = mF e g 1 −
ρF e ρF e

13
CAPÍTULO 1. ESTÁTICA DE FLUIDOS

Figura 1.12: [Serway] Problema 14-71,72

Figura 1.13: Diagrama de fuerzas sobre el Fe


[Serway] Problema 14-71,72

Sustiyuyendo con los datos  


918
T = (2.00)(9.8) 1 − = 17.31 N
7860
La lectura en la balanza de arriba es T = 17.31 N.
Para obtener la fuerza normal que hace la balanza de abajo, hagamos un procedimiento un poco extraño.
En lugar de considerar todo el sistema (el aceite,beaker y el bloque), consideremos nada mas el sistema
del aceite-beaker. El aceite-beaker experimentan la fuerza normal de la balanza de abajo N , la fuerza de
gravedad de sus propios pesos mb g + mo g. Y experimenta otra tercera fuerza. Ya que el aceite ejerce una
fuerza sobre el Fe, que es la fuerza boyante, el Hierro va a ejercer una fuerza sobre el aceite por la tercera
ley de Newton. Si el aceite ejerce una fuerza ρo VF e g sobre el hierro, el hierro ejerce una fuerza −ρo VF e g,
misma magnitud pero en dirección contraria. (La dirección poniendola en el menos.)
La sumatoria de fuerzas sería
X mF e
Fi = N − ρo vF e g − (mo + mb )g = 0 ⇒ N = ρo g + (mo + mb )g
ρF e
i

Evaluando con los datos


(2.00)
N = (916) (9.8) + (2.00 + 1.00)(9.8) = 31.68 N
7860
La lectura en la balanza de abajo es N = 31.68 N.
Para comprobar que lo hicimos bien, hagamos la sumatoria de fuerzas de todo el sistema, nos debería dar

14
CAPÍTULO 1. ESTÁTICA DE FLUIDOS

Figura 1.14: Diagrama de fuerzas sobre el beaker-aceite


[Serway] Problema 14-71,72

0.
Las fuerzas externas son N, T y las fuerzas de gravedad (mo + mb + mF e )g. La boyante no aparece ya que
es una fuerza interna.

Figura 1.15: Diagrama de fuerzas sobre el beaker-aceite


[Serway] Problema 14-71,72

X
Fi = T + N − (mo + mb + mF e )g = 17.31 + 31.68 − (2.00 + 1.00 + 2.00)(9.8) = 0 N
i

Lo cual es lo que se esperaba.

[Cengel] Ejemplo 1-10


El agua en un tanque es presurizado con aire, y la presion es medida con un manómetro multifluido. El
tanque está localizado en una montaña a una altitud de 1400 m donde la presión atmosférica es 85.6 kPa.
Determine la presión del aire en el tanque si h1 = 0.1 m, h2 = 0.2 m y h3 = 0.35 m. Tome las densidades
del agua, aceite y mercurio como 1000 kg/m3 , 850 kg/m3 , y 13600 kg/m3 , respectivamente.

15
CAPÍTULO 1. ESTÁTICA DE FLUIDOS

Figura 1.16: [Cengel] Ejemplo 1-10

Solución
Vayamos calculando presiónes mientras subimos y bajamos la altura y cambiamos de fluidos. Sea Pk la
presión en el punto k.
La presión P2 es la presión atmosférica. P2 = Patm . La presión 3 es la P2 + la presión de la altura h3 de
Hg.
⇒ P3 = P2 + ρHg gh3 = Patm + ρHg gh3
La presón P4 es la misma que P3 ya que esta al mismo nivel que el punto 3 y aún en el mismo fluido.

P4 = P3 = Patm + ρHg gh3

La presión en el punto 4 es P5 + la presión de la columna de aceite h2

P4 = P5 + ρo gh2 ⇒ P5 = P4 − ρo gh2 = Patm + ρHg gh3 − ρo gh2

Tambien se puede agregar a P5 como P5 = P4 − ρo gh2 para hacerlo más directo ponemos un menos ya
que vamos para arriba y hay menos presión.
Ahora la presión P6 es igual a P5 ya que estamos a la misma altura y no cambiamos de fluido. P6 = P5
Para obtener P1 , P6 es la presión de la interfase de agua-aceite, entonces subimos una altura h1 desde el
nivel de referencia en el agua

P1 = P6 − ρa gh1 = Patm + ρHg gh3 − ρo gh2 − ρa gh1

Evaluando con los datos3

P1 = 85600 + (13600)(0.35)(9.81) − (850)(9.81)(0.2) − (1000)(9.81)(0.1) = 129647 Pa ≈ 130 kPa


3
La presión atmosférica no es el dato usual, que el problema dice que están a una altura de 1400 m y la presión atmosférica
cambia con la altura

16
CAPÍTULO 1. ESTÁTICA DE FLUIDOS

La presión P1 es P1 = 130 kPa.

Problema de Desafío
Problema: Nivel del Agua
Insipirado en el video de Physics Girl: Can you solve the boat puzzle? 4 .
Imagine que usted sentado está en un bote flotando en el agua soteniendo una piedra. Usted tira esta
piedra y la piedra se hunde. El nivel del agua aumentó, disminuyó o se mantuvo igual?
Suponga que es una piscina (Digase, una cantidad de agua finita, una piscina tiene una área de base finita,
no es como un oceano donde la cantidad de agua es “infinita").
En el video, Dianna Cowern lo resuelve de una manera muy conceptual. La manera conceptual como lo
resuelve sirve para muchos tipos de problemas, no solo de fluidos en específico. Pensar como se compor-
tan los casos extremos. Corrobore el resultado de Dianna con la matemática. Suponga lo necesario para
resolverlo.5

Solución
Digamos que la piscina tiene área A y un volumen V de agua. Los 3 estados del problema se muestran
en la Fig. 1.17, la piscina solo con agua, cuando está en la piscina con la piedra antes de lanzar la roca al
piedra y después de arrojarla.

Figura 1.17: Niveles del agua en las 3 etapas.


Problema: Nivel del Agua

Inicialmente hay una altura h0 = V /A de agua en la piscina. Sea M la masa del bote y la persona, mp , ρp
y Vp la masa, densidad y volumen de la piedra y ma la masa de agua de la piscina. Si la piedra se hunde
significa que la densidad de la piedra es mayor que la del agua. ⇒ ρa < ρp Cuando se pone el bote
sosteniendo la piedra, va a haber una volumen sumergido Vi del bote dentro del agua para que el bote se
mantenga a flote. Encontramos esa volumen haciendo sumatoria de fuerzas.
X
Fi = ρa Vi g − (M + mp ) g = 0

1
⇒ Vi = (M + mp )
ρa
4
Link al video: https://www.youtube.com/watch?v=nVT1c0tr8NE
5
En el mundo real, no les van a dar variables, usted tiene que saber que es importante y que no para considerarlo cuando
resuelve un problema.

17
CAPÍTULO 1. ESTÁTICA DE FLUIDOS

Obtenemos la nueva altura hi de la columna de agua


V0 + Vi V0 1 1
hi = = + (M + mp )
A A A ρa
V0 1M 1 mp
⇒ hi = + + (1.1)
A A ρa A ρa
Para el estado donde la piedra está en el fondo, tenemos 3 volumenes que contribuyen a la altura. El
volumen de agua, todo el volumen de la piedra y el nuevo volumen sumergido Vf del bote en el agua. El
volumen Vf lo encontramos haciendo sumatoria de fuerzas
X M
Fi = ρVf g − M g = 0 ⇒ Vf =
ρa

Y cambiamos el volumen de la piedra con Vp = mp /ρp en la expresión de la nueva altura hf , que es

V0 + Vf + Vp V0 1M 1 mp
hf = = + + (1.2)
A A A ρa A ρp

Ocupamos comparar entre hi y hf de las Ec. 1.1 y 1.2 obtenidas para determinar cual es mayor. Pongamos
≶ y veamos cual respuesta es consistente.
?
hi ≶ hf (1.3)
V0 1M 1 mp ? V0 1M 1 mp
⇒ + + ≶ + + (1.4)
A A ρa A ρa A A ρa A ρp
Cancelamos términos
mp ? mp
⇒ ≶
ρa ρp
?
⇒ ρp ≶ ρa
?
Vemos que el signo tiene que ser ≶⇒>. Volviendo a la Ec. 1.3, tiene que ser

hi > hf

El nivel del agua baja cuando se tira la piedra al agua. Se obtiene el mismo resultado que obtuvo Dianna.
Usando el caso límite de una piedra de agujero negro (ρp → ∞) con masa constante, la comparación de
la Ec. 1.4 se vuelve trivial
V0 1M 1 mp ? V0 1M 1 mp
lı́m + + ≶ + +
ρp →∞ A A ρa A ρa A A ρa A ρp

V0 1M 1 mp ? V0 1M
+ + ≶ +
A A ρa A ρa A A ρa
1 mp ?
⇒ ≶0
A ρa
Vemos que tiene que ser el signo > ya que todas las cantidades son positivas.

18
2 | Oscilador Armónico Simple

[Resnick] Ejercicio 17-13


I Parcial 2011 II Ciclo Pregunta 1
Un oscilador es un bloque conectado a un resorte (k = 456 N/m). En algún momento t, la posición (medida
desde el lugar de equilibrio), la velocidad y la aceleración del bloque son x0 = 0.112 m, vx0 = −13.6 m/s,
ax0 = −123 m/s2 . Calcule (a) La frecuencia, (b) la masa del bloque, (c) la amplitud de la oscilación.
Solución
(b) Planteamos la ecuación de movimiento del resorte

−kx
max = −kx ⇒ m =
ax
La ecuación cumple para todos los puntos, entonces podemos evaluarlo en los puntos x0 con su vx0 y ax0
respectivos.
(456)(0.112)
m=− = 0.415 kg
(−123)
(a) Con la masa, ahora calculamos la frecuencia
r r r
k 1 k 1 456
ω= = 2πf → f = = = 5.27 Hz
m 2π m 2π 0.415
(c) Usamos la energía total del sistema
1 1 1
E = kA2 = kx20 + mv02
2 2 2
Y despejamos la amplitud A
r r
m 0.415
A = x20 + v02 = (0.112)2 + (−13.6)2 = 0.425 m
k 456

[Resnick] Problema 17-3, [Sears] Problema 14-42, [Serway] Problema 15-65


Reposición I Parcial IIC 2015 Problema 1
Dos bloques (m = 1.22 kg y M = 8.73 kg) y un resorte (k = 344 N/m) están dispuestos sobre una
superficie horizontal y sin fricción, como se muestra en la Fig. 2.1. El coeficiente de fricción estática
entre ellos es 0.42. Determine la máxima amplitud posible del movimiento armónico simple, si no ocurre
resbalamiento entre los bloques.

19
CAPÍTULO 2. OSCILADOR ARMÓNICO SIMPLE

Figura 2.1: [Sears] Problema 14-42

Solución
Analicemos primero el bloque de arriba. El bloque experimenta 3 fuerzas, la normal que le hace el bloque
de abajo N , la fuerza de gravedad mg y la fuerza de fricción. Ya que nos están pidiendo el caso de amplitud
máxima y no ocurre resbalamiento, esto corresponde al caso de aceleración máxima (Ya casi se plantea
eso), por lo que sabemos que la friccón va a ser máxima, que es la única fuerza horizontal que empuja al
bloque m para que se mueva en un M.A.S. La fricción va a tener el valor µs N .

Figura 2.2: Diagrama de fuerzas en m.


[Sears] Problema 14-42

Haciendo sumatoria de fuerzas tenemos


X
Fy,i = N − mg = 0 ⇒ N = mg
i
X
Fx,i = µs N = µs mg = max,max ⇒ ax,max = µs g
i
Ya tenemos cuanto es el máximo de aceleración. Relacionemos esto con la aceleración máxima
p del M.A.S
de todo el sistema. El M.A.S. es de la forma x(t) = A cos(ωt + φ), pero este omega es ω = k/(m + M ),
las suma de las 2 masas, ya que las 2 masas experimentan el M.A.S.. Derivando

d2 x
x(t) = A cos(ωt + φ) ⇒ = ax (t) = −ω 2 A cos(ωt + φ)
dt2
El máximo de las funciones sen y cos es el valor que está afuera de ellas. En este caso sería ax,max = Aω 2 .
De aquí relacionamos con la aceleración máxima obtenida por la fricción
 
2 k µs g(m + M )
ax,max = µs g = Aω = A ⇒A=
m+M k
Evaluando con los datos
(0.42)(9.8)(1.22 + 8.73)
A= = 0.119 m
344

20
CAPÍTULO 2. OSCILADOR ARMÓNICO SIMPLE

[Resnick] Ejercicio 17-17, [Sears] Problema 14.76, [Serway] Problema 15-81


Ampliación 2017 I Ciclo Problema 1
Un madero cilíndrico está cargado con plomo en un extremo, de modo que flota verticalmente en el agua.
La longitud de la parte sumergida es L = 2.56 m. Lo ponemos en oscilación vertical. (a) Demuestre que
la oscilación es armónica simple. (b) Encuentre el periodo de oscilación. No tenga en cuenta el hecho de
que el agua tiene un efecto amortiguador en el movimiento

Figura 2.3: [Resnick] Ejercicio 17-17

Solución
(a) Voy a usar una variables aunque no me los den, siempre y cuando en el alguna parte del procedimiento
logre independizarme de ellos. Digamos que el cilindro tiene un área A. El volumen inicial sumergido en
el agua es V0 = AL

Figura 2.4: Volumen del cilindro


[Resnick] Ejercicio 17-17

Y relacionemos la fuerza boyante con este volumen. El cilindro está en equilibrio, aún no está oscilando,
la sumatoria de fuerzas suma 0. X
Fi = ρVo g − mg = 0 (2.1)
i

Donde ρ es la densidad del agua.

21
CAPÍTULO 2. OSCILADOR ARMÓNICO SIMPLE

Ahora saquemos del equilibrio a ese cilindro. Movamos el cilindro una distancia q, definiendo el q positivo
para arriba.

Figura 2.5: Volumen del cilindro desplazado


[Resnick] Ejercicio 17-17

El volumen sumergido ahora es V 0 = A(L − q). Ya no está en equilibrio por que tiene menos volumen
sumergido, tiene menos fuerza boyante y la fuerza de gravedad sigue siendo la misma, por lo que el cilindro
es acelerado hacia abajo.
De aquí ya podemos escribir la ecuación de movimiento.
X d2 q
Fy,i = m = ρV 0 g − mg = ρA(L − q)g − mg = −ρAgq − ρALg − mg
dt2
i

Reconociendo que V0 = AL
d2 q
m = −ρAgq +ρV0 g − mg = −ρAgq
dt2 | {z }
=0
Sabemos que esa parte da 0, que es la ecuación 2.1 obtenida cuando el cilindro está en equilibrio. Reaco-
modando términos

d2 q


ρAg
+ q=0
dt2 m
| {z }
ω2
Ya que la ecuación de movimiento tiene la forma de un M.A.S., es un armónico simple.

(b) Con esto, tenemos la expresión para ω 2 , ahora eliminamos términos reacomodando la ecuación 2.1
ρAg L
ρV0 g − mg = ρALg − mg = 0 ⇒ =
m g

s
L L 2π
⇒ ω2 = ⇒ ω = =
g g T
r
g
⇒ T = 2π
L

22
CAPÍTULO 2. OSCILADOR ARMÓNICO SIMPLE

Esta es el periodo de oscilación con la que el cilindro se mueve hacia arriba-abajo.


Evaluando con los datos r
9.8
T = 2π = 12.29 s
2.56
ADVERTENCIA: Esto de pura casualidad tiene la misma forma que el péndulo simple. Es pura coin-
cidencia que se vea tan igual, pero el como se derivó es distinto.1

[Sears] Problema 14-11


Un bloque de 2.00 kg, que resbala sin fricción, se conecta a un resorte ideal con constante de fuerza de
300 N/m en t = 0, el resorte no está estirado ni comprimido, y el bloque se mueve en la dirección negativa
de 12 m/s. Calcule (a) la amplitud y (b) el ángulo de fase. (c) Escriba una ecuación para la posición en
función del tiempo.

Solución
El bloque se mueve en la dirección negativa de 12 m/s. Eso significa que la velocidad inicial es v0 = −12
m/s, tiene un menos. La posición y velocidad son vectores. Estamos en una dimensión, pero igual se
tienen que respetar los signos. Que no esté estirado ni comprimido significa que x0 = 0m. Derivemos
como encontrar el ángulo y la fase.
Si x(t) es de la forma2 x(t) = A cos(ωt + φ)

dx
x(t) = A cos(ωt + φ) ⇒ v(t) = = −ωA sin(ωt + φ)
dt
Evaluando en t = 0, la posición y velocidad quedan como

x(t = 0) = x0 = A cos(φ) (2.2)


v0
v(t = 0) = v0 = −ωA sin(φ) ⇒ − = A sin(φ) (2.3)
ω
Para obtener φ, dividamos la Ec. 2.3 entre la Ec. 2.2.

v0 A sin(φ)
− = = tan(φ)
ωx0 A cos(φ)
 
v0
φ = arctan − (2.4)
ωx0
Ahora encontremos la amplitud A. Elevemos al cuadrado las Ec. 2.2 y 2.3
 v 2
0
x20 + − = A2 cos2 (φ) + A2 sin2 (φ) = A2
ω
r
v2
⇒ A = x20 + 02 (2.5)
ω
1
Revisé una ampliación que usaban la fórmula del péndulo simple para obtener la respuesta, sin ningun otro procedimiento
mas que colocar la fórmula... Aunque esté “buena”, el procedimiento esta mal con decir “Aquí aplica la ecuación del péndulo
simple”. No es un péndulo simple.
2
Si x(t) tuviera otra forma, como x(t) = A sin(ωt ± φ) o x(t) = A cos(ωt − φ), las ecuaciones 2.4 y 2.5 varían la forma un
poco, así que mejor derivemola.

23
CAPÍTULO 2. OSCILADOR ARMÓNICO SIMPLE

(a) y (b) Encontremos los valores de ω, φ y A.


r

r
k 300
ω= = = 5 6 s−1 = 12.24 s−1
m 2
s √
2
(−12)2 2 6
A = (0) + √ 2 = m = 0.98 m
5 6 5

(c) Solo que para φ vamos a tener un problema, evaluamos con el x0 = 0


!
(−12)
φ = arctan − √  = arctan(∞)
5 2 (0)

Abusando de la notación, invirtamos para expresar con tan en lugar de arctan

sin(φ)
tan(φ) = =∞
cos(φ)

En ángulo que hace que esto cumple, es hacer que φ = π/2 para que el cos sea 0. Entonces φ = π/2...
Verdad? Pero también φ = −π/2. Y aunque de un infinito positivo (por el menos y el menos de la v0 ), si
uno sacara el límite de x0 → 0 da diferente sacarlo por la izquierda o derecha, ahí cambiaría el signo, y se
podría obtener ±∞. Ocupamos una manera de determinar el signo de φ = ±π/2. Usemos las condiciones
iniciales. Sabemos que la velocidad tiene signo negativo. Usemos la Ec. 2.3

v0 = −ωA sin(φ) < 0

Quitando a ω y A que sabemos que son positivo, ocupamos

− sin(φ) < 0

Viendo el signo de las dos opciones de φ tenemos


π   π
− sin <0 − sin − >0
2 2
Con esto vemos que φ = π/2 para cumplir con la condición inicial.
Ya tenemos todos los datos para escribir x(t). La posición es
√  √
2 6 π  π
x(t) = cos 5 6t + m = 0.98 cos 12.24t + m
5 2 2

En caso que hubiera sido condición inicial para algún x0 y v0 = 0, hay que usar las condiciones iniciales
para determinar el signo de φ.

[Sears] Problema 14-12


Repita el problema [Sears] Problema 14-11, pero suponga que en t = 0 el bloque tiene una velocidad
de -4.00 m/s y un desplazamiento de 0.200 m.
Este es el mismo tipo de problema de condiciones inciales. Solo que este caso es bien portado. Notese que
el ω sigue siendo el mismo (La masa m y la constante de resorte k siguen siendo las mismas). La frecuencia

24
CAPÍTULO 2. OSCILADOR ARMÓNICO SIMPLE

ω de un sistema no depende de las condiciones inciales, si no de parametros de como está construido el


sistema.3 Lo que depende de las iniciales son el φ y el A. que la Usando las Ec. 2.4 y 2.5
s r
2
(−4.00)2 1 11
A = (0.200) + √ 2 = m = 0.38 m
5 6 5 3
!
−4.00
φ = arctan − √  = 1.02 rad
5 6 (0.2)
La posición x(t) es
x(t) = 0.38 cos (12.24t + 1.02) m

[Irodov] Problema 4-20


Problema generalizado del I Parcial 2013 II Ciclo Pregunta 1
Una bola está suspendida de un hilo de longitud L en el punto O en el muro, que forma un ángulo
pequeño α con la vertical. La bola es desviada un ángulo pequeño β (β > α) y se deja libre. Asumiendo
que la colisión de la bola contra el muro es completamente elástica, encuentre el periodo de oscilación del
péndulo.
Extra: Chequee la respuesta con el caso límite α → β

Figura 2.6: [Irodov] Problema 4-20

Solución
Sea T0 y ω0 el periodo y frencuencia angular del péndulo como si no tuviera el muro.
s r
L g
T0 = 2π ω0 =
g L

El péndulo de β → 0 tarda T0 /4. Luego de 0 → α tarda un tiempo t0 que vamos a determinar. Ya que
la bolita rebota de manera elástica, va a salir con la misma velocidad con la que pego y va a tardar de
α → 0 el mismo tiempo t0 . Luego de 0 → β va a tardar T0 /4
El periodo total de ese péndulo es
T0 T0 T0
T = + t0 + t0 + = + 2t0
4 4 2
3
Por lo menos los sistemas que se describen por sistemas de ecuaciones diferenciales lineales. Un contra ejemplo de un
sistema que el periodo depende de las condiciones inciales, es un péndulo sin la aproximación de ángulo pequeño. Vease el
[Resnick] página 382

25
CAPÍTULO 2. OSCILADOR ARMÓNICO SIMPLE

Figura 2.7: Tiempo de cada sección


[Irodov] Problema 4-20

Para encontrar el tiempo t0 , demonos cierta libertad. No nos está pidiendo encontrar la ecuación de
movimiento θ(t) y no nos están dando condiciones inciales, ya que para cualquier condición inicial (Siempre
que cumpla que llegue hasta β), el periodo va a ser el mismo. Con esa libertad, “iniciemos” el movimiento
como si la bolita estuviera en θ = 0 y va hacia el muro hasta el ángulo α. Hasta antes de chocar, se
comporta como un péndulo sin el muro, va a tener la misma ecuación de movimiento hasta ese momento
t0 cuando choca.
θ(t) = β sin(ωo t) en el intervalo 0 < t < t0
Para 0 < t < t0 , tomando el θ positivo hacia la dirección de α. De está ecuacón, en θ(t0 ) va a cumplir
θ(t0 ) = α y despejar el tiempo t0
 
1 α
θ(t0 ) = α = β sin(ωo t0 ) ⇒ t0 = arcsin
ω0 β

El periodo del péndulo es s  


T0 1 L L α
T = + 2t0 = 2π + 2 arcsin
2 2 g g β
   s   
1 1 α L 1 1 α
= + arcsin 2π = + arcsin T0 = T
2 π β g 2 π β
Ese es el periodo de ese péndulo.
Podemos darnos confianza de este resultado verificando el límite cuando el muro se encuentre a un ángulo
β. Cuando está a β, la colisión va a ser con rapidez 0, es como si no estuviera ahí. Deberíamos obtener
T → T0 en el límite α → β. Usando que arcsin(α/β) → π/2 cuando α → β
    
1 1 α 1 1π
lı́m T = lı́m + arcsin T0 = + T0 = T0
α→β α→β 2 π β 2 π2

Se obtiene el resultado esperado. Es bueno chequear límites de casos simples conocidos siempre que se
pueda para corroborar.

[Sears] Problema 14-99

26
CAPÍTULO 2. OSCILADOR ARMÓNICO SIMPLE

Dos varillas delgadas idénticas, cada una con masa m y longitud L, se unen en ángulo recto para construir
un objeto en forma de L, el cual se balancea sobre la cúspide de un triángulo agudo. El objeto en forma
de L oscila cuando se desvía un poco. Calcule la frecuencia de oscilación

Figura 2.8: [Sears] Problema 14-99

Solución
La frecuencia angular de oscilación es r
mgd
ω=
I
Donde m es la masa total del sistema, d la distancia del eje de rotacion al centro de masa y I la inercia
rotacional del sistema. Aqui tenemos 2 varillas. La masa total es 2m, entonces en la fórmula evaluamos
con m → 2m. Calculemos la distancia d, que es d = |Rcm |, donde Rcm es el vector hacía el centro de
masa con origen en el eje de rotación.
Por definición !
X X
mi Rcm = mi ri
i i

De la figura 2.9, vemos


L π  L π  L 1  
r1 = cos î − sin ĵ = √ î − ĵ
2 4 2 4 2 2
L π  L  π  L 1  
r2 = − cos î − sin ĵ = √ −î − ĵ
2 4 2 4 2 2

Sumando
X L
mi ri = −m √ ĵ
i
2
!
X L L
⇒ mi Rcm = 2mRcm = −m √ ĵ ⇒ Rcm = √ ĵ
i
2 2 2
√ 
Ya aquí obtenemos la magnitud de Rcm , que es d = |Rcm | = L/ 2 2

27
CAPÍTULO 2. OSCILADOR ARMÓNICO SIMPLE

Figura 2.9: Posición de los centros


[Sears] Problema 14-99

Lo último que nos falta es la inercia rotacional I. La inercia rotacional de una barra respecto al centro de
masa es Icm = (1/12)M L2 . El eje de rotación está a L/2 del C.M. de cada barra, entonces tenemos que
trasladarlo con el teorema de ejes paralelos, y tenemos 2 barras, entonces sumamos las I de cada barra
 2 !  2 !
1 2 L 1 2 L 2
I= mL + m + mL + m = mL2
12 2 12 2 3
| {z } | {z }
Barra 1 Barra 2
Ya teniendo todo lo necesario, metemos en la fórmula de ω
v  
L
u

u
r u (2m) g s
mgd u 2 2 3 g
ω= =u = √ (2.6)
I 2 2 2L
mL2
t
3
El problema pide la frecuencia f , entonces obtenemos a f con
s s
1 1 3 g 1 6 g
f= ω= √ = √
2π 2π 2 2 L 4π 2L

Método 2
En lugar de usar la fórmula directa de ω, hagamolo con torques. El sistema está en equilibrio cuando está
como en la figura 2.9, cuando los lados de la L están a π/4 del eje x ( o del eje y también). Perturbemolo
un ángulo q del equilibrio como se muestra en la figura 2.10. Ignorando la fuerza que hace el pico por que
no hace torque4 , saquemos la torsión que hace la fuerza de gravedad de carra barra. Cada vector mg se
ubica en el C.M. de cada barra. La torsión positiva usandola en el sentido contrario a las manecillas del
reloj, obtenemos

X L π  L π 
τi = mg sin − q − mg sin +q
2 4 2 4
i
4
Es 0 ya que el vector r es r = 0 del pico al punto de rotación, r × F = 0

28
CAPÍTULO 2. OSCILADOR ARMÓNICO SIMPLE

Figura 2.10: Perturbando un ángulo q


[Sears] Problema 14.99

Ahora usemos la aproximación de ángulo pequeño. Tenemos que expandir en Taylor el sin, solo que este
sin está centrado en π/4, entonce no es la serie usual de sin(θ) ≈ θ. Derivemos hasta primer orden la serie.
La serie de Taylor hasta primer orden es

df (x)
f (x0 + ∆x) ≈ f (x0 ) + (∆x)
dx x=x0

Aplicandolo al sin(θ) centrado en θ = π/4


π π 
 d sin(θ) π  π 
sin + q ≈ sin +q = sin + q cos
4 4 dθ θ=π/4 4 4

Reescribiendo el torque
X L h π   π i L h π   π i π 
τi ≈ mg sin − q cos − mg sin + q cos = −mgL cos q
2 4 4 2 4 4 4
i

X mgL
⇒ τi = − √ q
i
2
Escribiendo la ecuación de movimiento, usando la inercia rotacional obtenida en la primera parte, I =
2/3M L2 .
d2 q 2 d2 q X mgL
I 2 = M L2 2 ⇒ τi = − √ q
dt 3 dt 2
i

2 d2 q
mgL
M L2 2 = − √ q
3 dt 2
Reacomodando
d2 q
 
3 g
⇒ 2 + √ q=0
dt 2 2L
| {z }
ω2
Está es la ecuación 2.6 obtenida con el otro método.
[Serway] Problema 15-67

29
CAPÍTULO 2. OSCILADOR ARMÓNICO SIMPLE

Un péndulo de longitud L y de masa M tiene un resorte con constante de fuerza k conectado a él a una
distancia h bajo su punto de suspensión. Encuentre la frecuencia de vibración del sistema para pequeños
valores de la amplitud (θ pequeño). Suponga que la barra de suspensión vertical de longitud L es rígida,
pero ignore su masa.
Extra: Chequee la respuesta con el caso límite k → 0.

Figura 2.11: [Serway] Problema 15-67

Solución
p
En este problema no podemos aplicar la fórmula del péndulo físico ω = mgd/I por que tenemos una
fuerza externa, la fuerza del resorte. Esta fórmula solo aplica cuando la fuerza de gravedad es la única
fuerza que hace torque respecto al punto de rotación. Hay que plantear el problema haciendo suma de
torques
d2 θ X
I 2 = τi
dt
i

Movamos el péndulo un ángulo θ pequeño y hagamos un diagrama de cuerpo libre (Figura 2.12)
Sea θ positivo la dirección hacia cual se movió en el diagrama. La fuerza de gravedad hace una torsión
de τ = −mg sin(θ)L. El menos por que va en la dirección contraria al θ positivo. Ahora obtengamos el
torque del resorte. Sea x la distancia que se estira el resorte. La fuerza en el resorte es kx. La torsión que
hace el resorte es −kxh cos(θ). El h cos(θ) sale por que ahora está a esa distancia de la vertical y el menos
por que va en la dirección contrario a θ positivo. La inercia rotacional, ya que es una masa puntual es
I = M L2
d2 θ X
I 2 = τi
dt
i

d2 θ
⇒ M L2 = −mg sin(θ)L − kxh cos(θ)
dt2
Esto no nos va a dar un M.A.S. pero si aproximamos a primer orden, el movimiento se aproxima al del
M.A.S. Usando5 sin(θ) ≈ θ y cos(θ) ≈ 1.
La distancia x es h sin(θ). Pero si nos mantenemos en ángulos pequeños, x se aproxima a hθ
5
http://www.smbc-comics.com/comic/2012-03-13

30
CAPÍTULO 2. OSCILADOR ARMÓNICO SIMPLE

Figura 2.12: Diagrama de cuerpo libre


[Serway] Problema 15-67

Figura 2.13: Aproximación de x en θ pequeño


[Serway] Problema 15-67

Reescribiendo, la ecuación con los torques nos quedaría


d2 θ
M L2= −mgθL − kh2 θ
dt2
Reacomodando términos para que nos quede en el formato de la ecuación diferencial de un M.A.S
d2 θ mgL + kh2
 
+ θ=0
dt2 mL2
| {z }
ω2
De ahí identificamos el ω 2 que es
mgL + kh2
ω2 =
mL2
Para corroborar que se hizo bien, obtengamos el caso límite donde k → 0, que es como si quitara el resorte.
Debería obtener recuperar la frecuencia de un péndulo simple
mgL + kh2 g
lı́m ω 2 = lı́m 2
=
k→0 k→0 mL L

31
CAPÍTULO 2. OSCILADOR ARMÓNICO SIMPLE

Recuperamos la frecuencia de un péndulo simple en el límite.

Problema de desafío
[Irodov] Problema 4-28
Una barra uniforme se coloca encima de 2 ruedas que están girando como se muestra en la figura 2.14. El
eje de las ruedas están separadas a una distancia L y el coeficiente de fricción cinético entre la barra y las
ruedas es µ. Demuestre que la barra se mueve en un movimiento armónico simple. Encuentre el periodo
de oscilación del movimiento.
Sugerencia: Haga la suma de fuerzas tanto en la dirección vertical como la horizontal para relacionarlo
con la aceleración y haga sumatoria de torques respecto al C.M. de la barra, con esas ecuaciónes ya puede
probar que es un M.A.S y encontrar la frecuencia respectiva.

Figura 2.14: [Irodov] 4-28

Solución
No nos dan ni la velocidad angular de las ruedas ni la masa de la barra. Sea m la masa de la barra. Voy
a usar la masa y tengo que ocuparme que mi resultado final no depende de m.
Realicemos el diagrama de fuerzas, que es como se muestra en la Fig. 2.15

Figura 2.15: Diagrama de fuerzas.


[Irodov] 4-28

Sea x(t) la posición del centro de masa de la barra respecto al centro de las 2 ruedas. El sistema no va a

32
CAPÍTULO 2. OSCILADOR ARMÓNICO SIMPLE

estar en equilibrio, si puede acelerar en la coordenada x. La sumatoria de fuerzas en el eje horizontal es


X
Fx,i = µN1 − µN2 = max = max
i

d2 x(t)
m = µN1 − µN2 (2.7)
dt2
Donde podemos asegurar que fuerza de fricción va a ser µN ya que la barra está resbalando sobre cada
rueda. Se ocupa saber con que velocidad giran las ruedas siempre y cuando resbale.
Hagamos la sumatoria de fuerzas en y. En y no está acelerando, las fuerzas suman 0.
X
Fy,i = N1 + N2 − mg = 0

⇒ N1 + N2 = mg (2.8)
Además, sabemos que el sistema no está girando, la suma de los torque respecto al centro de masa tiene
que ser 0    
X L L
τi = N2 − x(t) − N1 + x(t) = 0 (2.9)
2 2
i

Donde las fuerzas de fricción no hacen torque porque van en la dirección al centro de masa.
Queremos eliminar N1 y N2 de las Ec. 2.7, 2.8 y 2.9 y obtener la forma de la ecuación diferencial de un
M.A.S.. Le las Ec. 2.8 y 2.9 despejamos N1 y N2
      
1 1 N1 mg N1
= ⇒
− L2 − x(t) L
2 − x(t) N 2 0 N2

La solución es
L−2x
 
  mg 2L
N1
= 
N2 L+2x

mg 2L
Usando estos resultados en la Ec. 2.7 obtenemos
d2 x(t)
   
L − 2x(t) L + 2x(t) 2mgµ
m = µmg − µmg = x(t)
dt2 2L 2L L

d2 x(t) 2gµ
+ x(t) = 0
dt2 L
Ya que la ecuación diferencial de la barra es la de un M.A.S, la barra se mueve en un M.A.S, y vemos que
la frecuencia angular es r
2 2gµ 2mgµ
ω = ⇒ω=
L L
Entonces el periodo es s
2π L
T = = 2π
ω 2gµ

33
3 | Movimiento Ondulatorio

I Parcial 2017 I Ciclo Problema 3


Parecido al [Knight] Problema 16-58
Una cuerda de longitud L = 30 cm, de masa mc = 5 × 10−3 kg tiene atada al punto A y B una masa
M = 0.5 kg. La distancia entre las paredes es D = 25.6 cm. Desprecie la contribución de la masa de
la cuerda en el cálculo de las tensiones y determine: a) las tensiones en cada uno de los segmentos de
la cuerda (1), (2) y (3), b) el tiempo que le tomaría a un pulso generado en el extremo izquierdo de la
cuerda (1) recorrer toda la cuerda compuesta, es decir, los tres segmentos de la cuerda. c) La fracción
del tiempo total calculado en la parte b) que representa el tiempo que le toma al pulso recorrer cada
segmento de la cuerda.

Figura 3.1: I Parcial 2017 I Ciclo Problema 3

Solución
(a) Encontremos las tensiones en los cables. Ocupamos averiguar el ángulo θ que hace las cuerdas. La

Figura 3.2: Distancias


I Parcial 2017 I Ciclo Problema 3

34
CAPÍTULO 3. MOVIMIENTO ONDULATORIO

proyección horizontal de la cuerda es igual a D, igualamos


L L L L
cos(θ) + + cos(θ) = D ⇒ [1 + cos(θ)] = D
4 2 4 2
   
2D 2(25.6)
θ = arc cos − 1 = arc cos − 1 = 0.786 rad = 45◦
L (30)
Sacamos las tensiones usando que el sistema está en equilibrio

Figura 3.3: Diagrama de fuerzas.


I Parcial 2017 I Ciclo Problema 3

X Mg (0.5)(9.8) (0.5)(9.8)
Fy,i = T1 sin(θ) − M g = 0 ⇒ T1 = = ◦
=   = 6.93 N
sin(θ) sin(45 ) 1
i √
2
X 6.93
Fx,i = T2 − T1 cos(θ) = 0 ⇒ T2 = T1 cos(θ) = (6.93) cos(45◦ ) = √ = 4.89 N
i
2
La tensión T3 en el cable de la 3 es T3 = T1 ya que es simétrico. El sistema se ve igual si le damos la
vuelta.
(b) La velocidad varía en cada sección de la cuerda ya que la tensión varía, para k = 1, 2, 3, la velocidad
en la k-ésima cuerda es s r
Tk Tk L
vk = =
µ mc
El tiempo que tarda la onda en viajar cada sección k es t = Lk /vk .
El tiempo total de viaje es
r r r
1 L 1 L 1 L L mc L mc L mc
tT ot = t1 + t2 + t3 = + + = + +
v1 4 v2 2 v3 4 4 T1 L 2 T2 L 4 T1 L
   
1p 1 1 1p 1 1
= mc L √ + √ = (0.005)(0.3) √ +√ = 0.0161 s
2 T1 T2 2 6.93 4.89
(c) Cada segmento del pulso le toma una fraccón del tiempo total fk = tk /tT ot
 
1 L r
r  
v1 4 1 mc L 1 1 (0.005)(0.3) 1
f1 = f3 = = = = 0.228
t 4 T1 tT ot 4 6.93 0.0161

35
CAPÍTULO 3. MOVIMIENTO ONDULATORIO

 
1 L r
r  
v2 2 1 mc L 1 1 (0.005)(0.3) 1
f2 = = = = 0.543
t 2 T2 tT ot 2 4.89 0.0161
Veamos si tiene sentido la respuesta, tomando la unión de los segmentos de la cuerda 1 y 3 como uno
solo, la onda gasta un f1 + f3 = 0.456 de la fraccón total, en el segmento 2 gasta 0.543. La onda viaja
más rápido en 1 y en 3 que en 2, entonces ocupan menos tiempo en 1+3 que en 2, por lo que representa
una fraccón más pequenña, lo que se ve en que comparando las fracciones f1 + f3 < f2 . Si nos hubiera
dado f2 < f1 + f3 , sabiendo que v2 < v1 , habríamos hecho algo malo en el procedimiento.

[Resnick] Ejercicio 18-7


La ecuación de una onda transversal que se desplaza por una cuerda muy larga está dada por

y(x, t) = 6.0 sin [(2.0π rad/m)x + (4.0π rad/s)t] cm

Calcule a) la amplitud, b) la longitud de onda, c) la frecuencia, d) la velocidad, e) la dirección de pro-


pagación de la onda y f ) la velocidad transversal máxima de una partícula de la cuerda.

Solución
Esto es un problema de indentificar rápido los términos. Una onda viajera tiene la forma y(x, t) =
A sin(kx ± ωt + φ). Comparando con la ecuación dada
 

y(x, t) = (6.0 cm) sin (2.0π rad/m) x + (4.0π rad/s) t


 
| {z } | {z } |{z} | {z }
A k Izquierda ω

De aquí vemos que (a) La amplitud es A = 6.0 cm. (b) La longitud de onda λ la sacamos con λ = 2π/k =
2π/(2π) m = 1 m. (c) La frencuencia f es f = ω/(2π) = 4π/(2π) = 2 Hz. (d) La velocidad la podemos
sacar de 2 formas con v = ω/k o v = λf .
ω 4π
v= = = 2 m/s
k 2π
v = λf = (1)(2) = 2 m/s
(e) La dirección de propagación la vemos en el kw ± ωt. Esta onda tiene kx + ωt, es + indica que va hacia
la izquierda. (f ) La velocidad máxima de la onda la obtenemos con
∂y
vy (x, t) = = ωA cos(kx + ωt)
∂t
El máximo en una función cos/sen es el factor de afuera. Aquí el máximo ωA

⇒ vy,max = ωA = (4π)(6.0) = 24π cm/s

[Serway] Problema 16-39


La función de onda para una onda sobre una cuerda tensa es
 π
y(x, t) = 0.350 sin 10πt − 3πx +
4

36
CAPÍTULO 3. MOVIMIENTO ONDULATORIO

donde x y y están en metros y t en segundos. Si la densidad lineal de la cuerda es 75.0 g/m, (a) cual
es la rapidez promedio a la que se transmite la energía a lo largo de la cuerda? (b) Cual es la energía
contenida en cada ciclo de la onda?

Solución
Antes de empezar, identifiquemos parámetros como el problema anterior. A = 0.350 m, k = 3π m−1 ,
ω = 10π s−1 .
(a) La primera parte está preguntando por la potencia promedio. La potencia es P = (1/2)µω 2 A2 v. La
velocidad no nos la dan explicitamente, pero la averiguamos con v = ω/k. Ya tenemos todos los datos
para evaluar  
1 ω  1 10π
P = µω 2 A2 = (0.075)(10π)2 (0.35)2 = 15.11 W
2 k 2 3π
(b) La energía promedio la obtenemos con E = P T , donde T es el tiempo con que se promedia la potencia,
que sería el periodo de la onda, que obtenemos con T = 2π/ω
   
2π 2π
E = PT = P = (15.11) = 3.02 J
ω 10π

I Parcial 2015 II Ciclo Problema 2


Una onda sinusoidal transversal en una cuerda tiene un periodo T = 25 ms y viaja en la dirección x
negativa con una velocidad de 30 m/s. En t = 0 y x = 0, una partícula en la cuerda tiene una posición
transversal de 2 cm y viaja con una velocidad de 2 m/s, determine: a) la amplitud de la onda, b) el
ángulo de fase, c) la máxima velocidad transversal de la cuerda, d) la función de onda para la onda.

Solución
No hagamos matemática ahorita, antes hagamos un bosquejo de como debería verse la onda para t = 0.
Nos dicen que en x = 0 está en una posición transversal 2 cm, distancia hacia arriba por el signo positivo
y la velocidad tambien es positiva. Sabemos que no estamos en lo más alto de la cuerda por que aún
tenemos velocidad. Debería verse algo asi Podemos averiguar cual de las opciones es usando la dirección

Figura 3.4: Posibles casos para las condiciones iniciales


I Parcial 2015 II Ciclo Problema 2

de viaje de la onda. Viaja hacia la izquierda, dejemos que la onda avanze un ∆t pequeño y volvamos a
dibujarlas Donde las líneas discontinuas representan la posición de la cuerda para un momento poquito
después. Vemos que para que cumpla que la velocidad en el punto x = 0, t = 0 sea hacia arriba, tiene
que ser el dibujo de la izquierda. Los puntos del lado izquierdo están más arriba. El de la derecha tiene

37
CAPÍTULO 3. MOVIMIENTO ONDULATORIO

Figura 3.5: Posibles casos un ∆t pequeño después


I Parcial 2015 II Ciclo Problema 2

condición incial con velocidad hacia abajo ya que el segmento de cuerda se fue hacía abajo. Usemos esto
para corroborar el resultado obtenido con matemática.

Figura 3.6: Forma de la onda.


I Parcial 2015 II Ciclo Problema 2

(a) Identifiquemos términos que vamos a usar. Sabemos el periodo T , entonces podemos encontrar ω con
2π 2π
ω= = = 80π s−1
T 0.025
Y podemos saber k con
ω ω 80π 8
v= ⇒k= = = π m−1
k v 30 3
Recordatorio: Esta v es la velocidad de viaje de la onda, no la velocidad transversal vy de los elementos
de la cuerda. No tienen por que ser iguales.
La onda va a ser de la forma  
8
y(x, t) = A sin πx + 80πt + φ
3
(a) Averiguemos A. Sea la posición
y(x, t) = A sin(kx ± ωt + φ) (3.1)
Y sea la velocidad
∂y vy
= vy (x, t) = ±ωA cos(kx ± ωt + φ) ⇒ ± A cos(kx ± ωt + φ) (3.2)
∂t ω
Elevando al cuadrado y sumando las Ec. 3.1 y 3.2
vy2
y2 + = A2 sin2 (kx ± ωt + φ) + A2 cos2 (kx ± ωt + φ) = A2
ω2

38
CAPÍTULO 3. MOVIMIENTO ONDULATORIO

s
vy2
⇒A= x2 +
ω2
También podemos usar esta ecuación en ondas viajeras senoidales1 . Evaluando para las condiciones ini-
ciales s
(0.02)2
A = (0.02)2 + = 0.0215 m
(80π)2
(b) Evaluamos en x = 0 y t = 0, tenemos
x   
0 0.02
x0 = A sin(φ) ⇒ φ = arcsin = arcsin = 1.195
A 0.0215

Esto φ nos la da la calculadora, pero al sacar el arcsin, debería dar tiene 2 respuestas que hay que verificar
con las condiciónes iniciales, vamos a usar que la velocidad es positiva para ver cual es. Sea φ1 = φ y
φ2 = π − φ Viendo el signo de φ1 y φ2 en la velocidad v(x = 0, t = 0) = ωA cos(φ), que tiene que ser

Figura 3.7: Posibles respuestas del φ


I Parcial 2015 II Ciclo Problema 2

positriva
cos(φ1 ) = cos(1.195) > 0 cos(φ2 ) = cos(π − 1.195) < 0
El ángulo que lo cumple es φ1 .
(d) La ecuación de la onda es
 
8
y(x, t) = 0.0215 sin πx + 80πt + 1.195 m (3.3)
3

(c) La velocidad transversal es vy (x, t) = ωA cos(kx + ωt + φ). La vy,max es el factor del coseno, que es
vy,max = ωA = (80π)(0.0215) = 5.40 m/s.
Para corroborar que da la misma forma que la onda de la izquierda de la figura 3.4, que obtuvimos usando
los conceptos, grafiquemos la ecuación 3.3 en t = 0 La figura 3.8 es la misma que la figura 3.6.
[Serway] Problema 16-31
Pulsos transversales viajan con una rapidez de 200 m/s a lo largo de un alambre de cobre tenso cuyo
diámetro es de 1.50 mm. Cuál es la tensión del alambre?
1
No aseguro que sirva para cualquier tipo de onda. Por eso lo probé. Mejor verificar antes de usarla sin estar seguro.

39
CAPÍTULO 3. MOVIMIENTO ONDULATORIO

Figura 3.8: Onda graficada en t = 0 s.


I Parcial 2015 II Ciclo Problema 2

La densidad del cobre es de 8.92 g/cm3

Solución
Sabemos que la velocidad, tensión y densidad lineal están relacionadas por
s
T
v= (3.4)
µ

Pero en los datos tenemos la densidad volumétrica, no podemos evaluar con esa fórmula aún. Ocupamos
relacionar la densidad lineal y la volumétrica. Hagamos un cilindro de grosor diferencial dx, pero de Área
A, esa no la hagamos diferencial.

Figura 3.9: Cilindro diferencial.


[Serway] Problema 16-31

El diferencial de masa dm es dm = ρ dV , pero a la vez es dm = µ dx. Si lograra pasar de alguna función


desde el ρ dV hasta algún f (x) dx, sea lo que sea vaya ahí en ese f (x), esa va a ser la densidad lineal µ.
Estamos usando un cilindro diferencial, entonces el volumen dV es dV = A dx. Sustituimos en la expresión
con la densidad ρ, lo cual se transforma en

ρ dV = ρA dx = (ρA) dx ⇒ µ = ρA
| {z }
µ

40
CAPÍTULO 3. MOVIMIENTO ONDULATORIO

Ya sabiendo cuando es µ, por que ρ y el área del cable la conocemos2 , seguimos con el problema usando
la Ec. 3.4, lo cual nos da
π π 
T = µv 2 = ρAv 2 = ρ d2 v 2 = (8920) (1.5 × 10−3 )2 (200)2 = 630.5 N
4 4

[Resnick] Problema 18-8, [Serway] Problema 16-60, [Knight] Problema 16-82


Una cuerda uniforme de masa m y de longitud L, cuelga del techo. a) Demuestre que la rapidez de una

onda transversal en ella es una función de y, la distancia del extremo inferior y que está dada por v = gy.
b) Demuestre que p el tiempo que una onda transversal tarda en recorrer la longitud de la cuerda, está
dada por ∆t = 2 L/g.

Solución
(a) Está es una cuerda que sí tenemos que considerar el peso de ella. No se puede decir de manera simple
v∆t = L, donde v es la velocidad de la onda, ∆t el tiempo que tarda, por que la velocidad de la onda
no es constante. La velocidad depende la tensión F y µ, pero µ es constante (Es una cuerda uniforme).
Si la tensión está cambiando en el medio, la velocidad de la onda no es constante. Ocupamos obtener la
velocidad onda.
Hagamos un diagrama de cuerpo libre de un segmento de cuerda desde abajo hasta una distancia y hacia
arriba. La masa hasta una distancia y es m0 = µy. Lo que pesa ese segmento de cuerda es m0 g = µyg Ya

Figura 3.10: Diagrama de cuerpo libre.


[Serway] Problema 16-60

que en la vertical está en equilibrio, tenemos F = µyg


s
µyg √
r
F
v= = = gy
µ µ
(b) Para obtener cuanto tarda, pongamos la definición general de velocidad.
dy
v=
dt
Aquí, la velocidad depende de la posición v = v(y). Mandamos el diferencial dt al otro lado e integramos.
dy dy dy
v = v(y) = ⇒ dt = =√
dt v(y) gy
2
Sabemos el diametro, entonces sabemos el área

41
CAPÍTULO 3. MOVIMIENTO ONDULATORIO

Y de aquí integramos a ambos lados


ˆ ∆t ˆ L
s
dy L
dt = ∆t = √ =2
0 0 gy g
s
L
⇒ ∆t = 2
g

[Knight] Problema 16-59


I Parcial 2017 II Ciclo Problema 4
Un cable fue hecho soldando 2 metales de diferentes densidades. La figura muestra una sección de 2 metros
de largo con la unión en el centro, pero el cable se extiende mucho más en ambas direcciones. El cable es
puesto bajo una tensión de T = 2500 N. Una onda con una frecuencia de 1500 Hz y una amplitud de 3
mm es enviada a lo largo del cable. Determine cuantas longitudes de ondas (ciclos completos) de la onda
caben en esta seccón de 2 metros del cable.3

Figura 3.11: [Knight] Problema 16-59

Solución
Hagamolo para cada medio a la vez con subíndice k, donde k = 1 para el metal de la izquierda k = 2 para
el de la derecha. Sea vk la velocidad en el medio
s
T
vk =
µk

Sabiendo la velocidad, usamos la relación vk = λk f . La frecuencia en ambos medios es la misma, por lo


que no escribo fk , solo f . Despejamos λk que es
s
vk 1 T
λk = =
f f µk

Sea nk el número de longitudes de onda que caben en el segmento de L = 1 m del cable en cada medio.
La cantidad de ondas que caben es r
L µk
nk = = Lf
λk T
Ya podemos evaluar para cada medio
r r
2.89 1.21
n1 = (1.00)(1500) = 51 n2 = (1.00)(1500) = 33
2500 2500
El número total de ondas completas es n1 + n2 = 51 + 33 = 84.
3
La tensión aquí fue cambiada para que den números más lindos.

42
4 | Ondas Sonoras

[Resnick] Problema 19-7


Cierto altavoz (supuestamente una fuente puntual) emite Pf 31.6 W de potencia acústica. Un pequeño
micrófono con una superficie transversal efectiva de A = 75.2 mm2 está situado a d = 194 m de él. Calcule
(a) La intensidad del sonido en el micrófono. (b) la potencia incidente en el micrófono y (c) la energía
que llega al micrófono en t = 25.0 min.

Solución
(a) La intensidad una distancia d de una fuente con potencia Pf es

Pf 31.6 W
I= 2
= 2
= 6.68 × 10−5 2
4πd 4π(194) m

(b) La potencia que recibe el micrófono es por una intensidad I es

Pm = IA = (6.68 × 10−5 )(7.52 × 10−5 ) = 5.02 × 10−9 W

(c) La energía que recibe se obtiene el micrófono es

Em = P t = (5.02 × 10−9 )(25)(60) = 7.53 × 10−6 J = 7.53 µJ

[Sears] Problema 16-59


Una soprano y un tenor están cantando a dueto. Mientras la soprano canta un la sostenido a fS 932 Hz,
el tenor canta un la sostenido pero tres octavas más abajo. En esta sala de conciertos, la densidad del
aire es de ρ 1.20 kg/m3 y su módulo volumétrico es de B = 1.42 × 105 Pa. Para que sus notas tengan el
mismo nivel de instendad de sonido, cual debe ser (a) la razón entre las amplitudes de presión del tenor y
de la soprano y (b) la razón entre las amplitudes de desplazamiento del tenor y de la soprano? (c) Cual
amplitud de desplazamiento (en m y mm) produce la soprano para cantar su la sostenido a 72.0 dB?
Dato adicional: 3 octavas más abajo es fT = 932/23 = 116.5 Hz

Solución
(a) La intensidad se relaciona con el ∆Pm de la forma I = (∆P )2 / (2ρv). No hay una dependencia de la
frecuencia, cuando se iguales los términos para la soprano y el tenor se obtiene

(∆Pm,S )2 (∆Pm,T )2
I= = ⇒ ∆Pm,S = ∆Pm,T
2ρv 2ρv

43
CAPÍTULO 4. ONDAS SONORAS

Los ∆Pm son iguales para los 2.


(b) Usando la relación ∆Pm = Bsm k y usando el resultado de la parte a)

Pm,S = ∆Pm,T ⇒ Bsm,S kS = Bsm,T kT

Con v = ω/k ⇒ k = ω/v = 2πf /v

2πfS 2πfT sm,T fS 932


⇒ sm,S = sm,T ⇒ = = =8
v v sm,S fT 116.5

La razón es sm,T /sm,S = 8.


(c) Si la soprano canta a β = 72.0 dB
 
I β 72
β = 10 log ⇒ I = I0 10 10 = (10−12 )10 10 = 1.585 × 10−5 W/m2
I0
s
1 2 1p 2 1 2I
I = ρv (ωS sm,S ) = ρB (2πfS sm,S ) ⇒ sm,S = √
2 2 2πfS ρB
s
1 2(1.585 × 10−5 )
sm,S = p = 4.73 × 10−8 m = 47.3 nm
2π(932) 5
(1.20)(1.42 × 10 )

[Sears] Problema 16-60


El sonido de una trompeta radia uniformemente en todas direcciones en aire a 20◦ C. A una distancia de
d = 5.00 m de la trompeta, el nivel de intensidad de sonido es de βd 52.0 dB. La frecuencia es de f = 587
Hz. (a) Determine la amplitud de presión a esta distancia. (b) Calcule la amplitud de desplazamiento.
(c) A qué distancia el nivel de intensidad del sonido es de 30.0 dB?

Solución
(a) Obtenemos la intensidad Id primero

Id β 52
βd = 10 log( ) ⇒ I = I0 10 10 = (10−12 )10 10 = 1.58 × 10−7 W/m2
I0

(∆Pm )2 p p
I= ⇒ ∆Pm = 2ρvI = 2(1.20)(344)(1.585 × 10−7 ) = 0.0114 Pa
2ρv

(b) Sabiendo ∆Pm , obtenemos sm con

ω 2πf v∆Pm (344)(0.0114)


∆Pm = Bsm k = Bsm = Bsm ⇒ sm = = = 7.49 × 10−9 m
v v B2πf (1.42 × 105 )2π(587)

(c) Distancia r para tener βr = 30 dB.


     
Ir Id Ir
βr − βd = 10 log − 10 log = 10 log
Io Io Id

44
CAPÍTULO 4. ONDAS SONORAS

Ya que viene de la misma fuente, tenemos P = 4πr2 I, por lo tanto, cumple


Ir d2
4πd2 Id = 4πr2 Ir ⇒ = 2
Id r
d2
     
Ir d d βr −βd βd −βr 52−30
βr −βd = 10 log = 10 log = 20 log ⇒ = 10 20 ⇒ r = d10 20 = (5.00)10 20 = 62.9m
Id r2 r r
Tiene que estar a una distancia de r = 62.9 m

I Parcial 2013 II Ciclo Problema 4


Un equipo de una fábrica produce β1 = 130 dB a una distancia de r1 = 5 m. Para disminuir el ruido se
utilizan láminas de espuma acústica, cada lámina disminuye 10 veces la intensidad del sonido. Determi-
ne: (a) la cantidad de láminas que se necesitan para disminuir la intensidad del sonido a 40 dB, (b) la
distancia a la que debe alejarse de la fuente (sin espumas), para medir la mitad de la intensidad sonora
(en decibles) que emite el equipo.

Solución
(a) Por cada espuma acústica se disminuye 10 veces. Si hacemos I0 pase por la espuma, del  otro lado sale
Ii /10 = Ii ∗ 10−1 . Por la segunda espuma, entra una intensidad Ii ∗ 10−1 y sale Ii ∗ 10−1 /10 = Ii ∗ 10−2 .
Si se hace n veces, la intensidad se reduce Ii ∗ 10−n .
Para los decibeles βi llega con una intensidad de Ii . Si filtramos con n espumas, vamos a tener βn con
In = I0 ∗ 10−n .
Usamos la relación entre decibeles e intensidad de sonido β = 10 log(I/I0 ) y hacemos la resta
       
I1 In I1 I1
β1 − βn = 10 log − 10 log = 10 log = 10 log = 10 log (10n ) = 10n
I0 I0 In I1 ∗ 10−n
β1 − βn 130 − 40
=⇒n= =9
10 10
Se ocupan n = 9 espumas para reducir la intensidad del sonido a 40 dB.
(b) Ahora queremos reducir que los decibeles que se miden sean β2 = 130/2 = 65 dB. Hacemos de nuevo
la resta y obtenemos igual que la parte pasada
 
I2
β1 − β2 = 10 log
I1

Ya que se está midiendo la misma fuente de sonido (la fábrica), tenemos que P = 4πr2 I, donde P es la
potencia de la fuente de sonido.
Relacionamos las posiciones iniciales y finales con
I2 r2
4πr12 I1 = 4πr22 I2 ⇒ = 12
I1 r2
Y substituimos
r12
     
I2 r1
β2 − β1 = 10 log = 10 log = 20 log
I1 r22 r2
   
β2 −β1 β1 −β2
r1 130−65
⇒ = 10 20
⇒ r2 = r1 10 20
= (5)10( 20 ) = 8891.4 m
r2

45
CAPÍTULO 4. ONDAS SONORAS

Se ocupa estar a una distancia de r2 = 8891.4 m para medir la mitad de la intensidad sonora.

Reposición I Parcial 2013 I Ciclo Problema 5


En un aula ruidosa hay NT = 52 estudiantes hablando simultáneamente y generan un nivel sonoro de
βT = 76 dB. Determine: (a) la intensidad que produce un estudiante hablando (suponiendo que todos
hablan a la misma intensidad), (b) el nivel sonoro que se produce cuando dejan de hablar el 75 % de los
estudiantes, (c) la máxima cantidad de estudiantes que pueden hablar simultáneamente si quiere un tener
una disminución de un 15 % del nivel sonoro.

Solución
(a) Sea NT = 52 estudiantes y βT = 76 dB. Y sea IE la intensidad de un solo estudiante. La intensidad
de todos los estudiantes es la suma de las intensidades ⇒ IT = IE NT . Plantenado con la relación de
decibeles e intensidad
   
IT NT IE NT IE βT I0 βT
βT = 10 log = 10 log ⇒ = 10 10 ⇒ IE = 10 10
I0 I0 I0 NT

1 × 10−12 76 W
⇒ IE = ∗ 10 10 = 7.655 × 10−6 2
52 m
Cada estudiante habla a una intensidad de IE = 7.655 × 10−7 W/m2 .
(b) Si dejan de hablar el 75 % de los estudiantes, queda un 25 % hablando. La intensidad total de esos
estudiantes es 0.25 (NT IE )

(0.25)(52)(7.655 × 10−7 )
   
0.25 ∗ 52 ∗ IE
β0.25 = 10 log = 10 log = 69.97 dB
I0 10−12

(c) Ahora queremos que el nivel sonoro sea 0.85βT , que corresponde a una intensidad de n ∗ IE , donde n
es el número de estudiantes hablando
10−12
   
nIE I0 0.85βT 0.85(76)
⇒ 0.85βT = 10 log ⇒n= 10 10 = 10 10 = 3.76
I0 IE 7.655 × 10−7

Ya que los estudiantes vienen en números enteros. Si queremos estar debajo del 85 %, deberían hablar 3
estudiantes. Si queremos estar un poquito más arriba del 85 %, deberían hablar 4 estudiantes.

46
5 | Superposición de Ondas

I Parcial 2013 II Ciclo Problema 3


Dos ondas se propagan en dirección opuesta en una cuerda tensa, estas interfieren y crean una onda
estacionaria de la siguiente manera:

y(x, t) = 0.02 sin(30x) cos(300t) m

Donde x está en metros. La cuerda que es de nylon tiene un área transversal de 10−5 m2 y una densidad
de 1150 kg/m3 . Una masa colgada mediante una polea en un extremo de la cuerda es la que produce la
tensión en la cuerda. Considerando que la cuerda vibra en su tercer armónico calcule: (a) La longitud de
la cuerda, (b) la velocidad y frecuencia de las ondas, (c) El valor de la masa colgante.

Solución
Identifiquemos términos1
y(x, t) = 0.02 sin(30x) cos(300t) m
y(x, t) = A sin(kx) cos(ωt) m
Vemos que A = 0.02 m, k = 30 m−1 y ω = 300 s−1 .
(a) En lugar de usar fórmulas de golpe, relacionemos la longitud de la cuerda L y la longitud de onda λ
con un dibujo. Está en el tercer armónico. Se ve así De aquí vemos que L = 3(λ/2). Ahora cambiamos λ

Figura 5.1: I Parcial 2013 II Ciclo Problema 3

con k
3 3 2π 3 2π π
L= λ= = = m = 0.314 m
2 2 k 2 30 10
1
Esta A puede variar en la definición dependiendo de lo que se pregunte. La A de ahorita es la amplitud de la onda de
superposición, solo que se podría colocar como 2A, usando la A como la amplitud de las ondas que causan la nueva onda.

47
CAPÍTULO 5. SUPERPOSICIÓN DE ONDAS

(b) La velocidad de la onda es v = ω/k = 300/30 = 10 m/s.


La frecuencia es f = ω/(2π) = (300)/(2π) = 47.75 Hz.
(c) Vease que nos están dando la densidad volumétrica ρ. Ocupamos la densidad lineal µ para relacionarla
con la velocidad. En el problema ya resuelto antes, el [Serway] Problema 16-31, habíamos obtenido el
resultado µ = ρA. Aprovechando que ya lo obtuvimos, ya tenemos los datos para encontrar la masa
colgante. La masa colgante hace una tensión T = mg
s r
T mg
⇒v= =
µ ρA

102 (1150)(1 × 10−5 )



v 2 ρA
⇒m= = = 0.117 kg
g 9.8

[Sears] Problema 15-74


Una cuerda de guitarra vibra en su modo fundamental, con nodos en sus extremos. La longitud del seg-
mento de cuerda que vibra libremente es de 0.386 m. La aceleración transversal máxima de un punto en
el punto medio del segmento es de L = 8.40 × 103 m/s2 y la velocidad transversal máxima es de 3.80 m/s.
(a) Calcule la amplitud de esta onda estacionaria. (b) Qué rapidez tienen las ondas viajeras transversales
en esta cuerda?

Solución

Figura 5.2: Cuerda de guitarra.


I Parcial 2013 II Ciclo Problema 3

Viendo por dibujo, la longitud de la cuerda L es L = λ/2 ⇒ λ = 2L. Ocupamos escribir la ecuación de la
onda, que tiene la forma 2
y(x, t) = A sin(kx) cos(ωt)
Empezamos a sustituir el k en términos de L, que es el dato conocido.
 
2π π 
y(x, t) = A sin(kx) cos(ωt) = A sin x cos(ωt) = A sin x cos(ωt)
λ L
2
La función de la onda también puede ser escrita de la forma y(x, t) = A sin(kx) sin(ωt). Lo que cambia con el sin(ωt) es
que la cuerda está toda en el punto de equilibrio (completamente horizontal), con el cos(ωt) está en la forma más “curveada".

48
CAPÍTULO 5. SUPERPOSICIÓN DE ONDAS

Nos están dando la vy,max y ay,max en el punto medio3 , que es x = L/2


   
L πL
y x = , t = A sin cos(ωt) = A cos(ωt)
2 L2
Ahora derivamos para tener la velocidad y aceleración en el punto x = L/2
∂y
= vy = −ωA sin(ωt) ⇒ vy,max = ωA (5.1)
∂t
∂vy
= ay = −ω 2 A sin(ωt) ⇒ ay,max = ω 2 A (5.2)
∂t
Para obtener ω, dividimos 5.2 entre 5.1
ay,max ω2A ay,max 8.43
= =ω⇒ω= = = 2.21 × 103 s−1
vy,max ωA vy,max 83.4
Conociendo ω, podemos usar las Ec. 5.1 o 5.2 para obtener A
vy,max 3.80
vy,max = ωA → A = = = 1.72 × 10−3 m
ω 2.21 × 103
(b) La velocidad la obtenemos con v = λf y cambiando estos términos por datos conocidos
ω Lω (0.386)(2.21 × 103 )
v = λf = 2L = = = 271.5 m/s
2π π π

[Sears] Problema 15-18


I Parcial 2017 I Ciclo Pregunta 2
Una cuerda de L = 1.50 m que pesa Wc = 0.0125 N está atada al techo por su extremo superior, mientras
que el extremo inferior sostiene un peso W . Desprecie la pequeña variación de tensión a lo largo de la
cuerda producida por el peso de la misma. Cuando usted da un leve pulso a la cuerda, las ondas que
viajan hacia arriba de esta obedecen la ecuación

y(x, t) = (8.50 mm) cos(172m−1 x − 4830s−1 t)

Suponga que la tensión de la cuerda es constante e igual a W . (a) Cuánto tiempo tarda un pulso en
recorrer toda la cuerda? (b) Cuál es el peso W ? (c) Cuántas longitudes de onda hay en la cuerda en
cualquier instante? (d) Cuál es la ecuación para las ondas que viajan hacia abajo de la cuerda?

Solución
Identifiquemos términos. k = 172 m−1 y ω = 4830 s−1

(a) La velocidad de la onda es v = ω/k. Ya que se mueve con velocidad constante aplica vt = L
L Lk (1.5)(172)
⇒t= = = = 0.0534 s = 53.4 ms
v ω (4830)
3
Aquí estamos en un punto de amplitud máxima, entonces la parte espacial se hace sin → 1, caso contrario, tendríamos
que mantener el término sin(πx0 /L).

49
CAPÍTULO 5. SUPERPOSICIÓN DE ONDAS

(b) Aquí la tensión es causada por el peso, T = W . Además, ya que nos dan el peso de la cuerda, podemos
saber la densidad lineal
mc Wc
µ= =
L gL
Entonces
ω2 (0.0125)(4830)2
 
T W W W
2
v = =  ⇒ W = c v2 = c =
µ W c gL gL k2 (9.8)(1.5)(172)2
gL
⇒ W = 0.6705 N
(c) La cantidad de longitudes de ondas en toda la cuerda es N = L/λ y λ lo podemos saber que ya
sabemos k
L L Lk (1.5)(172)
N= = = = = 41.06
λ 2π 2π 2π
k
(d) Está es un poco extraña dependiendo de como se interprete. A como quería el libro y el examen, no
son las ondas que rebotan en la pared y viajan hacia abajo, si no que se tiene la fuente generadora de
ondas en algún punto arbitrario de la cuerda como se muestra en la Fig 5.3

Figura 5.3: Ondas hacia arriba y abajo.


[Sears] Problema 15-18

La ecuación y(x, t) = A cos(kx − ωt) son lad ondas que viajan hacia arriba. Para hacerlas que viajen
hacia abajo, le cambiamos la dirección haciendo kx − ωt → kx + ωt y mantenemos el mismo signo de
la amplitud, no le ponemos un menos como si estuviera rebotando en la pared, la amplitudes generadas
hacia arriba y abajo están “del mismo lado", entonces la amplitud tiene el mismo signo. La ecuación de
las ondas hacia abajo es
y(x, t) = 8.50 cos(172m−1 x + 4830s−1 t) mm

[Serway] Problema 18-27


Reposición I Parcial 2017 I Ciclo Pregunta 3

50
CAPÍTULO 5. SUPERPOSICIÓN DE ONDAS

Un objeto se cuelga de una cuerda (con densidad lineal µ = 0.00200 kg/m) que pasa sobre una polea
ligera. La cuerda se conecta a un vibrador de frecuencia constante f y la longitud de la cuerta entre el
puntp P y la polea es L = 2.00. Cuando la masa m del objeto es 16.0 kg o 25.0 kg, se observan ondas
estacionarias, sin embargo, no se observan ondas estacionarias con alguna masa entre estos valores. (a)
Cuál es la frecuencia del vibrador? Nota: Mientras mayor es la tensión de la cuerda, menor es el número
de nodos en la onda estacionaria. (b) Cuál es la masa más grande del objeto con el cual se podría observar
ondas estacionarias?

Figura 5.4: [Serway] Problema 18-27

Solución
Veamos por que la nota dice eso, a mayor tensión, menor número de nodos. La frecuencia de las ondas
generadas en las cuerdas van a ser f , lo importante es que es constante, la misma en todas. Lo que va a
cambiar el la velocidad y la longitud de onda λ. La velocidad puede cambiar dependiendo de la masa que
se coloque. La tensión en T = mg, entonces la velocidad depende de la masa por
s r
T mg
v= =
µ T

La velocidad v de las ondas aumenta cuando la masa m aumenta. Igual v disminuye cuando m disminuye.
Ahora despejando λ r
1 mg
v = λf → λ =
f µ
La longitud de onda λ crece cuando la masa m aumenta. PERO λ tiene un límite de que tan grande se
puede hacer. El máximo que puede ser λ es λ = 2L. Hay una masa que hace que llegue a este máximo de
λ. Y no tenemos una restricción de que tan pequeño puede hacerse λ, entonces sí podemos ir haciendo la
masa más pequeña y asegurarnos que vamos a ver ondas estacionarias.

Para relacionarlo la masa m con el n−ésimo modo de vibración, sabemos que λ ∝ m, entonces
λ 2L
n =L⇒n=
2 λ
1 1
⇒n∝ ⇒n∝ √
λ m

51
CAPÍTULO 5. SUPERPOSICIÓN DE ONDAS

De aquí vemos que mientras más pequeña sea la masa m, vamos a estar en un modo de oscilación n
mayor. O mientras mayor sea la tensión (la masa), vamos a tener un menor modo de oscilación n, que
nos daría menos número de nodos. Esta es la nota que nos dice el libro.

Figura 5.5: Relación entre n, m y λ.


[Serway] Problema 18-27

El enunciado nos dice que para las masas 16 kg y 25 kg se observan ondas estacionarias y no con masas
entre estos valores. De esto podemos concluir que son modos de vibración vecinos n y n + 1. Solo que
tenemos que ver cual modo le pertecene a que masa. Con la discusión anterior, sabemos que mientras más
pequeña sea la masa, mayor va a ser el n. No sabemos en que modo está vibrando para cada masa, así que
asignemole el n modo a la masa de 25 kg ( mn = 25 kg ) y al n + 1 modo a mn+1 = 16 kg. Expresamos
la relación de λ, n y L y relacionamos con las otras variables del problema
  s
n n v n1 T
L= λ= =
2 2 f 2f µ

Para los modos n y n + 1, con T = mg para cada masa y reacomodando


r
n 1 mn g
f= (5.3)
2L µ
r
n + 1 1 mn+1 g
f= (5.4)
2 L µ
Dividiendo 5.4 entre 5.3 y evaluando para las masas
r r  
n + 1 mn+1 n + 1 16 n+1 4
=1= = =1
n mn n 25 n 5
n+1 5
⇒ =
n 4
De esta última ecuación, podemos ver que n = 4. Cuando se coloca una masa de 25 kg, se está en el n = 4
modo de oscilación. Para la masa de 16 kg está en el n = 5 modo. Usamos cualquiera de las dos Ec. 5.3
o 5.4 para encontrar el valor de f
r s
n 1 mn g 4 1 (25)(9.8)
f= = = 350 Hz
2L µ 2 (2.00) (2 × 10−3 )

52
CAPÍTULO 5. SUPERPOSICIÓN DE ONDAS

(b) La masa más grande va a estar en el n−ésimo modo de vibración, que va a ser n = 1. Usemos la
ecuación 5.3 y despejemos m1

4L2 f 2 µ 4(2.0)2 (350)2 (2.0 × 10−3 )


r
1 1 m1 g
f= ⇒ m1 = = = 400 kg
2L µ g (9.8)

La masa más grande que se puede colocar para ver ondas estacionarias es de 400 kg.

[Sears] Problema 16-32


Parecido al I Parcial 2017 I Ciclo Problema 5
Un tubo abierto-cerrado de longitud ajustable se encuentra cerca de un alambre de Lc = 85.0 cm y de
mc = 7.25 g que está siendo sometido a una tensión de T = 4110 N. Se desea ajustar la longitud del
tubo de manera que, cuando se produzca un sonido a su frecuencia fundamental, este sonido haga que
el alambre vibre en su segundo sobretono con una amplitud muy grande. De qué longitud debe ser el tubo?

Solución
La cuerda está en el segundo sobretono, es una manera de decir que está en el tercer modo de oscilación,
nc = 3 y el tubo abierto-cerrado está en el fundamental, la longitud de onda más larga permitida con esas
condiciones de frontera es la que se muestra4 en la Fig. 5.6

Figura 5.6: [Sears] Problema 16-32

Usemos el índice p para el tubo.5 . El sonido del tubo ajustable está haciendo que el alambre vibre con
una amplitud muy grande, significa que tiene la misma frecuencia fc = fp . Encontremos la frecuencia de
cada sistema por aparte y las igualamos para encontrar la longitud del tubo.
Primero la frecuencia de la cuerda. De la Fig. 5.6, vemos que (3/2)λc = Lc , entonces
s r
vc 3 1 T 3 1 T Lc
λc fc = vc ⇒ fc = = =
λc 2 Lc µ 2 Lc mc
4
Aquí lo dibuje para el desplazamiento s(x, t), pero haciendo lo mismo con la presión ∆p(x, t) se puede concluir lo mismo.
5
Pipe

53
CAPÍTULO 5. SUPERPOSICIÓN DE ONDAS

r
3 T
⇒ fc = (5.5)
2 mc Lc
Ya tenemos la frecuencia de la cuerda, ahora sigue la del tubo.
La velocidad de las ondas en el tubo, ya que son ondas sonoras, la velocidad del sonido vs = 344 m/s.
Viendo en la Fig. 5.6, la relación para Lp = (1/4)λp , entonces
vs
λp fp = vs ⇒ fp =
λp
vs
⇒ fp = (5.6)
4Lp
Igualando las Ec. 5.5 y 5.6 r
3 T vs
fc = fp ⇒ =
2 m c Lc 4Lp
r s
1 mc Lc 1 (7.25 × 10−3 ) (0.85)
⇒ Lp = vs = (344) = 0.0702 m = 7.02 cm
6 T 6 (4110)

[Serway] Problema 18-52


Ampliación 2015 II Ciclo Problema 1
Un estudiante usa un oscilador de audio de frecuencia ajustable para medir la profundidad de un pozo
de agua. El estudiante reporta que escucha dos resonancias sucesivas a 51.87 Hz y 59.85 Hz. (a) Qué tan
profundo es el pozo? (b) Cuántos antinodos hay en la onda estacionaria a 51.87 Hz?
Solución
Saquemos la secuencia de los tubos abiertos-cerrado. Voy a sacarlo graficando el s(x, t), pero se puede
obtener el mismo resultado graficando con ∆P (x, t)

Figura 5.7: Secuencia de λ permitido


[Serway] Problema 18-52

(a) Viendo la Fig. 5.7, vemos que la secuencia de las λn permitidas es son los impares ×λ/4, escrito más
formal
(2n − 1)
L= λn (5.7)
4

54
CAPÍTULO 5. SUPERPOSICIÓN DE ONDAS

Donde n reccore de n = 1, 2, 3, ..., mejor acomodarlo así como hacen algunos autores que ponen la secuencia
recorriendo los impares6 Podemos obtener la secuencia de frecuencias permitidas despejando de
1 (2n − 1)
λn fn = vs ⇒ fn = vs = vs
λn 4L
(2n − 1)
fn = vs (5.8)
4L
Donde vs es la velocidad del sonido.
El problema nos está diciendo que solo se escuchan resonancias sucesivas entre 51.87 y 59.85 Hz, esto nos
dicen que son modos vecinos. De la Ec. 5.8, vemos que fn aumenta conforme aumenta n, y el problema
no nos dice en que modo de oscilación estamos, solo que son modos vecinos. Escojamos entonces que
fn = 51.87 Hz y fn+1 = 59.85 Hz que es el siguiente. Podemos independizarnos del n que es desconocido
aprovechando que la Ec. 5.8 es lineal en n, entonces si saco diferencia entre fn+1 y fn elimino a n
(2(n + 1) − 1) (2n − 1) 1 vs
fn+1 − fn = vs − vs =
4L 4L 2L
1 vs
⇒ fn+1 − fn =
2L
Despejamos L
1 vs 1 (343)
L= = = 21.5 m
2 fn+1 − fn 2 (59.85 − 51.87)
(b) Sabiendo cuanto es L, usamos la Ec. 5.8 y despejamos n para saber en que modo está
(2n − 1) 2Lfn 1
fn = vs ⇒ n = +
4L vs 2
Para la frecuencia de 51.87 Hz
2(21.5)(51.87) 1
n= + = 7.002 = 7
343 2
Para corroborar que nos dio bien, hagamolo para la frecuencia de 59.85 Hz, nos debería dar n = 8
2(21.5)(59.85) 1
n= + = 8.003 = 3
343 2
Lo cual nos da bien. Es consistente el resultado. Ahora grafiquemos el s(x, t) o ∆P (x, t). Ya que n = 7,
usamos la Ec 5.7 para ver cuantos segmentos de λ/4 caben dentro del tubo.
2(7) − 1 13
L= λ= λ
4 4

Graficando tanto el desplazamiento s(x, t) como el ∆P (x, t) se cuentan 7 antinodos.


Enlaces de interés:
Tubo de Ruben: Se puede visualizar los antinodos de presión con fuego
Steve Mould: A better description of resonance
https://www.youtube.com/watch?v=dihQuwrf9yQ
6
Colocandolo de este modo, cuando saque diferencias entre modos, sé que el siguiente modo esta a n + 1, no en n + 2,
para hacer restas o sumas de ese estilo, mejor lo acomodo que recorra todos los naturales y no tengo que acordarme que el
siguiente está a n + 2, más facil.

55
CAPÍTULO 5. SUPERPOSICIÓN DE ONDAS

Figura 5.8: Antinodos es f = 51.87 Hz.


[Serway] Problema 18-52

56
6 | Temperatura

[Serway] Problema 19-54


Dos barras metálicas se fabrican de Invar y una tercera barra se elabora de aluminio. A 0◦ C cada una
de las tres barras se taladra con dos orificios separados a 40.0 cm. (a) Encuentre una expresión para el
ángulo entre las barras de Invar en función de la temperatura. Analice cualitativamente si tiene sentido.
(b) Cual es el mayor y menor ángulo que se pueden obtener.
El punto de fusión del Aluminio es 660 ◦ C y el del Invar es 1472 ◦ C. Datos adicionales: Invar: αIn =
0.9 × 10−6 (C◦ )−1 y Aluminio: αAl = 24 × 10−6 (C◦ )−1

Figura 6.1: [Serway] Problema 19-54

Solución
(a) El triangulo cuando cambie la temperatura va a ser un isoceles porque 2 de los lados están hechos
del mismo material y cambian su longitud de igual manera. El centro siempre va a caer perpendicular a
la barra de Aluminio.
Entonces podemos usar la relación para cualquier temperatura
1
 
θ LAl
sin = 2
2 LIn
Ahora cambiamos el LAl y LIn en función de ∆T . Sea L0 = 40 cm.
 
θ 1 L0 (1 + αAl ∆T )
sin = (6.1)
2 2 L0 (1 + αIn ∆T )
 
1 (1 + αAl ∆T )
⇒ θ = 2 arcsin (6.2)
2 (1 + αIn ∆T )

57
CAPÍTULO 6. TEMPERATURA

Figura 6.2: [Serway] Problema 19-54

Veamos a ver si tiene sentido. Imaginemos casos límites. Usemos la Ec 6.1 que es más facil ver de ese lado.
El sin(30◦ ) es 1/2. Si tenemos un poquito más de 1/2, el ángulo va a crecer un poquito (Poquito más de
30). Si tenemos un poquito menos de 1/2, el ángulo va a hacerse un poquito más pequeño (Poquito menor
de 30). Para números positivos, sea un número a, si hago a ∗ x donde x > 1, el resultado es un número
mayor a a. Si hago a ∗ x con 0 < x < 1, el resultado es un número menor a a. Usemos esto.
Primer caso límite: Que solo el Aluminio se expanda (αIn = 0). Sin ver la ecuación, si solamente el
Aluminio se expande y las barras de Invar no, el angulo θ debería crecer La Ec. 6.1 se reduce a
 
θ 1
sin = (1 + αAl ∆T )
2 2

Que es una función que crece conforme ∆T . Entonces (1 + αAl ∆T ) es un número mayor a 1, ⇒ 1/2 ∗
(1 + αAl ∆T ) > 1/2, entonces el ángulo va a crecer y ser mayor a 30◦30 . Es consistente con la parte
cualitativa, que debería crecer.
Otro caso límite las barras de Invar se expandar y la de Aluminio no (αAl = 0). La parte de abajo del
triangulo se queda constante mientras los otros lados crecen. El ángulo debería hacerse más chiquito.
Usando el caso límite en la Ec. 6.1 se reduce a
 
θ 1 1
sin =
2 2 (1 + αIn ∆T )

El 1/2 está siendo multiplicado por un número menor a uno, entonces al sacar el arcsin se obtiene un
número menor a 30.
El último caso límite es cuando las expansiones de todas las barras son las mismas. Si todos los lados se
expanden iguales, los ángulos no cambian. Usando la Ec. 6.1 en el límite αIn ⇒ αAl pasa a ser
 
θ 1 π
sin = ⇒ θ = = 60◦
2 2 3

Pasa a ser una constante el ángulo.


Veamos el caso en que ambas barras se expandan. Comparando los coeficientes αAl > αIn ⇒ Esto implica
que el Aluminio cambia más su longitud para el mismo ∆T . Usando los casos límites donde las barras se
expanden igual y el aluminio es la única que se expande, este caso está en el intermedio. El ángulo va a

58
CAPÍTULO 6. TEMPERATURA

crecer para un ∆T > 0. Comprobemos con la mátematica este resultado. Partiendo de αIn < αAl
1 + αAl ∆T
αIn < αAl ⇒ 1 + αIn ∆T < 1 + αAl ∆T ⇒ 1 <
1 + αIn ∆T
1 1 1 + αAl ∆T

<
2 2 1 + αIn ∆T
Lo que está del R.H.S de la desigualdad es el R.H.S de la Ec 6.1, que es un mayor a un medio. Significa
que el ángulo crece cuando aumenta la temperatura.
Sí tiene sentido este resultado de la Ec 6.1
(b) El mayor ángulo se alcanza hasta donde el triangulo se logra expandir manteniendose como triangulo.
Deja de ser triangulo cuando alguno de los metales se derrita. El que tiene el menor punto de fusión es el
Aluminio con 660 ◦ C. Tomando el Ti = 0 ◦ C el mayor ∆T es ∆T = 660 ◦ C. Evaluando en la Ec 6.2
!
1 1 + (24 × 10−6 )(660)
θmax = 2 arcsin = 1.06484 rad = 61.01◦
2 (1 + (0.9 × 10−6 )(660))

El ángulo aumenta cuando se calienta.


La menor temperatura sería bajar hasta la temperatura más baja posible, el 0K = −273, los 2 metales
siguen siendo sólidos, entonces “sí” pueden bajar hasta esa temperatura 1 . El ∆T es ∆T = −273
!
1 1 + (24 × 10−6 )(−273)
θmin = 2 arcsin = 1.03992 rad = 59.58◦
2 (1 + (0.9 × 10−6 )(−273))

I Parcial 2017 I Ciclo Problema 6


−6 (C◦ )−1 , con
 
En una instalación industrial, se llena hasta el tope un contenedor de acero α = 12 × 10
650 kg glicerina β = 5 × 10−4 (C◦ )−1 , cuando la temperatura está a 30◦ C. La densidad de la glicerina
 

a dicha temperatura es de 1.261 g/cm3 . En la madrugada la temperatura desciende hasta los 19◦ C .
Calcule para la glicerina: (a) la densidad a 19◦ C, (b) el volumen a 19◦ C, (c) la cantidad de litros de
glicerina que se debería agregar para llenar nuevamente el contenedor a 19◦ C.

Solución
Usemos el subindice 0 para T = 30 ◦ C y f para T = 19 ◦ C. (a) El volumen incial es V0 = m/ρ0 . El
volumen final es
m
Vf = V0 [1 + β∆T ] = [1 + β∆T ] (6.3)
ρ0
650 
1 + (5 × 10−4 )(19 − 30) = 0.512 m3

=
1261
Consejo: Por favor, aunque sepan que se disminuye la temperatura, ponga el [1 + β∆T ] y no [1 − β∆T ].
Dejen que el ∆T = Tf − Ti les de el signo.
(b) Sabiendo el volumen de la Ec 6.3
m m ρ0
ρf = = =
Vf m [1 + β∆T ]
[1 + β∆T ]
ρ0
1
Bueno, llegando a casi 0 y suponiendo que el α no cambia con la temperatura. Sospecharía que ya se sale demasiado de
la aproximación de L = L0 (1 + α∆T ), pero seamos felices.

59
CAPÍTULO 6. TEMPERATURA

Evaluando
1261 m
ρf = −4
= 1267 3
[1 + (5 × 10 )(19 − 30)] kg
(c) La cantidad de glicerina que hay que agregar es la diferencia de volumenes, ocupamos el volumen final
del contenedor, que lo podemos averiguamos con

Vf,c = V0 [1 + 3α∆T ]

Donde el 3α es por ser volumen. Le restamos esto a la Ec. 6.3 y obtenemos

∆V = Vf,c − Vf = V0 [1 + 3α∆T ] − V0 [1 + β∆T ]


m
∆V = V0 (3α − β) ∆T = (3α − β) ∆T (6.4)
ρ0
Evaluando
650 
3(12 × 10−6 − (5 × 10−4 )) (19 − 30) = 2.63 × 10−3 m3 = 2.63 L

1261
Veamos un caso límite, que pasaría si el contenedor se expande igual que la glicerina? El ∆V al final
debería ser 0. Para corroborar esto, usemos la Ec. 6.3 con 3α = β ⇒ 3α − β = 0

∆V = V0 (3α − β) ∆T = 0

Se obtiene el límite esperado.


[Sears] Problema 17-79
Parecido al II Parcial 2012 I Ciclo Pregunta 3, Reposición I Parcial I Ciclo 2017 Pregunta 6
Usted está preparando
 pesto para pasta y una taza medidora cilíndrica de h = 10.0 cm de altura hecha
de vidrio ordinario βg = 2.7 × 10−5 (C◦ )−1 se llena con aceita de oliva βo = 6.8 × 10−4 (C◦ )−1 hasta
una altura de ` = 2.00 mm por debajo del borde de la taza. En un principio, la taza y el aceite están a
temperatura ambiente (20◦ C). El teléfono suena y usted se olvida del aceite de oliva, que por descuido,
dejó calentando sobre la estufa encendida. La taza y el aceite se calientan lentamente y tienen la misma
temperatura. A qué temperatura comenzará a derramarse el aceite?

Solución

Figura 6.3: [Sears] Problema 17-79

Nota: El β ya representa el cambio volumétrico, no hay que hacer β = 3α.


No nos están dando el área o el volumen inicial/final. Pero sabemos que para la taza y el aceite: Tienen
la misma área antes y después de calentarse y tiene el mismo volumen después de calentarse.r La altura
inicial del aceite la podemos saber ya que nos están diciendo que está a una altura ` debajo del borde.

60
CAPÍTULO 6. TEMPERATURA

La altura inicial es h − `. Sea el subindice g para el vidrio y el o para el aceite. Los volumenes son iguales
cuando se calientan
Vf,g = Vf,o
⇒ Vi,g (1 + βg ∆T ) = Vi,o (1 + βo ∆T )
Usamos el área A, que aunque no la sabemos, podemos cancelarla

⇒ Ah (1 + βg ∆T ) = A(h − `) (1 + βo ∆T )

⇒ h (1 + βg ∆T ) = (h − `) (1 + βo ∆T )

De esta expresión ya podemos despejar ∆T , que es


` 0.002
∆T = =
[(βo − βg ) h − βo `] [[(6.8 × 10 ) − (2.7 × 10 )] (0.1) − (6.8 × 10−4 )(0.002)]
−4 −5

⇒ ∆T = 31.3 ◦ C
Sabiendo cuando vale ∆T , averiguamos la temperatura final

∆T = Tf − Ti ⇒ Tf = Ti + ∆T = 22 + 31.3 = 53.3 ◦ C

[Resnick] Ejercicio 21-21


Con una regla de acero a la temperatura de 20◦ C se mide una varilla y se descubre que tiene una longitud
de L0 = 20.05 cm. Tanto la varilla como la regla se colocan en un horno a 270◦ C; la varilla mide ahora
L0 = 20.11 cm si usamos la misma regla. Calcule el coeficiente de la expasión térmica α del material de
que está hecha la varilla.
Dato adicional: Coeficiente del Hierro αF e = 11 × 10−6 (◦ C)−1
Solución
La longitud de la varilla inicialmente es L0 . La varilla al calentarse tiene una longitud final Lf que no es L0 ,
la regla está descalibrada porque la regla al calentarse cambio la longitud. Entonces L0 6= L0 [1 + α∆T ].

Figura 6.4: [Resnick] Ejercicio 21-21

El dato que nos están diciendo que mide L0 con la regla calentada, es que la varilla tiene una longitud
igual a la de una regla de acero de longitud inicial L0 que fue calentada con el mismo ∆T . Entonces la
longitud de la varilla Lf después de calentarse es

Lf = L0 [1 + αF e ∆T ]

61
CAPÍTULO 6. TEMPERATURA

De aquí podemos igualar con la longitud que se expandió la varilla.

Lf = L0 [1 + α∆T ] = L0 [1 + αF e ∆T ]

Y despejamos α
L0
 
1
α= (1 + αF e ∆T ) − 1 (6.5)
∆T L0
Evaluamos
 
1 (20.11) −6 1
1 + (11 × 10 )(270 − 20) − 1 = 2.30 × 105 ◦

α=
(270 − 20) (20.05) C
Veamos el caso límite, que pasaría si la regla después de calentar aún ve que la varilla “mide” L0 . Si la
medida no cambio, la varilla se tiene que expandir igual que el acero. Veamos con la Ec. 6.5 que concluimos
lo mismo. Para evaluar el límite, usamos L0 = L0 en la Ec. 6.5
 
1 L0 1
α= (1 + αF e ∆T ) − 1 = (αF e ∆T ) = αF e
∆T L0 ∆T
Se obtiene el resultado esperado.
[Resnick] Ejercicio 21-44
Un tubo abierto-cerrado con una longitud de L = 25.0 m contiene aire a presión atmosférica. Se lanza
verticalmente a un lago de agua fresca hasta que el agua alcanza la mitad del tubo. Qué profundidad
h tiene el borde inferior del tubo? Suponga que la temperatura es la misma en todas partes y que no
cambia.

Figura 6.5: [Resnick] Ejercicio 21-44

Solución
La temperatura y los moles en el gas cuando el tubo se hunde es constante. Planteando la ecuación de
estado del gas ideal, tenemos
Pi V i = Pf V f
EL tubo tiene un área A, que no nos la dan, pero la podemos cancelar. La columna de aire inicial mente
es L y al final es L/2. La presión inicial es Patm . Entonces
L
Patm AL = Pf A ⇒ Pf = 2Patm
2

62
CAPÍTULO 6. TEMPERATURA

La presión en el nivel de referencia la podemos obtener por 2 diferentes rutas. La primera es desde afuera
en la atmosfera hacia abajo, y la otra es desde dentro del tubo bajamos.
L L
Patm + ρgh = Pf + ρg = 2Patm + ρg
2 2
Patm L (1.013 × 105 ) 25
⇒h= + = + = 22.83 m
ρg 2 (1000)(9.8) 2

[Sears] Problema 18-57


Un cilindro de h = 1.00 m de altura con diámetro interior de d = 0.120 m contiene gas propano (masa
molar M = 44.1 g/mol) que se usará en una parrillada. Inicialmente, el tanque se llena hasta que la
presión manométrica es de ∆Pm,i = 1.30 × 106 Pa y la temperatura es de 22.0◦ C. La temperatura del gas
se mantiene constante mientras el tanque se vacía parcialmente hasta que la presión manométrica es de
∆Pm,f = 2.50 × 105 Pa. Calcule la masa de propano que se utilizó.

Solución
La masa usada en la parrillada es mi − mf . Sabemos la masa molar del gas, entonces lo relacionamos con
los moles con
mi − mf = (ni − nf )M (6.6)
Los moles ni y nf los podemos encontrar usando la ecuación de estado del gas ideal. La temperatura incial
y la final son las mismas ( Ti = Tf = T = 295 K ). Es un cilindro rigido, entonces los volumenes finales e
iniciales son iguales, Y sabemos cuando es el volumen en ambos casos Vi = Vf = V = hA = πhd2 /4. De
dato sabemos la presión manométrica, podemos calcular la presión absoluta con

Pi = ∆Pm,i + Patm = 13.0 × 105 + 1.013 × 105 = 14.0 × 105 Pa

Pf = ∆Pm,f + Patm = 2.50 × 105 + 1.013 × 105 = 3.51 × 105 Pa


Con el gas ideal, despejamos n que es n = P V /RT y sustituimos en 6.6
 
Pf V Pi V VM 1 M
mi − mf = − M = (Pi − Pf ) = (Pi − Pf ) πd2 h
RT RT RT 4 RT
1 (0.0441)
mi − mf = (14.013 − 3.51) × 105 π(0.12)2 (1.00)

= 0.214 kg
4 (8.314)(295)
La masa de propano que se utilizó fue de mi − mf = 0.214 kg.

[Serway] Problema 19-76


Un cilindro que tiene un radio de r = 40.0 cm y ha = 50.0 cm de profundidad se llena con aire a T = 20.0◦ C
y Pa = 1.00 atm. Ahora en el cilindro baja un pistón de mp 20.0 kg y comprime el aire atrapado en el
interior mientras llega a una altura de equilibrio hb . Finalmente, un perro de md = 25.0 kg de coloca sobre
el pistón y comprimera más el aire, que permanece a 20.0◦ C. (a) A que distancia hacia abajo ∆h = hb −hc
se mueve el pistón cuando el perro se para sobre él? (b) A qué temperatura debería ser calentado el gas
para elevar el pistón y el perro de regreso a hb ? Nota: La temperatura se mantiene constante hasta antes
de ser calentado para subir el pistón con el perro encima.
Solución

63
CAPÍTULO 6. TEMPERATURA

Figura 6.6: [Serway] Problema 19-76

(a) Desde que se pone el pistón encima, la cantidad de n del gas confinado permanece constante. Sabemos
el área del piston que es A = πr2 El volumen incial antes de que el peso del pistón haga que se vaya hacia
abajo es Va = ha A y la presión inicial es Pa . Planteamos la ecuación de estado del gas ideal

Pa Va = Pb Vb (6.7)

Pero no podemos resolverlo aún, tenemos 2 variables y una ecuación. Ocupamos otra. Hagamos el diagrama
de fuerzas sobre el pistón

Figura 6.7: Fuerzas sobre el pistón.


[Serway] Problema 19-76
X
Fi = Pb A − mp g − Patm A = 0
i
mp g
⇒ Pb = Patm + (6.8)
A
Sustituimos el Pb de la Ec. 6.8 en la Ec. 6.7

Pa Va = Pb Vb
 mp g 
⇒ Pa ha A = Patm + hb A
A
Despejando hb
ha ha
hb = mp g = mp g (6.9)
1+ 1+ 2
APatm πr Patm

64
CAPÍTULO 6. TEMPERATURA

Evaluando
0.50
hb = = 0.498083 m
(20)(9.8)
1+
π(0.4)2 (1.013 × 105 )

Para obtener hc , hacemos el mismo análisis. Solo que en lugar de tener la solo la masa del pistón mp ,
tenemos mp + md por la masa extra del perro. Hacemos en cambio mp → mp + md en la ecuación 6.8
para obtener Pc y se obtiene es
(mp + md ) g
Pc = Patm +
A
Igual, siguiendo los procedimientos para hb , reemplazamos mp → mp + md en la Ec. 6.9 y se obtiene
ha ha
hc = =
(mp + md )g (mp + md )g
1+ 1+
APatm πr2 Patm
0.50
hc = = 0.495707 m
(20 + 25)(9.8)
1+
π(0.4)2 (1.013 × 105 )
Y sacamos la diferencia

∆h = hb − hc = 0.498083 − 0.495707 = 0.00237 m = 2.37 mm

(b) Digamos que es el caso d. Ahora subimos el pistón y el perro hasta hd = hb calentando el gas. Si
hacemos el diagrama de fuerzas sobre el pistón, vemos que la presión sigue siendo la misma Pd = Pc , por
que la presión en este sistema está determinada por la presión atmosférica y el peso del pistón y el perro.
Ninguno de estos cambio con la temperatura. Lo que cambia es el volumen. Igualando con la ecuación de
estado del gas ideal
Vc Vd Vd hd A hb
= ⇒ Td = Tc = Tc = Tc
Tc Td Vc hc A hc
Evaluando. Recordando que en la temperatura en P V = nRT está en temperatura absoluta, Kelvin
0.498083
Td = (20 + 273) = 294.4 K
0.495707

Enlaces de interés:
La “sensación del calor” no es lo mismo que la temperatura.
Veritasium: Misconceptions About Temperature
https://www.youtube.com/watch?v=vqDbMEdLiCs
La historia de la escala Fahrenheit.
Veritasium: What the Fahrenheit?!
https://www.youtube.com/watch?v=LgrXd0NM2y8
La historia de la escala Celsius. La escala original estaba invertida. El 100 era para el hielo y 0 para vapor.
Veritasium: Celsius Didn’t Invent Celsius
https://www.youtube.com/watch?v=rjht4oAByCI
Definir que 100 ◦ C sea el punto de evaporación del agua no sería util ya que cambia el punto de ebullición
cambia con la presión. En este video se estaba midiendo el punto de ebullición mientras se escala el

65
CAPÍTULO 6. TEMPERATURA

Everest. La presión disminuye a medida que se aumenta la altura.


Periodic Videos Water Boiling at Everest - Periodic Table of Videos
https://www.youtube.com/watch?v=JTL4dj3Gx1o
La expansión térmica es usada para remover tatuajes.
Smarter Every Day: How Laser Tattoo Removal Works - Smarter Every Day 123
https://www.youtube.com/watch?v=D0B7F5UbTOQ

66
7 | La Primera Ley de la Termodinámica

P
Recomendación: En problemas de calorimetría, se sabe que Qi = 0, pero no se conoce de ante mano
la forma de la ecuación cuando pueden haber cambios de fase, por que pueden aparecer términos de
cambio de fase, y luego el nuevo calor especifico después de haber cambiado la fase.
Consejo personal, comparen calores si pueden cambiar completamente la fase de un componente para ir
estimando por donde va a terminar el sistema cuando llegue a equilibrio térmico.

[Resnick] Problema 23-7


Una persona prepara cierta cantidad de té helado mezclando m` = 520 g de té caliente (escencialmente
agua) con una masa mh igual de hielo a 0 ◦ C. Cuál es la temperatura y la masa finales del resto de hielo,
si el té caliente inicial tenía una temperatura de (a) 90 ◦ C y (b) de 70.0 ◦ C.

Solución
Qi = 0 aún. Hagamos para 70 ◦ C primero. En los 2
P
(b) Sigamos la recomendación, no planteemos el
casos tenemos hielo. Cuanto calor ocupamos para derretir todo el hielo y comparemolo con cuanto calor
ocupamos sacar del agua a 70 ◦ C antes de plantear la ecuación.
Calor para derretir todo el hielo

Qd = mh Lf = (0.520)(3.34 × 105 ) = 173680 J (7.1)

Calor para pasar el el agua de 70 → 0 ◦ C

Q70→0 = m` c` (0 − T`,i ) = (0.520)(4190)(0 − 70) = −152516 J

Comparando magnitudes, vemos que |Q70→0 | < |Qd |. El agua puede llegar de 70 → 0 ◦ C y aún queda
hielo para poder derretirse, pero ya llegaron a la misma temperatura, entonces ya dejan de transferirse
calor. La temperatura final es T = 0 ◦ C. La masa de hielo derretirda md la averiguamos con
X m` c` (T`,i )
Qi = md Lf + m` c` (0 − T`,i ) ⇒ md = (7.2)
Lf
i

(0.52)(4190)(70)
⇒ md = = 0.456 kg
3.34 × 105
Queda una masa de hielo de

mh,queda = mh − md = 0.52 − 0.456 = 0.064 kg

67
CAPÍTULO 7. LA PRIMERA LEY DE LA TERMODINÁMICA

(a) Veamos si podemos pasar todo el agua de 90 → 0 ◦ C sin derretir todo el hielo. El calor para derretir
es Qd = 173680 J, que ya lo habíamos calculado en la Ec 7.1.

Q90→0 = m` c` (0 − T`,i ) = (0.520)(4190)(0 − 90) = −196092 J

Vemos que a |Qd | < |Q90→0 |. Sí podemos derretir todo el hielo y el agua que estaba a 90 ◦ C aún no llega a
0 ◦ C, entonces el hielo derretido ahora agua empieza a aumentar la temperatura hasta llegar al equilibrio
térmico.
El sistema va a llegar a un equilibrio térmico con una temperatura entre 0 < Tf < 90 ◦ C con todo el hielo
derretido. Planteamos la ecuación de calorimetría
X
Qi = mh Lf + mh c` (Tf ) + m` c` (Tf − T`,i ) = 0
i

Cancelamos las masas que en este caso son iguales y despejamos


Lf
T`,i −
c`
Tf = (7.3)
2
Evaluamos
3.34 × 105
90 −
Tf = 4190 = 5.14 ◦ C
2
Sigamos viendo el por qué de la sugerencia, hagamos mal la parte (b) de problema apropósito, suponiendo
que todo se derrite y voy a encontrar la temperatura de equilibrio. Reusamos la Ec 7.3 con la temperatura
inicial de T`,i = 70 ◦ C
3.34 × 105
70 −
Tf = 4190 = −4.85 ◦ C
2
Nos da una temperatura menor a la del Hielo. Este resultado no tiene sentido, el sistema tiene que terminar
minimo a 0◦ , menor no puede. Si uno asume que va a terminar en un estado donde realmente no termina,
dan cosas extrañas.
Hagamos esta parte (a) mal apropósito también y pensemos que no se derrite todo el hielo. Usamos la
Ec. 7.2 con T`,i = 90 ◦ C tenemos

(0.520)(4190)(90)
md = = 0.587 kg
3.34 × 105
Ocupamos 0.587 kg de hielo derretido, pero solo tenemos 0.520 kg. Este resultado tampoco tiene sentido.
Es mejorPir comparando calores viendo donde va a terminar el sistema que estar bateando la ecuación
final de Qi

[Sears] Problema 17-57


Un vaso aislado con masa despreciable contiene 0.250 kg de agua a 75◦ C. Cuántos kilogramos de hielo de
−20.0◦ C deben ponerse en el agua para que la temperatura final del sistema sea 40.0◦ C?

Solución

68
CAPÍTULO 7. LA PRIMERA LEY DE LA TERMODINÁMICA

Aquí ya sabemos cual es el estado final del sistema, si sabemos todos los cambios de P estado que van a
haber. La recomendación es cuando no se sabe. Entonces si podemos poner la ecuación Qi de una vez.
Que es X
Qi = mh cs (0 − Ti,h ) + mh Lf + +mh c` (Tf − 0) +m` c` (Tf − Ti,` ) = 0
i
| {z } | {z } | {z }
1 2 3

El término 1 tiene el calor especifico del hielo y el rango de temperatura es solo cuando el “agua está
sólida”, desde Ti,h = −20 → 0 ◦ C. Cuidado ponen el ∆T de la forma ∆T = Tf − T`,i por que eso asume
que el agua tiene calor específico del hielo cuando ya está líquido. La parte 2 es el cambio de fase y la 3
es el hielo derretido que ahora tiene calor específico del agua líquida. De aquí despejamos mh que da

mh cl (Ti,` − Tf ) (0.25)(4190)(75 − 40)


mh = = = 0.0674
−cs Ti,h + Lf + cl Tf −(2100)(−20) + (3.35 × 105 ) + (4190)(40)

[Serway] Problema 20-28


(a) Determine el trabajo hecho en un gas que se expande desde i hasta f como se muestra en la Fig 7.1.
(b) Cuanto trabajo es hecho en el gas si este es comprimido de f hasta i en la misma trayectoria?

Figura 7.1: [Serway] Problema 20-28

Solución
Hay 2 maneras de sacar el trabajo. La primera es obtener P (V ) para las 3 curvas. La´ única curva que da
un poco de difícultad seria la parte inclinada. La otra es reconociendo que W = − P dV es el negativo
del área que está entre el eje de V y P (V ). Esta mánera en este caso es más facil.

Figura 7.2: [Serway] Problema 20-28

69
CAPÍTULO 7. LA PRIMERA LEY DE LA TERMODINÁMICA

Saquemos el trabajo calculando el área, dividamolo en 3 regiones como se muestra en la figura 7.2 y
usando que el area del trapecio es A = (b + B)h/2
ˆ f
W =− P dV = − [A1 + A2 + A3 ]
i
 
(6 + 2)
= 6(2 − 1) + (3 − 2) + 2(4 − 3) × 106 = −12 × 106 J = −12 MJ
2

(b) El trabajo en la dirección contraria sería


ˆ i ˆ f
W =− P dV = P dV = A1 + A2 + A3 = 12 MJ
f i

Tiene la misma magnitud pero con signo contrario.

(a) Método complicado: Obtengamos el mismo resultado de la parte solo que integrando P (V ). Para
obtener rápido la curva inclinada, usamos

(y − y0 ) = m(x − x0 ) ⇒ y = y0 + m(x − x0 )

La pendiente es
2−6 Pa
m= × 106 = −4 × 106 3
3−2 m
Y tomamos y0 = 6 × 106 Pa y x0 = 2 m3 . La presión P (V ) en la parte inclinada es

P (V ) = 6 × 106 − 4 × 106 (V − 2)

Ahora integramos
ˆ f ˆ 2 ˆ 3 ˆ 4 
6 × 106 dV + 6 × 106 − 4 × 106 (V − 2) dV + 2 × 106 dV = −12×106 J

W =− P dV = −
i 1 2 3

Se obtiene el mismo resultado.


[Serway] Problema 20-41
Un gas ideal inicialmente en Pi , Vi y Ti se pone a a traves del ciclo que se muestra en la figura. (a)
Encuentre el trabajo neto hecho sobre el gas por ciclo en 1.00 mol de gas inicialmente a 0 ◦ C. (b) Cuanto
es la energía neta añadida por calor al gas por ciclo?
Solución
Hay varias maneras de hacerlo. La manera facil usando
´ que el trabajo de un ciclo es el área que que
encierra la curva P (V ) y la otra en haciendo W = − P dV y Q = nc∆T con el c respectivo para cada
proceso.
Método 1 Es un ciclo, entonces ∆U = 0, entonces usando la primera ley tenemos

∆U = W + Q = 0 ⇒ Q = −W

El W es el el área del ciclo, que es

W = −A = −(3Pi − Pi )(3Vi − Vi ) = −4Pi Vi = −4nRT0

70
CAPÍTULO 7. LA PRIMERA LEY DE LA TERMODINÁMICA

Figura 7.3: [Serway] Problema 20-41

⇒ Q = −W = 4nRT0 (7.4)
Otro método es calcular el W haciendo la definición de integral y usar la primera ley ∆U = Q + W = 0
para obtener Q.
Método 2 Calculemos el Q para cada proceso. La temperaturas son TB = 3T0 , TC = 9T0 , TD = 3T0 . El
calor para cada proceso es
Q = QAB + QBC + QCD + QDA
QAB = nCv (TB − TA ) = nCv (3T0 − T0 ) = 2nCv T0
QBC = nCp (TC − TB ) = nCp (9T0 − 3T0 ) = 6nCp T0
QCD = nCv (TD − TC ) = nCv (3T0 − 6T0 ) = −6nCv T0
QDA = nCp (TA − TD ) = nCp (T0 − 3T0 ) = −2nCp T0
Sumando todos los términos y usando que en gases ideal cumple Cp − Cv = R

Q = 2nCv T0 + 6nCp T0 − 6nCv T0 − 2nCp T0

Agrupando términos

Q = −2n (Cp − Cv ) T0 + 6n (Cp − Cv ) T0 = −2nRT0 + 6nRT0 = 4nRT0


| {z } | {z }
R R

Obtenemos el mismo resultado de la Ec 7.4.


Evaluando con los datos, con T0 = 273 K

Q = 4(1.00)(8.314)(273) = 9.08 kJ

[Serway] Problema 20-29


Un gas ideal se toma a traves de un proceso cuasiestático descrito por P = αV 2 , con α = 5.00 atm/m6 ,
como se muestra en la figura. El gas se expande al doble del volumen original de 1.00 m3 . Cuanto trabajo
es realizado sobre el gas expandiendose en este proceso?

´ En este problema ya conocemos P (V ). Para calcular el trabajo, solo tenemos que integrar
Solución
W = − P dV .
ˆ ˆ V2 ˆ V2
α
αV 2 dV = − V23 − V13

W =− P d=− P (V ) dV = −
V1 V1 3

71
CAPÍTULO 7. LA PRIMERA LEY DE LA TERMODINÁMICA

Figura 7.4: [Serway] Problema 20-29

1
⇒ W = − [5.00] 1.013 × 105 23 − 13 = −1.18 MJ
  
3
Donde en 1.013 × 105 sale para hacer conversión que el α está en atm.

[Resnick] Problema 23-19


Un motor lleva 1.00 mol de un gas monoatómico ideal durante el ciclo que se describe en la figura. El
proceso AB tiene lugar a volumen constante, el proceso BC es adiabático y el proceso CA se produce a
una presión constante. (a) Encuentre el calor Q, el cambio de enegía intenta Eint , y el trabajo W en los
tres procesos y en el ciclo en general. (b) Si la presión en el punto A es 1.00 atm, encuentre la presión y
el volumen en los puntos B y C. Utilice 1 atm = 1.013 × 105 Pa y R = 8.314 J/(mol K).
Extra: (Esto es para el cápitulo 22) Calcule la eficiencia de la máquina y la eficiencia de Carnot

Figura 7.5: [Resnick] Problema 23-19

Solución
(b) Es un gas monoatómico, los calores específicos son Cv = 3R/2 y Cp = 5R/2, y γ = 5/3 Obtengamos
los valores de P V T para los puntos A, By C.
Los números negros son los datos iniciales y los de color son los que se van averiguando en la tabla 7.1 de
PV T
El volumen VA lo encontramos con

nRTA (1.00)(8.314)(300)
VA = = = 0.0246 m3
PA (1.013 × 105 )

72
CAPÍTULO 7. LA PRIMERA LEY DE LA TERMODINÁMICA

Cuadro 7.1: Valores de P V T .


[Resnick] Problema 23-19

P (atm) V (m3 ) T (K)


A 1.00 0.0246 300
B 2.00 0.0246 600
C 1.00 0.0373 455

De A → B es volumen constante, entonces VB = VA = 0.0246 m3 y encontramos PB con


PA PB TB 600
= ⇒ PB = PA = PA = 2PA = 2.00 atm
TA TB TA 300
De C → A vemos que PC = PA = 1.00atm. El valor de Vc lo encontramos con

VA VC TC (455)
= ⇒ VA = (0.0246) = 0.0373 m3
TA TC TA (300)

(a) Sabiendo todos los valores de P V T , encontremos el ∆U, Q y W para cada proceso.
Proceso A → B
Es volumen constante ⇒ el trabajo es WA→B = 0.
El calor es
 
3 3
QA→B = nCv ∆T = n R (TB − TA ) = (1.00) (8.314)(600 − 300) = 3471 J
2 2

Usando la primera ley


∆U = W + Q ⇒ ∆UA→B = QA→B = 3741 J
Proceso B → C Es un proceso adiabático, el calor es QB→C = 0. El trabajo es

PB VB − PA VA [((1)(0.0373) − (2)(0.0246))]
1.013 × 105 = −1808 J

WB→C = =
γ−1 5
−1
3
El ∆UB→C lo podemos encontrar de 2 maneras. Podemos usar la primera ley

∆UB→C = QB→C + WB→C = WB→C = −1808 J

O podemos usar que para gases ideales, para cualquier proceso cumple que
 
3 3
∆U = nCv ∆T = n R (TC − TB ) = (1.00) (8.314)(455 − 600) = −1808 J
2 2

Da lo mismo con ambos cálculos.


Proceso C → A
El proceso es presión constante. Usamos el Cp para el calor
 
5 5
QC→A = nCp ∆T = n R (TA − TC ) = (1.00) (8.314)(300 − 455) = −3221 J
2 2

73
CAPÍTULO 7. LA PRIMERA LEY DE LA TERMODINÁMICA

El trabajo es
ˆ ˆ
dV = −PA (VA − VC ) = − 1.013 × 105 (0.0246 − 0.0373) = 1286 J

WC→A = − P dV = −P

El DeltaU es
 
3 3
∆UC→A = nCv ∆T = n R (TA − TC ) = (1.00) (8.314)(300 − 455) = −1933 J
2 2
También se pudo haber usado la primera ley

∆UC→A = WC→A + QC→A = 1286 − 3221 = −1935 J

Da lo mismo. Tenemos una pequeña diferencia de 1935 -1933 = 2 J, pero estamos en la escala de los miles,
es un error de 0.1 %. Sale por arrastre numérico, así que estamos bien.

Cuadro 7.2: Valores de ∆U, Q y W en [J]


[Resnick] Problema 23-19

∆U W Q
A→B 3741 0 3741
B→C -1808 -1808 0
C→A -1933 1286 -3221

Para comprobar que lo hicimos bien, el ∆U de todo el ciclo nos debería dar 0.

∆Uneto = ∆UA→B + ∆UB→C + ∆UC→A = 3741 − 1808 − 1933 = 0 J

Estamos bien. También podemos corroborar que tiene que cumplir que Wneto + Qneto = 0. Es la mismo
significado que el cálculo pasado, que el ∆U tiene que dar 0 para todo el ciclo, pero lo cálculamos con
rutas diferentes, igual debería dar 0.

Wneto = WA→B + WB→C + WC→A = 0 − 1808 + 1286 = −522 J

Qneto = QA→B + QB→C + QC→A = 3741 + 0 − 3221 = 520 J


∆Uneto = Wneto + Qneto = −522 + 520 = −2 J
No nos dió 0, pero en la escala de miles, es de nuevo un error de 0.1 %, podemos ser felices. Nos “da” 0.
Extra: El calor QH es la suma de todos los calores positivos que entran a la máquina. En este caso
QH = QAB La eficiencia por definición es
Wmaq 522
e= = = 0.14
QH 3741
Comparando con la eficiencia de Carnot, la temperatura más baja es TC = TA = 300 K y la más alta es
TH = TB = 600 K. La eficiencia es

[Sears] Problema 19-51


Al empezar con n = 2.50 moles de N2 gaseoso (que se supone ideal) en un cilindro de Pa = 1.00 atm y
Ta = 20.0 ◦ C, un químico calienta primero el gas a volumen constante, agrega Qab 1.52 × 104 J de calor,

74
CAPÍTULO 7. LA PRIMERA LEY DE LA TERMODINÁMICA

luego continúa calentando y permite que el gas se expanda a presión constante al doble de su volumen
original. (a) Calcule la temperatura final del gas. (b) Determine la cantidad de trabajo efectuado por el
gas. (c) Calcule la cantidad de calor agregado al gas mientras se expande. d) Calcule el cambio de energía
interna del gas en todo el proceso.

Solución
(a) El N2 es un gas diatómico. Los calores específicos son Cv = 5R/2 y Cp = 7R/2. El proceso completo
es como se muestra en la figura 7.6

Figura 7.6: Proceso en el diagrama P V .


[Sears] Problema 19-51

Hallamos Tb usando el calor Qab = 1.52 × 104 J.


5 Q
Qab = nCv ∆T = n R (Tb − Ta ) ⇒ Tb =  ab + Ta
2 5
n R
2

Evaluando
1.52 × 104
Tb =   + 293 = 586 K
5
(2.50) (8.314)
2
Encontramos la temperatura Tc con
Vb Vc Vc
= ⇒ Tc = Tb = 2(586) = 1172 K
Tb Tc Vb

Donde usamos que Vc /Vb = 2, aunque no sepamos cuando vale Vb ó Vc , sabemos cuanto es la proporción.
(b) Esto 2 maneras de hacerlas, una un poco más rápida que la otra. El trabajo neto es Wneto =
Wab + Wbc = Wbc por que Wab es 0.
Manera “rápida” La manera un poco más rápida es viendo que ocupamos W = −P ∆V = −Pb (Vc − Vb ),
pero no ocupamos avergiuar los valores explícitos de Pb , Vb ni Vc . Hagamos la resta de las Ec. 7.5 y 7.6

P Vf = nRTf (7.5)

P Vi = nRTi (7.6)
P (Vf − Vi ) = −P ∆V = nR(Tf − Ti ) = nR∆T (7.7)

75
CAPÍTULO 7. LA PRIMERA LEY DE LA TERMODINÁMICA

Y sí conocemos los valores de n, Tb y Tc . El trabajo neto es

W = Wbc = −P ∆V = −nR∆T = −nR (Tc − Tb ) = −(2.5)(8.314)(1172 − 586) = −12180 J (7.8)

Manera “no tan rápida” Esto es desviandose a averiguar datos que no nos están pidiendo como el
Pb , Vb , Vc . Averiguemolos.
Sabemos que Pa = 1.00 atm y Ta = 293K. Entonces
nRTa (2.5)(8.314)(293)
Va = = = 0.0601 m3
Pa (1.013 × 105 )
De a → b es volumen constante, ⇒ Vb = Va .
nRTb (2.5)(8.314)(586)
Pb = = = 202662 Pa
Vb 0.0601

Y sabemos del enunciado que Vc = 2Vb = 2(0.0601) = 0.1202 m3 . Ahora podemos calcular el trabajo Wbc

Wbc = −Pb (Vc − Vb ) = − (202662) (0.1202 − 0.0601) = −12180 J

Que da lo mismo que la Ec. 7.8.


(c) El calor mientras el gas se expande es el calor de b → c. De a → b no se expande
 
7 7
Qbc = nCp ∆T = n R (Tc − Tb ) = (2.5) (8.314)(1172 − 586) = 42630 J
2 2

(d) Esto tenemos 2 maneras de hacerlo. Veamos que da lo mismo por las 2 para comprobar. Usemos la
fórmula directa, que para cualquier proceso en gases ideales, cumple que
 
5 5
∆Uac = nCv ∆T = n R (Tc − Ta ) = (2.5) (8.314)(1172 − 293) = 45675 J
2 2
Usemos otro método para comprobar. El trabajo de a → c es Wac = Wab + Wbc = Wbc ya que Wab = 0.
Igual para el calor, el calor neto es Qac = Qab + Qbc . Entonces

∆Uac = Wac + Qac = Wab + Wbc + Qab + Qbc

∆Uac = 0 − 12180 + 15200 + 42630 = 45650 J

El resultado nos difiere de 45675-45650 = 25 J. Eso es un error cerca del 0.05 %, es solo arrastre de cosas
númericas. Seamos felices, da lo mismo.
Todo el problema se pudo haber resuelto sin averguiar los valores de Pb , Vb , Vc .
[Resnick] Problema 23-1
(a) Calcule la rapidez con que se pierde calor a través de una ventana de vidrio cuya superficie mide A =
1.4 m2 y cuyo espesor es de Lv = 3.0 mm, si la temperatura externa es de −20 ◦ F y si la interna es 72 ◦ F.
(b) Se instala una contraventana que tiene el mismo espesor pero un intervalo de aire de La = 7.54 cm
entre las dos ventanas. Cuál será la rapidez correspondiente de la pérdida de calor, si se supone que la
conducción es el único mecanismo importante de la pérdida de calor? Extra: (c) Suponga que se quiere
conseguir el mismo aislamiento térmico (misma transferencia de energía) usando solo vidrio que tenga la
misma área que la parte (b) y sin aire dentro. Cual debe ser el grosor L0v del vidrio?

76
CAPÍTULO 7. LA PRIMERA LEY DE LA TERMODINÁMICA

Datos adicionales: Vidrio: kv = 1.0 W/(m K), Aire: kv = 0.026 W/(m K).

Solución
Para obtener la potencia, ocupamos en ∆T . Las temperaturas que nos están dando están en Fahrenheit
y las ocupamos en Celsius. Se puede probar de la misma manera que se obtuvo la Ec. 7.7, sacando la
diferencia entre 2 puntos y usando la conversión de TF = (9/5)TC + 32 se obtiene
9 5 5
∆TF = ∆TC ⇒ ∆TC = ∆TF = [72 − (−20)] = 51.1 ◦ C
5 9 9
Usemos nada mas ∆T = 51.1◦ C sin el subíndice c de aquí en adelante.

Figura 7.7: Vidrio, parte (a).


[Resnick] Problema 23-1

La potencia es
∆T ∆T (51.1)
Pa = k v A = kv A = (1.0)(1.4) = 23846 W
∆x Lv (0.003)
(b) Ahora ponemos aire entre dos ventanas. El calor pasa por el vidrio → aire → vidro. Los medios están
en serie La potencia de medios en serie con la misma área es

Figura 7.8: Vidrio, parte (b).


[Resnick] Problema 23-1

∆T A∆T
Pb = A X =X
Ri Li
i ki
i

77
CAPÍTULO 7. LA PRIMERA LEY DE LA TERMODINÁMICA

Sumando para los 3 medios


∆T ∆T (51.1)
Pb = A   = A  = (1.4)     = 24.75 W
Lv La Lv Lv La 0.003 0.075
+ + 2 + 2 +
kv ka kv kv ka 1.00 0.026
(c) Queremos que la potencia que pase por este vidrio sea Pb . Despejamos L0v de
∆T ∆T (51.1)
Pb = kv A 0
⇒ L0v = kv A = (1.0)(1.4) = 2.89 m
Lv Pb (24.75)

[Sears] Problema 17-113


Una cabaña rústica tiene un piso cuya área es de 3.50 m × 3.00 m. Sus paredes, que miden 2.50 m de
alto, están hechas de madera (conductividad térmica de kw 0.0600 W/m·K) de Lw 1.80 cm de grosor y
están aisladas con Ls 1.50 cm de un material sintético. Cuando la temperatura exterior es de 2.00 ◦ C, es
necesario calentar la habitación a una tasa de P = 1.25 kW para mantener su temperatura a 19.0 ◦ C.
Calcule la conductividad térmica del material aislante. Desprecie la pérdida de calor a través del techo y
el piso. Suponga que las superficies internas y externas de la pared tienen la misma temperatura que el
aire en el interior y afuera de la cabaña.

Solución
Nos están diciendo que se ocupa mantener la temperatura del cuarto a 19.0 ◦ C. La temperatura del cuarto
es una constante. Como la temperatura es constante, la energía en el cuarto es constante, pero tenemos
2 flujos de energía en el tiempo, el calentador que brinda energía con una potencia P = 1.25 kW y la
transferencia de calor por las paredes, pongalome Pout . Como la energía del cuarto es constante, toda la
energía que entra es igual a la que sale.
X
Pi = P − |Pout | = 0 ⇒ P = |Pout |
i

La potencia que sale por calor es igual a la potencia de entrada que hace el calentador. Encontremos la
potencia de salida.
El ∆T entre el interior y el exterior es 19.0 − 2.00 = 17◦ C.
Veamos una pared indiviual ahorita. El calor pasa por los dos materiales (El orden aquí no importa). Los
materiales están en serie.

Figura 7.9: Materiales en serie.


[Sears] Problema 17-113

La resistencia térmica equivalente es


X X Li Lw Ls
Ri = = +
ki kw ks
i i

78
CAPÍTULO 7. LA PRIMERA LEY DE LA TERMODINÁMICA

Donde w es para la madera y s para el material sintético.


Las 4 caras que conducen calor se muestran en la Fig. 7.9. El área Ai conduce la potencia Pi . Viendo la
figura 7.10, las áreas cumplen que A1 = A3 y A2 = A4 , con valores de

A1 = A3 = (2.5)(3.0) = 7.5 m2

A2 = A4 = (2.5)(3.0) = 8.75 m2
Tenemos 4 caras conduciendo calor que están en paralelo. La potencia total es la suma de las potencia

Figura 7.10: Las potencias hacia fuera.


[Sears] Problema 17-113

de cada cara. Cada i−ésima cara tiene un potencia de


∆T ∆T
Pi = ki Ai P = ki Ai  
R
i i Lw Ls
+
kw ks

La potencia de salida es
Pout = P1 + P2 + P3 + P4 = P
∆T ∆T ∆T ∆T
⇒ P = A1   + A2   + A3   + A4  
Lw Ls Lw Ls Lw Ls Lw Ls
+ + + +
kw ks kw ks kw ks kw ks
Usando que A1 = A3 y A2 = A4 y factorizando términos

2 (A1 + A2 ) ∆T
P =  
Lw Ls
+
kw ks

Despejando ks
Ls 0.015 W
⇒ ks =  =  = 0.1056
2 (A1 + A2 ) ∆T Lw 2 (7.5 + 8.75) (17) 0.018 m·K
− −
P kw (1250) 0.0600

II Parcial 2017 II Ciclo Pregunta 1


Una caja con dimensiones a = 50 cm, b = 40 cm y c = 30 cm con grosor ∆x = 1 cm, hecha de acero
inoxidable conductividad térmica k = 14 W/(m·K), contiene una masa de hielo de m = 55 kg a tempera-
tura T = 0 ÂřC. La temperatura al exterior de la caja es de 20 ◦ C. Calcule: (a) La tasa de transferencia
de energía del ambiente al interior de la caja, (b) el tiempo que tarda en derretirse todo el hielo, (c)

79
CAPÍTULO 7. LA PRIMERA LEY DE LA TERMODINÁMICA

Suponga que ahora usted desea aumentar la temperatura exterior a la caja para que se derrita todo el
hielo en 5 minutos. Calcule dicha temperatura . Nota: Todas las caras se encuentran expuestas al ambiente.

Solución
(a) Tenemos 6 caras, 2 con área ab, 2 con área ac y las otras 2 con área bc, todas las caras en paralelo y
con grosor ∆x. La potencia que le entra a la caja es la suma de las potencias de cada cara.
∆T ∆T ∆T ∆T ∆T ∆T
P = kab + kab + kac + kac + kbc + kbc
∆x ∆x ∆x ∆x ∆x ∆x
Juntando términos y evaluando
∆T
P = 2k(ab + ac + bc) (7.9)
∆x
(20 − 0)
⇒ P = 2(14) [(0.5)(0.4) + (0.5)(0.3)(0.4)(0.3)] = 26320 W
(0.01)
(b) La potencia P de entrada es constante, entonces la energía que entra a la caja en un tiempo tb es
E = P tb . La cantidad de energía que se ocupa para derretir la masa m de hielo es E = mLf . Igualando
y despejando
P tb = mLf (7.10)
mLf (55)(3.34 × 105 )
⇒ tb = = = 697.95 s
P 26320
(c) Sea TH0 la nueva temperatura para derretir el hielo en tc = 5 min, usamos la Ec 7.9 con un nuevo
∆T = TH0 − 0 y la Ec. 7.10 y despejamos TH0

(TH0 − 0)
 
0
P tc = mLf ⇒ P = 2k(ab + ac + bc) tc = mLf
∆x

∆x mLf 0.01 (55)(3.34 × 105 )


⇒ TH0 = = = 46.53 ◦ C
2k(ab + ac + bc) tc 2(14) [(0.5)(0.4) + (0.5)(0.3)(0.4)(0.3)] (60)(5)

80
8 | Teoría Cinética de los Gases

Más problemas de ciclos con calor, cambios de energía, o cambios adiabáticos se están otros capítulos,
aquí es para incluir problemas con vrms .
[Knight] Problema 20-61
La velocidad rms de m = 1.0 g de moléculas de H2 gaseoso es de vrms,i = 1800 m/s. (a) Cual es la energía
total del gas? (b) Se hace 500 J de trabajo para comprimir el gas. En este mismo proceso, 1200 J de
energía forma de calor es transferido del gas al ambiente. Cual es la velocidad final rms de las moléculas
después del proceso?
La masa molar del H2 es M = 2.0 g/mol

Solución
Calculemos la cantidad de moles
m 0.001 1
n= = = mol
M 0.002 2
(a) Ya que conocemos la vrms,i y la masa molar M del gas, podemos determinar la temperatura del gas
con
2
r
3RTi Mvrms,i (0.002)(1800)2
vrms,i = ⇒ Ti = = = 260 K
M 3R (3)(8.314)
El H2 es un gas diatómico, tiene 5 grados de libertad. Usando el teorema de equipartición, la energía total
del gas es   
5 5m 5 1
Ei = nRTi = RTi = (8.314)(260) = 2702 J = 2.7 kJ
2 2M 2 2
(b) Usando la convención ∆E = W + Q, como el gas se comprimera, el trabajo es hacia el gas, es
positivo, ⇒ W = 500 J. El calor es transferido de gas al ambiente, va hacia fuera del sistema, es negativo,
⇒ Q = −1200 J. Y usando que el gas es diatómico, el calor específico es cv = 5R/2. Con la primera ley
tenemos
5
∆E = W + Q = ncv (Tf − Ti ) = n R (Tf − Ti )
2
Despejando Tf
W +Q (500 − 1200)
Tf = Ti +   = 260 +     = 193 K
5 1 5
n R (8.314)
2 2 2
Con Tf , calculamos la vrms,f
r r
3RTf (3)(8.314)(193) m
vrms,f = = = 1551
M 0.002 s

81
CAPÍTULO 8. TEORÍA CINÉTICA DE LOS GASES

II Parcial I Ciclo 2017 Pregunta 1


Las nubes moleculares son regiones del espacio que contienen gas hidrógeno H2 , y donde frecuentemente
se forman estrellas. El peso molecular del H2 es de M = 2.01589 g/mol. (a) Calcule la presión de una
nube molecular que contiene 106 moléculas por centímetro cúbico a una temperatura de T = 20 K, (b)
Calcule la rapidez cuadrática media de las moléculas del gas, (c) Calcule la energía cinética traslacional1
por unidad de volumen que tiene el gas. (d) Si el gas se expande adiabáticamente hasta duplicar su
volumen, calcule la nueva presión y temperatura.

Solución
El dato de “106 moléculas por centímetro cúbico” es una densidad de partículas, digase N/V , donde N
son el número de partículas y V el volumen.2 Haciendo conversión a m3
N partículas partículas
= 106 3
= 1012
V cm m3
No conocemos N ni V , pero podemos obtener la proporción N/V con la ecuación de estado del gas ideal
 
N
P V = N kB T ⇒ P = kB T
V
Y evaluamos que ya conocemos los datos

P = 1012 1.38 × 1023 (20) = 2.76 × 10−10 Pa


 

(b) Con saber la T y el tipo de gas, sabemos la vrms


r s
3RT 3(8.314)(20) m
vrms = = = 497.4
M (2.01589 × 10−3 ) s

(c) Nos están preguntando por E/V . La contribución de la energía cinética traslacional promedio es
3
E = N kB T
2
Dividimos entre V
   
E 3 N 3 J
1012 1.38 × 10−23 (20) = 4.14 × 10−10 3
 
= kB T =
V 2 V 2 m

(d) El enunciado nos dice que Vf = 2V0 y se expande de manera adiabática. EL H2 es diatómico ⇒ γ =
7/5. Entonces  γ  γ
γ γ Vi 1
Pi Vi = Pf Vf ⇒ Pf = Pi = Pi
Vf 2
 7/5
 1
⇒ Pf = 2.76 × 10−10 = 1.01 × 10−10 Pa
2
1
El enunciado original decía interna en lugar de cinética traslacional, pero a como venía en la solución de ese examen,
preguntaban por esa y no la enería interna total (o nada más, energía interna), ya que esa hay que incluir la energía por
rotación que el H2 es diatómico.
2
Nos la están dando en conjunto, no por aparte, no se dan cuantas partículas hay o cuanto es el volumen, no puedo
separarlos diciendo que V = 1 m3 que he visto gente haciendo esa mala maña, asumir datos que no nos dan.

82
CAPÍTULO 8. TEORÍA CINÉTICA DE LOS GASES

La temperatura podemos encontrar de 2 maneras:


La primera es usando la Ec. del gas ideal

Pi V i Pf Vf Pf Vf (1.04 × 10−10 )
= ⇒ Tf = Ti = (20) (2) = 15.07 K
Ti Tf Pi Vi (2.76 × 10−10 )

Usando que es una expansión adiabática. Con está expresión no ocupamos saber la Pf a diferencia de
usar la Ec del gas ideal
 γ−1  (7/5)−1
Vi 1
Ti Viγ−1 = Tf Vfγ−1 ⇒ Tf = Ti = (20) = 15.15 K
Vf 2

Casi que lo mismo. Diferencias por cuestiones numéricas.

II Parcial 2016 II Ciclo Pregunta 4


Un gas ideal experimenta una compresión adiabática de P1 = 122 kPa, V1 = 10.7 m3 , T1 = −23 ◦ C a P2
= 1450 kPa, V2 = 1.36 m3 . Determine: (a) la razón de las capacidades caloríficas molares, (b) el trabajo
realizado sobre el gas, (c) la temperatura final del gas, (d) la energía cinética traslacional total antes y
después de la compresión, (e) la razón de las rapideces rms antes y después de la compresión.

Solución
La temperatura T1 es T1 = −23 + 273 = 250 K.
(a) Es una expansión adiabática, entonces
 γ
P1 V2
P1 V1γ = P2 V2γ ⇒ =
P2 V1
 
P1
   γ    ln
P1 V2 V2 P2
⇒ ln = ln = γ ln ⇒γ=  
P2 V1 V1 V2
ln
V1
 
122
ln
1450
⇒γ=   = 1.20
1.36
ln
10.7
(b) Sabiendo γ, obtenemos el trabajo con

P2 V 2 − P1 V 1 (1450)(1.36) − (122)(10.7) 3
W = = (10 ) = 3.33 MJ
γ−1 (1.2 − 1)

(c) La temperatura lo podemos encontrar de 2 maneras igual que el problema pasado, con el gas ideal o
usando que es una expansión adiabática.
Por gas ideal   
Pi Vi Pf Vf Pf V f 1450 1.36
= ⇒ Tf = Ti = (250) = 377.6 K
Ti Tf Pi Vi 122 10.7

83
CAPÍTULO 8. TEORÍA CINÉTICA DE LOS GASES

Por compresión adiabática


 γ−1  1.2−1
Vi 10.7
Ti Viγ−1 = Tf Vfγ−1 ⇒ Tf = Ti = (250) = 377.6 K
Vf 1.36

(d) Averiguemos los moles


122 × 103 (10.7)

PV
n= = = 628 mol
RT (8.314)(250)
El tipo de gas no lo sabemos. El γ = 1.2 = 6/5 no encaja en alguno de mono/di/poli-atómico. Pero no
importa. La contribución de la energías cinetical traslacional sigue siendo la misma independiente del tipo
de gas.
Veamos la energía de una partícula
1 1 1
E = mvx2 + mvy2 + mvz2 + Eotro
2 2 2
Donde Eotro son otros tipos de energías (cinetica rotacional, vibracional...). Si promediamos, para los
primeros términos, el teorema de equipartición me asegura que tengo una energía promedio de (1/2)kB T
para cada uno      
1 1 1
mvx2 + mvy2 + mvz2 + Eotro



E =
2 2 2
| {z } | {z } | {z }
1 1 1
2 kB T 2 kB T 2 kB T

3

E = kB T + Eotro
|2 {z }
Cinética
Así que la contribución cinética traslacional sigue siendo (3/2)kB T independiente del tipo de gas. Esca-
landolo para todo el gas, la energía cinetica promedio total es3
3 3
E = N kB T = nRT
2 2
Evaluamos para las condiciones iniciales y finales
3
Ei = (628)(8.314)(250) = 1.96 × 106 J
2
3
Ef = (628)(8.314)(377.6) = 2.96 × 106 J
2
(e) Nos esán pidiendo la razón. No podemos saber la vrms por que no sabemos la masa de cada partícula
(o masa molar) del gas, pero es una proporción, si dividimos, nos quitamos ese término. La vrms es
r
3kB T
vrms =
m
Haciendo la razón r
3kB Tf s r
vrms,f m Tf 377.6
= r = = = 1.23
vrms,i 3kB Ti Ti 250
m
3
Ya abandonando la notación promedio.

84
CAPÍTULO 8. TEORÍA CINÉTICA DE LOS GASES

Problemas de desafío
[Knight] Problema 20-65
Un experimento que usted esta diseñando requiere un gas con un γ = 1.50. Usted recuerda de sus clases
de física que no hay gases individuales que tengan este valor, pero a usted se le ocurre que puede producir
un gas que tenga un γ = 1.50 haciendo una mezcla de un gas monoatómico y un gas diatómico. Que
fracción de las moléculas del gas tienen que ser monoatómico?
Nota: Para tener una idea de como hacer, se puede probar en general que, si se tienen 2 gases, con una
fracciónes de la cantidad de total de moles f1 y f2 , con coeficientes adiabáticos γ1 y γ2 para cada gas, se
puede llegar a la relación
1 f1 f2
= +
γ−1 γ1 − 1 γ2 − 1
Donde γ sería el coeficiente adiabático equivalente de los 2 gases combinados. Si se prueba eso, de inme-
diato se resuelve el problema inicial.

Solución
Resolvamolo por comparación de términos. Primero para un gas de un solo tipo. Para la relación adiabática
con T y V es
T V γ−1 = Constante
Saquemos la derivada total
V γ−1 dT + T (γ − 1)V γ−2 dV = 0
Reacomodando
1 dT dV
+ =0 (8.1)
γ−1 T V
Si encontramos una ecuación diferencial de la forma
dT dV
K + =0
T V
Lo que vaya en ese K por comparación es K = 1/(γ − 1).
Tomemos en cuenta esta relación
Cv Cv 1 1
= =  = (8.2)
R Cp − Cv Cp γ−1
−1
Cv
Ahora hagamolo para 2 gases.
Escribamos n1 y n2 como n1 = f1 n y n2 = f2 n donde n son los moles totales en el gas y fi la fracción
respectiva. El cambio de energía diferencial en el gas es

dU = n1 Cv,1 dT + n2 Cv,2 dT = (f1 Cv,1 + f2 Cv,2 ) ndT (8.3)

Usando la primera ley, tenemos

dU = dW + dQ ⇒ dQ = dU − dW

Es un proceso adiabático, entonces dQ = 0. Cambiamos el dU por lo obtenido en la Ec 8.3 y dW = −P dV


y obtenemos
(f1 Cv,1 + f2 Cv,2 ) ndT + P dV = 0

85
CAPÍTULO 8. TEORÍA CINÉTICA DE LOS GASES

Cambiamos el P por P = nRT /V


nRT
(f1 Cv,1 + f2 Cv,2 ) ndT + dV = 0
V
Reordenando, diviviendo entre T y quitando el n
 
Cv,1 Cv,2 dT dV
f1 + f2 + =0
R R T V
Ahora usamos la relación obtenida en la Ec 8.2 para cada gas
 
f1 f2 dT dV
+ + =0
γ1 − 1 γ2 − 1 T V
Comparando términos con la Ec 8.1, llegamos a
1 f1 f2
= +
γ−1 γ1 − 1 γ2 − 1
Cambiemos a los datos númericos del problema. Si 1 es el gas monoatómico y el 2 el diatómico, entonces
γ1 = 5/3 y γ2 = 7/5. Además con la restricción f1 + f2 = 1 ⇒ f2 = 1 − f1 , la Ec 8.1 sería
1 f1 1 − f1
=  + 
1.50 − 1 5 7
−1 −1
3 5
Despejando f1 , obtenemos f1 = 0.5. Para obtener un γ equivalente de γ = 1.50, se ocupa que la mitad
del gas sea monoatómico y la otra mitad sea diatómico.

Problema: Concentración Atmosférica


Suponga que la atmósfera está compuesta solamente4 de O2 y N2 . Usando que la densidad del aire es
ρ = 1.2250 kg/m3 a una presión de P = 101.325 kPa y T = 15 ◦ C. Determine la fracción molar por
unidad de volumen del O2 y N2 .
La masa molar del O2 es MO = 31.999 g/mol y del N2 es MN = 28.013 g/mol.
Sugerencia: Puede determinar primero la fracción de masa por unidad de volumen que ocupa cada gas y
de ahí obtener la fracción molar por unidad de volumen. La fracción de masa para cada gas no son iguales
a las fracciones molares respectivas.

Solución
Sea yO y yN las fracciones molares del O2 y N2 , encontremos primero las fracciones de masa fO y fN .
Por sus definiciones, fO y fN son
mO mN
fO = fN = (8.4)
m m
donde m es la masa total de gas en un volumen V . Usando la ecuación de estado del gas ideal, donde
usamos que la cantidad total de moles es n = nO + nN

P V = nRT = (nO + nN ) RT
4
Entre el O2 y el N2 componen alrededor del 98 % de la atmósfera, hacer esta supocisión no está mal para efectos del
problema.

86
CAPÍTULO 8. TEORÍA CINÉTICA DE LOS GASES

Cambiamos los nk por por sus masas respectivas nk Mk = mk ⇒ nk = mk /Mk


 
mO mN
PV = + RT
MN MN
y cambiamos por las definiciones de fracciones con la Ec. 8.4
   
fO m fN m fO fN
PV = + RT = + mRT
MN MN MN MN
   
fO fN  m  fO fN
⇒P = + RT = + ρRT
MN MN V MN MN
Usemos que las fracciones tiene que sumar fO + fN = 1 ⇒ fN = 1 − fO
 
fO 1 − fO
⇒P = + ρRT
MN MN
Despejando fO obtenemos  
MO MN P
fO = −1
MN − MO ρRT
Evaluando con los datos
(28.013 × 10−3 )(101325)
 
31.999
fO = −1 = 0.2634
28.013 − 31.999 (1.2250)(8.314)(15.273.15)
Entonces la fracción de masa de N2 es fN = 1 − fO = 1 − 0.2633 = 0.7366.
Con las fracciones de masa, podemos obtener las fracciones molar con las definiciones y expresandolas en
términos de las fracciones de masa
mO fO
nO nO M MO
yO = = = mO OmN =
n nO + nN + fO m fN m
MO MN +
MO MN
fO
MO
⇒ yO =
fO fN
+
MO MN
Igual para yN , tiene la forma
fN
MN
yN =
fO fN
+
MO MN
Evaluando para yO  
0.2634
31.999
⇒ yO =     = 0.2384
0.2634 0.7366
+
31.999 28.013
La concentración de N2 es
⇒ yN = 1 − yO = 1 − 0.2384 = 0.7654
Nos acercamos bastante a los datos reales de yO = 0.2095 y yN = 0.7809.

87
9 | La Segunda Ley de la Termodinámica

[Resnick] Ejercicio 23-17


Reposición II Parcial 2016 II Ciclo Pregunta 3, II Parcial 2017 I Ciclo Pregunta 4
Un tazón de cobre de mc1 = 146 g contiene m` = 223 g de agua. El tazón y el agua tiene una temperatura
de T1,i = 21.0 ◦ C. Se deja caer un cilindro muy caliente de cobre de mc2 = 314 g. Esto hace que el agua
hierva, mv = 4.70 g se convierten en vapor y la temperatura final del el sistema entero es Tf = 100 ◦ C
Determine: (a) la temperatura original T2,i del cilindro, (b) el cambio de entropía el sistema.
Datos adicionales: C` = 4186 J/(kg · K), Lv = 2.26 × 106 J/kg, Cc = 387 J/(kg · K)

Solución
(a) Ya sabemos el estado final del sistema. El agua debe llegar a 100 ◦ C por que ya está pasando a vapor,
pero ◦
Pno paso de 100 C por que no se evaporó todo al llegar al equilibrio. Sí podemos plantear ya la Ec
de Qi por que sabemos todos los términos que van a aparecer.
X
Qi = mc1 Cc (Tf − T1,i ) + m` c` (Tf − T1,i ) + mv Lf + mc2 Cc (Tf − T2,i ) = 0
i

Despejamos T2,i
mc1 Cc (Tf − T1,i ) + m` c` (Tf − T1,i ) + mv Lf
T2,i = Tf +
mc2 Cc
(0.146)(387)(100 − 21) + (0.223)(4186)(100 − 21) + (0.0047)(2.26 × 106 )
⇒ T2,i = 100 +
(0.314)(387)
⇒ T2,i = 831 ◦ C

(b) El cambio de entropía para cambios de temperatura sin cambio de fase es


ˆ ˆ Tf  
dQ mCdT Tf
∆S = = = mC ln
T Ti T Ti

Para cambios es fase es ˆ ˆ


dQ 1 dQ mLf ase
∆S = = =
T Tf ase T Tf ase
Hay 3 componentes el agua, el cobre 1 y el cobre 2, el cambio de entropía total es1

∆S = ∆S` + ∆Sc1 + ∆Sc2


1
Esto se evalúa con la temperatura absoluta (Kelvin), por favor, no lo olviden.

88
CAPÍTULO 9. LA SEGUNDA LEY DE LA TERMODINÁMICA

     
Tf md Lv Tf Tf
= m` C` ln + + mc1 Cc ln + mc2 Cc ln
T1,i (373) T1,i T1,i
(0.0047)(2.26 × 106 )
     
373 373 373
= (0.223)(4186) ln + + (0.146)(387) ln + (0.314)(387) ln
294 (373) 294 1104
J
⇒ ∆S = 132.23
K

[Serway] Problema 22-74 y 22-77


22-74 Un sistema que contiene n moles de un gas ieal con un calor específico molar a presión constante Cp
experimenta dos procesos reversibles. Comienza con presión Pi y volumen Vi , se expande isotérmicamente
y luego se contrae adiabáticamente para alcanzar un estado final con presión Pi y volumen 3Vi . (a)
Encuentre su cambio de entropía en el proceso. (b) Como se relaciona la respuesta con el problema
22-77?
22-77 Una muestra que consiste de n moles de un gas ideal se somete a una expansión isobárica reversible
desde el volumen Vi hasta el volumen 3Vi . Encuentre el cambio de entropía.

Solución
(22-74) Sea el punto inicial (Vi , Pi ) y el final (3Vi , Pi ). Hay un punto intermedio k con (Vk , Pk ) descono-
cidos cuando termina de expandirse isotérmicamente y se empieza a contraer adiabáticamente.

Figura 9.1: [Serway] Problema 22-77, 22-77

Podemos encontrar el punto de intersección usando:


Por proceso isotérmico
Pi V i = Pk V k (9.1)
Por proceso adiabático
Pi (3Vi )γ = Pk (Vk )γ (9.2)
Dividiendo 9.2 entre 9.1
Pi (3Vi )γ Pi (Vi )γ
= ⇒ 3γ Viγ−1 = Vkγ−1 ⇒ Vk = 3γ/(γ−1) Vi
Pi V i Pi V i
Con Vk , ahora podemos averiguar el cambio de entropía.

∆S = ∆Si→k + ∆Sk→f = ∆Si→k

89
CAPÍTULO 9. LA SEGUNDA LEY DE LA TERMODINÁMICA

Donde ∆Sk→f = 0 ya que el proceso k → f es adiabático. Planteamos la primera Ley de forma diferencial
para el proceso isotérmico (dU = 0) y despejamos dS

P nR
dU = dW + dQ = −P dV + T dS = 0 ⇒ dS = dV = dV
T V
Integramos
ˆ Vk
!
3γ/(γ−1) Vi
 
nR Vk   γ
⇒ ∆S = dV = nR ln = nR ln = nR ln 3γ/(γ−1) = nR ln (3)
Vi V Vi Vi γ−1

Reescribamos un poco más claro para comparar con la siguiente parte


 
Cp
γ Cv Cp Cp
=  = =
γ−1 Cp Cp − Cv R
−1
Cv

Reescribimos el resultado  
Cp
∆S = nR ln(3) = nCp ln(3) (9.3)
R
(22-77) Aquí vamos de (Vi , Pi ) → (3Vi , Pi ), los mismos puntos i y f que el problema 22-74, solo que lo
hacemos a presión constante, el cálculo es más directo.
dQ nCp dT
dS = =
T T
ˆ f  
dT Tf
∆S = nCp = nCp ln
i T Ti
Encontramos la relación de las temperaturas conwho killed captain alex
Vi Vf Tf Vf 3Vi
= ⇒ = = =3
Ti Tf Ti Vi Vi

∆S = nCp ln(3) (9.4)


Vemos que el ∆S de la Ec. 9.3 y la Ec. 9.4 son el mismo. Ambos procesos inician en i y terminan en f .
Ambos procesor van a tener el mismo ∆S ya que la entropía es una función de estado y la diferencia va
a depender de los puntos i y f , no de la trayectoria que conecta a los puntos.

[Serway] Problema 22-76


Una muestra de n moles de un gas ideal monoatómico se lleva a través del ciclo que se muestra en la figura.
En el punto A la presión, el volumen y la temperatura son Pi , Vi y Ti , respectivamente. En términos de R
y Ti , encuentra (a) la energía total que entra al sistema por calor por ciclo, (b) la energía que entra sale
del sistema por calor por ciclo y (c) la eficiencia de una máquina que opera en este ciclo. (d) Explique
cómo se compara con la de una máquina que opera en un ciclo de Carnot entre los mismos extremos de
temperatura.
Solución

90
CAPÍTULO 9. LA SEGUNDA LEY DE LA TERMODINÁMICA

Figura 9.2: [Serway] Pregunta 22-76

Es un gas monoatómico ⇒ Cv = 3R/2 y Cp = 5R/2.


El trabajo del ciclo es el área que encierra el ciclo.

W = − (3Pi − Pi ) (2Vi − Vi ) = −2Pi Vi = −2nRTi

Es facil de probar que TA = Ti , TB = 3Ti , TC = 6Ti , TD = 2Ti . Usando esas temperaturas, calculemos el
calor de los procesos.
3
QAB = nCc (TB − TA ) = n R (3Ti − Ti ) = 3nRTi > 0
2
5 15
QBC = nCp (TC − TB ) = n R (6Ti − 3Ti ) = nRTi > 0
2 2
3
QCD = nCv (TD − TC ) = n R (2Ti − 6Ti ) = −6nRTi < 0
2
5 5
QDA = nCp (TA − TD ) = n R (Ti − 2Ti ) = − nRTi < 0
2 2
(a) La energía total que entra por calor QH es la suma de los Qij > 0. En este caso QAB y QBC .

15 21
QH = QAB + QBC = 3nRTi + nRTi = nRTi
2 2
(b) La energía que sale por calor Qc es la suma de los Qij < 0, que son QCD y QDA .

5 17
QC = QCD + QDA = −6nRTi − nRTi = − nRTi
2 2
Para corroborar que lo hicimos bien, obtengamos el calor neto por ciclo. Debería darnos igual al trabajo
por ciclo
21 17
Q = |QH | − |QC | = nRTi − nRTi = 2nRTi = W
2 2
Lo cual dá. (c) Lo podemos obtener de 2 maneras

|Wmaq | 2nRTi 4
e= =  = = 0.190
QH 21 21
nRTi
2

91
CAPÍTULO 9. LA SEGUNDA LEY DE LA TERMODINÁMICA

O con  
17
nRTi
|QC | 2 4
e=1− =1−   =
|QH | 21 21
nRTi
2
(d) Comparemos con una máquina de Carnot. La mayor temperatura que alcanza el ciclo es TH = 6Ti ,
la menor es TC = Ti . La eficiencia es
TC Ti 5
eCarnot = 1 − =1− = = 0.833
TH 6Ti 6
Sí nos da un resultado con sentido, la eficiencia de Carnot tiene que ser mayor que la eficiencia de la
máquina.

[Serway] Problema 22-73


II Parcial 2017 I Ciclo Problema 5
Una muestra de 1.00 mol de un gas ideal monoatómico se lleva a través del ciclo de la figura 9.3. El
proceso A → B es una expansión isotérmica reversible. Calcule (a) El trabajo neto realizado por el gas,
(b) la energía agregada al gas por calor, (c) la energía expulsada del gas por calor y (d) la eficiencia del
ciclo. (e) Explique cómo se compara la eficiencia con la de una máquina de Carnot que funciona entre los
mismos extremos de temperatura.

Figura 9.3: [Serway] Problema 22-73

Solución
Es un gás monoatómico ⇒ Cv = 3R/2, Cp = 5R/2. Calculemos todo aunque no nos lo estén preguntando.
Proceso A → B
El factor 1.013 × 105 (1 × 10−3 ) = (101.3) para hacer la conversión de atm·L→ Pa·m3 Es un proceso


isotérmico, la energía es constante ⇒ ∆UAB = 0.

∆UAB = WAB + QAB = 0 ⇒ QAB = WAB


ˆ ˆ Vf  
nRT Vf
W= − P dV = − − dV = −nRT ln
Vi V Vi

92
CAPÍTULO 9. LA SEGUNDA LEY DE LA TERMODINÁMICA

     
Vf Vf 50
WAB = −nRTA ln = PA VA ln = −(5)(10)(101.3) ln = −8151 J
Vi Vi 10
⇒ QAB = −WAB = 8151 J
Proceso B → C
ˆ ˆ
W =− P dV = −P dV = −P ∆V

WBC = −PB (VC − VB ) = −(1)(10 − 50)(101.3) = 4052 J


5 5 5
QBC = nCp ∆T = n R (TC − TB ) = PC (VC − VB ) = (1)(10 − 50)(101.3) = −10130 J
2 2 2
El ∆UBC lo podemos calcular de 2 maneras
3 3 3
∆UBC = nCv ∆T = n R (TC − TB ) = PC (VC − VB ) = (1)(10 − 50)(101.3) = −6078 J
2 2 2
O también
∆UBC = WBC + QBC = 4052 − 10130 = −6078 J
Proceso C → A Es a volumen constante ⇒ dV = 0

dW = −pdV = 0 ⇒ WCA = 0

Entonces ∆UCA = WCA + QCA = QCA


3 3 3
∆UCA = QCA = nCv ∆T = n R (TA − TC ) = VA (PA − VC ) = (10)(5 − 1)(101.3)
2 2 2
⇒ ∆UCA = QCA = 6078J

Cuadro 9.1: Valores de ∆U, Q y W en [J]


[Serway] Problema 22-73

∆U W Q
A→B 0 -8151 8151
B→C -6078 4052 -10130
C→A 6078 0 6078

El trabajo neto es
W = WAB + WBC + WCA = −8151 + 4052 + 0 = −4099 J
El trabajo que hace la máquina es Wmaq = 4099 J Para corroborar sumemos todos los calores

Q = QAB + QBC + QCA = 8151 − 10130 + 6078 = 4099 J

Da lo mismo que el trabajo de la máquina. (b) La energía que entra por calor QH es la suma de los Qij
positivos
QH = QAB + QCA = 8151 + 6078 = 14229 J
(c) La energía expulsada por calor QC es la suma de los Qij negativos

QC = QBC = −10130 J

93
CAPÍTULO 9. LA SEGUNDA LEY DE LA TERMODINÁMICA

(d) La podemos calcular de 2 maneras

Wmaq 4099
e= = = 0.288
QH 14229

|QC | 10130
e=1− =1−
|QH | 14229
(e) La mayor temperatura es 3s en el proceso isotérmico A → B y la menor es en el punto C
 
Pc V c
TC nR P 1
eCarnot = 1 − =1−   =1− C =1−
TH Pc V c PA 5
nR

4
⇒ eCarnot = = 0.80
5
La eficiencia de la máquina es menor que la eficiencia de Carnot.
[Serway] Pregunta 22-81
II Parcial 2017 I Ciclo Problema 3
Una muestra de 1.00 mol de gas ideal diatómico se lleva a través del ciclo de Carnot como se muestra en
la figura 9.4. La presión en el punto A es de 25.0 atm y la temperatura es de 600 K. En el punto C la
presión es 1.00 atm y la temperatura es de 400 K. (a) Determine las presiones y volúmenes en los puntos
A, B, C y D. (b) Calcule el trabajo neto realizado por el ciclo. (c) La eficiencia del ciclo.

Figura 9.4: [Serway] Problema 22-81

Solución
Los valores en negro son los conocidos. Los valores rojos son los que se van encontrando.
Recordatorio: La relación adiabática que se recuerda usualmente es

Pi Viγ = Pf Vfγ

Esta relaciona a P y V . Dependiendo del problema (como este), pueda que esta relación no sea muy util
y y se ocupe relacionar otras variables. Se pueden obtener otras relaciones adiabáticas reemplazando con

94
CAPÍTULO 9. LA SEGUNDA LEY DE LA TERMODINÁMICA

Cuadro 9.2: Valores de P V T .


[Serway] Problema 22-81

P (atm) V (m3 ) T (K)


A 25.0 1.97 600
B 4.14 11.9 600
C 1.00 32.8 400
D 6.04 5.43 400

el gas ideal la variable que se quiere eliminar.


Eliminemos P y relacionemos V y T
 
γ nRT
P V = Const = V γ ⇒ T V γ−1 = Const
V

⇒ Ti Viγ−1 = Tf Vfγ−1
La otra (que vamos a usar) es relacionar P y T . Eliminemos V

nRT γ
 
γ
PV = P = Const ⇒ P 1−γ T γ = Const ⇒ P T γ/(γ−1) = Const
P
γ/(γ−1) γ/(γ−1)
Pi Ti = Pf Tf (9.5)
Volviendo al problema.
Es un gas diatómico ⇒ Cv = 5R/2, Cp = 7R/2, γ = 7/5 = 1.4.
(a) De A → B es isotérmico, entonces TA = TB = 600 K, igual de C → D, entonces TC = TD = 600 K.
De B → C es adiabático, usamos la Ec. 9.5
 γ/(γ−1)
γ/(γ−1) γ/(γ−1) TC
PB TB = PC TC ⇒ PB = PC
TB
 1.4/(1−1.4)
400
PB = (1.00) = 4.13 atm
600
Hacemos lo mismo de D → A, que es un proceso adiabático
 γ/(γ−1)
γ/(γ−1) γ/(γ−1) TA
PA TA = PD TD ⇒ PD = PA
TD
 1.4/(1−1.4)
600
PD = (25.0) = 6.05 atm
400
Ya con todos los valores de P y T , averiguamos todos los volumenes.
nRTk
Pk Vk = nRTk ⇒ Vk =
Pk
(1.00)(0.08205)(600)
VA = = 1.97 L
25
(1.00)(0.08205)(600)
VB = = 11.9 L
4.14

95
CAPÍTULO 9. LA SEGUNDA LEY DE LA TERMODINÁMICA

(1.00)(0.08205)(400)
VC = = 32.8 L
1.00
(1.00)(0.08205)(400)
VD = = 5.43 L
6.04
(b) Hagamolo de 2 maneras.
Método 1 Este es más rápido. Aprovechemos que es una máquina de Carnot. La eficiencia es
TC
e=1−
TH
A la vez, la eficiencia es por definición
Wmaq
e=
QH
Igualamos  
TC Wmaq TC
1− = ⇒ Wmaq = Qh 1 −
TH QH TH
Ocupamos averiguar QH . En la máquina de Carnot, la única entrada de calor es de A → B, que es
isotérmico.  
VB
QH = −WAB = nRTH ln
VA
     
VB TC 11.9 400
⇒ Wmaq = nRTH ln 1− = (1.00)(8.314)(600) ln 1−
VA TH 1.97 300
⇒ Wmaq = 2.99 kJ (9.6)
Método 2 Encontremos Wmaq sumando todos los trabajos. El trabajo en isotérmicos es
 
Vf
W = −nRT ln
Vi
Y para adiabáticos es
Pf Vf − Pi Vi
W =
γ−1
Para adiabáticos le ponemos 1.013 × 105 (1 × 10−3 ) = (101.3) para hacer la conversión de atm·L→


Pa·m3    
VB 11.9
WAB = −nRTH ln = (1.00)(8.314)(600) ln = −8971 J
VA 1.97
PC VC − PB VB (1)(32.8) − (4.14)(11.9)
WBC = = (101.3) = −4170 J
γ−1 1.4 − 1
   
VD 5.43
WCD = −nRTC ln = (1.00)(8.314)(400) ln = 5981 J
VC 32.8
PA V A − PD V D (25)(1.97) − (6.04)(5.43)
WBC = = (101.3) = 4166 J
γ−1 1.4 − 1
W = WAB + WBC + WCD + WDA = −8971 − 4170 + 5981 + 4166 = −2.99 kJ
Por la convención ∆U = W + Q, este es el trabajo hecho sobre el gas, el trabajo que hace el gas sobre el
entorno (El trabajo de la máquina es el negativo) es

Wmaq = −W = 2.99 kJ

96
CAPÍTULO 9. LA SEGUNDA LEY DE LA TERMODINÁMICA

Se obtiene el mismo resultado que la Ec. 9.6


(c) Igual se puede de 2 maneras. La primera que ya usamos en la parte (b) que sabemos que es un ciclo
de Carnot, entonces
TC 400 1
e=1− =1− = = 0.333
TH 600 3
La segunda es calcular por definición con el trabajo neto de la máquina y el calor de entrada QH = −WAB
2990
e= = 0.333
8971
Da lo mismo.2
Comentario: Durante la evaluación de este problema en el examen, habían algunos preocupados que la
presión del punto B era menor que la del punto D,por que según la Fig 9.4, “el punto B está más arriba
que el D. (Que PB > PD ) ”
Estas figuras no son hechas a escala, se usan para representar algo. En el diagrama de Carnot, lo importate
es los procesos son isotérmico → adiabático → isotérmico → adiabático. El punto B sí puede estar más
abajo.
El gráfico real de este ciclo de Carnot es como se muestra en la Fig 9.5.

Figura 9.5: Ciclo con la escala correcta.


[Serway] Problema 22-81

No se tomen las figuras muy literal.

[Serway] Problema 22-70


Un laboratorio de biología se mantiene a una temperatura constante de 7.00 ◦ C mediante un acondicio-
nador de aire, que da salida al aire exterior. En un típico día caluroso de verano, la temperatura exterior
es de 27 ◦ C y la unidad de acondicionamiento de aire emite energía al exterior en una cantidad de 10.0
kW. Modele la unidad como teniendo un coeficiente de rendimiento (COP) igual a 40 % del COP de un
dispositivo ideal de Carnot. (a) Con qué rapidez el acondicionador de aire retira energía del laboratorio?
2
Esta dividisón no es el.0.3 periódico, esta división es 0.333296..., pero es por arrastre de errores numéricos.

97
CAPÍTULO 9. LA SEGUNDA LEY DE LA TERMODINÁMICA

(b) Calcule la potencia requerida para la entrada de trabajo. (c) Encuentre el cambio de entropía del
universo que produce el acondicionador de aire en 1.00 h.
Solución
En este problema nos dan potencia. Si agarramos la primera ley (en refrigeradores y agarrando todo
positivo y evitarnos signos implicitos), tenemos

W = QH − QC

Si el ciclo tarda un tiempo ∆t, dividamos entre ∆t


W QH Q
= − ⇒ PW = PH − PC
∆t ∆t ∆t
Hagamos lo mismo para el COPR
 
QC
QC ∆t PC
COPR = =   =
W W PW
∆t
Las relaciones de eficiencia del ciclo se ven iguales si cambiamos la energía por su potencia respectiva. Las
temperaturas son TH = 27 + 273 = 300 y TC = 7 + 273 = 280 K.
(a) Emitir energía al exterior es el PH que sale al deposito térmico caliente. PH = 10.0 kW. La definición
de la eficiencia en un refrigerador es (Usando valores positivos para evitar pensar en los signos implicitos)
PC PC 1
COPR = = =  (9.7)
W PH − PC PH
−1
PC
Para un refrigerador de Carnot, cumple que PH /PC = TH /TC , entonces el COP de Carnot es
1 1
COPR,Carnot =   = 300
 = 14
TH −1
−1 280
TC
El refrigerador tiene el 40 % del COP de Carnot

⇒ COPR = 0.4 (COPR,Carnot ) = 0.4(14) = 5.6

De una de las expresiones de COPR de la Ec 9.7, despejamos PC


1 PH 10
COPR =   ⇒ PC =   = 
PH 1 1
−1 +1 +1
PC COPR 5.6
PC = 8.48 kW
(b) Usamos la primera ley por ciclo con potencias, tenemos

PW = PH − PC = 10 − 8.48 = 1.52 kW

(c) La entropía es
1
dS = dQ
T

98
CAPÍTULO 9. LA SEGUNDA LEY DE LA TERMODINÁMICA

Dividimos entre dt
dS 1 dQ P
= =
dt T dt T
La generación de entropía por unidad de tiempo es P/T . En este problema, las potencias por trasnferencia
de calor son constantes, entonces ˆ t
P 0 Pt
∆S = t =
0 T T
Tenemos 2 flujos de potencia por calor, entonces
PH t PC t
∆ST otal = ∆SH + ∆SC = −
TH TC
Donde el deposito térmico frío C va negativo por que el calor sale de él. Usando t = hora = 3600 s
!
10 × 103 8.48 × 103

J
∆ST otal = − (3600) = 1.09 × 104
(300) (280) K

Enlaces de interés
Se puede hacer correr una pequeña máquina térmica con agua caliente.
Sixty Simbols: The coffee-powered engine - Sixty Symbols

Una explicación del pajaro que toma agua. Una máquina térmica que parece funcionar con depositos
térmicos a la misma temperatura (Que debería ser imposible de ser así).
engineerguy: The Engineering of the Drinking Bird

Una explicación de como sirven los motores de combustión interna. En lugar de usar una animación,
hicieron un motor con una tapa transparente para ver el ciclo.
SmarterEveryDay: How Engines Work - (See Through Engine in Slow Motion) - Smarter Every Day 166

99
10 | Campo Eléctrico

[Sears] Problema 21-82


III Parcial I Ciclo 2017 Problema 1
Dos esferas diminutas de masa igual a 6.80 mg tienen cargas de igual magnitud, 72.0 nC, pero de signo
contrario. Se atan al mismo gancho del techo con cuerdas ligeras de longitud igual a 0.530 m como se
muestra en la Fig. 10.1. Cuando se activa un campo eléctrico horizontal y uniforme E, digirido a la
izquierda, las esferas cuelgan con un ángulo θ entre las cuerdas igual a 50◦ . (a) Cual esfera (la de la
derecha o la de la izquierda) tiene carga positiva? (b) Cual es la magnitud E del campo?

Figura 10.1: [Sears] Problema 21-82

Sea α = θ/2 = 25◦


(a) Para saber el signo de las cargas, dibujemos los 2 casos y veamos a ver si pueden ser estables.
El caso donde la carga derecha sea positiva, en la Fig 10.2 se ve donde todas las fuerzas (excepto la
tensión) hacen torque en la misma dirreccón para cualquiera de las 2 cargas. No hay manera de que la
suma de torques en este caso pueda dar 0, esto implica una aceleración y no está en equilibrio. En el caso
que la carga izquierda sea positiva y derecha negativa, sí es posible que pueda estár en equilibrio por que
hay posibilidad que los torques puedan sumar 0.
La carga de la izquierda es positiva y la de la derecha es negativa.
(b) Hagamos sumatoria de fuerzas sobre la carga negativa (De la derecha).
Ya que no nos están preguntando la tensión, para evitar considerarla, hagamos sumatoria de fuerzas sobre
el eje n̂ que se muestra en la figura 10.3. Para hacer esto, ocupamos sacar la proyección de cada fuerza
sobre este eje. la dirección n̂ la estoy escojiendo que sea perperdicular a T. La proyección de T sobre el
eje n̂ es 0. ( T · n̂ = 0 )

100
CAPÍTULO 10. CAMPO ELÉCTRICO

Figura 10.2: Ambos casos de los signos.


[Sears] Problema 21.82

Figura 10.3: Diagrama de fuerzas.


[Sears] Problema 21.82

Entonces, la sumatoria de fuerza sobre n̂ es

X 1 q2
Fn,i = qE cos (α) − mg sin(α) − cos(α) = 0
4π0 (2L sin(α))2
i
Despejando E
1 q mg
⇒E= 2 + tan(α)
4π0 (2L sin(α)) q
Evaluando
(72 × 10−9 ) (6.8 × 10−6 )(9.8)
E = (9 × 109 ) + tan(25◦ )
[(2)(0.530)(sin(25◦ ))]2 72 × 10−9
N
⇒ E = 3.66 × 103
C
[Sears] Problema 21-106
Dos cagas se colocan como se muestra en la Fig. 10.4. La magnitud de q1 es 3.00 µC, pero se desconocen
el signo y el valor de la carga q2 . La dirección del campo eléctrico neto E en el punto P está completo en

101
CAPÍTULO 10. CAMPO ELÉCTRICO

la dirección negativa del eje y. (a) Considerando los posibles signos diferentes de q1 y de q2 , hay cuatro
posibles diagramas que podrían representar los campos eléctricos de E1 y E2 producidos por q1 y q2 .
Dibuje las cuatro posibles configuraciones de campo eléctrico. (b) Con base en los diagramas del inciso
(a) y la dirección de E, deduzca los signos de q1 y q2 . (c) Determine la magnitud de E.

Figura 10.4: [Sears] Problema 21-106

Solución
(a),(b) Para saber el signo, veamos como suman los vectores. Si tengo dos vector v1 y v2 y sumo los
vectores, el vector v1 + v2 va a estar en la región de color de la Fig. 10.5

Figura 10.5: Región de v1 + v2 .


[Sears] Problema 21-106

Hagamos las 4 posibilidades y veamos cual de esas tiene el vector de campo eléctrico E en esa región
Vemos que las 2 cargas tienen que ser negativas para que el campo eléctrico esté en la región de abajo.
(c) Calculemos la magnitud de las cargas y magnitud de E. Sabemos que E va hacia abajo, entonces
E tiene la forma E = Ey ĵ, la componente Ex es Ex = 0. Antes de hacer el cálculo, en ningún lado nos
están preguntando cuanto valen los ángulos internos. Como regla de dedo, no calcule cosas innecesarias,
podemos usar trigonometria para saber cuanto vale el coseno y el seno de los ángulos necesarios usando
las definiciones de coseno y seno como se muestra en la Fig. 10.7.

Sea a = 5, b = 12 y c = 13 cm. Sumemos el campo eléctrico producido por ambas cargas. Voy a tomar
las cargas como “positivas” usando nada mas q, en lugar de ponerles − |q| y voy a evaluar con el signo al
final, para comprobar que también sirve trabajar así.
El campo eléctrico es un vector, se suma como vector, por componentes. El campo eléctrico total es

E = E1 + E2
1 q1   1 q2  
⇒E= cos(θ)î + sin(θ)ĵ + − cos(ϕ) î + sin(ϕ)ĵ
4π0 a2 4π0 b2

102
CAPÍTULO 10. CAMPO ELÉCTRICO

Figura 10.6: Posibildades de las cargas.


[Sears] Problema 21-106

Figura 10.7: Trigonometría del sistema.


[Sears] Problema 21-106

Aquí cambiamos con las definiciones de coseno y seno que se ven en la Fig. 10.7, sin necesidad de averiguar
el ángulo y arrastrar errores numéricos
   
1 q1 a b 1 q2 b a
E= î + ĵ + − î + ĵ
4π0 a2 c c 4π0 b2 c c
Juntamos las componentes
 
 
1  q1 q2  q1 b q2 a 
E=  −
 ac bc î + + ĵ (10.1)
4π0 | {z } a2 c b2 c 
=0
La componente Ex = 0, entonces el término de î tiene que serlo. Despejando q2
q1 q2 b
− = 0 ⇒ q2 = q1
ac bc a
Evaluamos con el signo de q1 = −3.00 µC
12
q2 = (−3.00) = −7.2 µC
5

103
CAPÍTULO 10. CAMPO ELÉCTRICO

Trabajando todo general con los q “positivos” y evaluar con el signo negativo que tiene la carga, da que
la carga q2 es negativa, que es consistente con lo que obtuvimos haciendo las 4 combinaciones de la Fig.
10.6. Con el valor de las 2 cargas, calculamos el E de la Ec. 10.1
   
1 q1 b q2 a 9 (−3.00)(0.12) (−7.2)(0.05)
E= + ĵ = (9 × 10 ) + × 106 ĵ
4π0 a2 c b2 c (0.05)2 (0.13) (0.12)2 (0.13)
N N
E ⇒= −1.17 × 107 ĵ ⇒ |E| = 1.17 × 107
C C
Ampliación 2017 II Ciclo Problema 5
Se tiene un cable cargado con densidad lineal de carga λ0 como se muestra en la Fig. 10.8. Calcule la
aceleración que experimentaría un carga puntual de carga q y masa m si se coloca en el centro de los semi
círculos.

Figura 10.8: III Parcial 2016 I Ciclo Problema 2

Solución
Antes de hacer el cálculo para las 4 diferentes secciones (Los 2 cables rectos y los 2 semiciírculos), veamos
si podemos cancelar algo. La componente Ex va a ser Ex = 0 ya que el sistema es simétrico si le doy
vuelta en el eje y. Y podemos ver que el campo de las secciones horizontales se van a cancelar entre ellas.
Entonces solo ocupamos calcular el campo eléctrico de las secciones semi circulares. Y en realidad solo
tenemos que hacer un cálculo. Lo calculamos para el de radio R y el círculo más grande es el mismo
resultado, solo que con R → 2R.

Figura 10.9: Proyección para dEy .


III Parcial 2016 I Ciclo Problema 2

El diferencial de campo eléctrico dEy es


1 dq 1 λ0 ds 1 λ0 Rdθ
dEy = − sin(θ)dE = − sin(θ) 2
= − sin(θ) 2
= − sin(θ)
4π0 R 4π0 R 4π0 R2

104
CAPÍTULO 10. CAMPO ELÉCTRICO

1 λ0
⇒ dEy = − sin(θ)dθ
4π0 R
Donde el menos está por que la dirección va en −y. De esta expresión ya podemos integrar.
ˆ ˆ
1 λ0 π 2 λ0
Ey = dEy = − sin(θ)dθ = −
4π0 R 0 4π0 R
Para el círculo de radio 2R, el campo eléctrico es
2 λ0 1 λ0
Ey,2R = − =−
4π0 2R 4π0 R
El campo eléctrico total en el centro es
 
2 λ0 1 λ0 3 λ0
E= − − ĵ = − ĵ
4π0 R 4π0 R 4π0 R
La aceleración la encontramos usando la fuerza sobre la partícula. La fuerza es F = qE, entonces
q
F = ma = qE ⇒ a = E
m
3 q λ0
⇒a=− ĵ
4π0 m R

III Parcial 2016 I Ciclo Problema 2


Una barra de longitud L, masa M y densidad lineal uniforme de carga λ1 se encuentra cerca de una línea
infinita con densidad lineal uniforme de carga λ2 como se muestra en la Fig 10.10. Si la barra se encuentra
en reposo, calcule la distancia de separación d entre la línea y la barra.
Extra: Obtenga el límite cuando L → 0. En el límite mantenga la carga Q1 = λ1 L constante. En el límite
la densidad de carga λ1 → ∞ a medida que L → 0, por lo que ocupamos expresarlo en términos de Q1
constante. Debería obtener una expresión igual a que si hubiera hecho el problema con una carga puntual
en lugar de una barra.

Figura 10.10: III Parcial 2016 I Ciclo Problema 2

Obtengamos el E2 de la barra 2 usando la Ley de Gauss1 , solo el área lateral tiene un flujo diferente de
0, y E es constante y paralelo a la superficie de integración
˛
qin
E2 · dA =
0
1
Estoy usando algo del siguiente capítulo, pero esto era en un examen, obtenerlo por integración es más lento.

105
CAPÍTULO 10. CAMPO ELÉCTRICO

Figura 10.11: Flujo eléctrico en la superficie .


III Parcial 2016 I Ciclo Problema 2

ˆ
qin λ2 h λ2 1
⇒ E2 dA = E2 A = E2 (2πrh) = = ⇒ E2 =
0 0 2π0 r

Para obtener la separación, ocupamos obtener la fuerza que hace el cable 2 sobre la barra 1. No podemos
decir nada más que F = q1 E2 , por que el campo E2 no es constante en la barra 1. Podemos averiguarlo
planteando la fuerza diferenciales que hace la línea 2 sobre la un diferencial de la barra 1.
La fuerza diferencial es (Haciendolo escalar que todo va en una dirección, hacia arriba)
λ1 λ2 dr
dF = E2 dq1 = E2 λ1 dr =
2π0 r
Y de aquí integramos
ˆ d+L  
λ1 λ2 dr λ1 λ2 d+L
F = = ln
d 2π0 r 2π0 d
Donde los límites de integración son donde está distribuida la carga, que sería iniciando desde r = d
hasrta r = d + L.
Con la fuerza eléctrica, ya podemos plantear suma de fuerzas. El sistema está en equilibrio, entonces
 
X λ1 λ2 d+L
Fi = ln − mg = 0
2π0 d
i

De aquí podemos despejar d, que da


L
d=   (10.2)
mg2π0
exp −1
λ1 λ2
Extra: Saquemos el límite. Primero eliminemos λ1 para no tener 2 límites a la vez.
Q1
Q1 = λ1 L ⇒ λ1 =
L
y sustituyendo en 10.2, tenemos
L
d=  
mg2π0 L
exp −1
Q1 λ2

106
CAPÍTULO 10. CAMPO ELÉCTRICO

Expresado de esta manera, no nos tenemos que preocupar por que λ1 tiende a ∞, ya solo tenemos un
límite que evaluar. Sacamos el límite L → 0
L L’H 1
lı́m d = lı́m   = lı́m    
L→0 L→0 mg2π0 L L→0 mg2π0 mg2π0 L
exp −1 exp
Q1 λ2 Q1 λ2 Q1 λ2

Q1 λ 2
⇒d=
mg2π0
L’H
Donde = significa que usé la regla de L’Hôpital.
Reacomodamos para que se vea obvio que es como si lo estuvieramos aplicando a una carga puntual
 
λ2 1
mg = Q1 (10.3)
2π0 d

El parentesis del R.H.S de la Ec. 10.3 es el campo eléctrico del cable infinito, siendo aplicado a una carga
puntual, que es igual a la fuerza de gravedad a una distancia d.
Sí obtuvimos el resultado esperado en el límite.

Parecido al [Serway] Problema 23-71


III Parcial II Ciclo 2015 Problema 2
Se dobla una varilla que tiene poco espesor para formar el arco de un círculo cuyo radio es lo muestra
la Fig. 10.12, la varilla tiene una carga de acuerdo con la función λ(θ) = λ0 cos(θ), donde λ0 es una
constante. El arco contiene un ángulo total de 2θ0 , que es simétrico respecto al eje x.
(a) Determine el campo eléctrico en el origen O.
(b) Calcule la carga total distribuida sobre la longitud del arco.
Sugerencia: Recuerde la identidad cos2 (θ) = (1 + cos(2θ)) /2
Extra: Usando los resultados de (a) y (b), obtenga el E en el límite de θ0 → 0, pero manteniendo la
carga de la varilla constante.

Figura 10.12: III Parcial 2015 II Ciclo Problema 2

Solución
La distribución de la carga es simétrica si se el sistema en l eje x. El campo Ey va a ser Ey = 0. Solo va
a tener una componente Ex . Igual, calculemolo para comprobar.

107
CAPÍTULO 10. CAMPO ELÉCTRICO

Figura 10.13: Proyección del dE en x.


III Parcial 2015 II Ciclo Problema 2

Proyectando dE sobre el eje x, viendo la Fig. 10.13, el dEx es


1 dq 1 λ(θ)ds 1 λ0 cos(θ)Rdθ
dEx = − cos(θ)dE = − cos(θ) 2
= − cos(θ) 2
= − cos(θ)
4π0 R 4π0 R 4π0 R2
1 λ0
dEx = − cos2 (θ) dθ
4π0 R
Integrando
ˆ θ0 ˆ
1 λ0 2 1 λ0 θ0 1 + cos(2θ)
⇒ Ex = − cos (θ) dθ = − dθ
4π0 R −θ0 4π0 R −θ0 2
 
1 λ0 sin(2θ0 )
⇒ Ex = − θ0 + (10.4)
4π0 R 2
Para verificar que Ey da 0, la proyección de dE en y es

1 dq 1 λ(θ)ds 1 λ0 cos(θ)Rdθ
dEy = − sin(θ)dE = − cos(θ) 2
= − sin(θ) 2
= − sin(θ)
4π0 R 4π0 R 4π0 R2
1 λ0
dEy = − sin(θ) cos(θ) dθ
4π0 R
Integrando
ˆ θ0
1 λ0
Ey = − sin(θ) cos(θ) dθ = 0
4π0 R −θ0
Ya que la integral da 0.
(b) La carga total la encontramos integrando la densidad de carga

dQ = λ(θ)ds = λ0 cos(θ)Rdθ
ˆ θ0
Q= λ0 R cos(θ)dθ = 2Rλ0 sin(θ0 ) (10.5)
−θ0

Extra: Saquemos el límite θ0 con Q constante. Si reducimos la barra a un punto, deberíamos recuperar el
campo eléctrico de una carga puntual. Para mantener la carga constante, la densidad de carga λ0 → ∞.
Para evitar tratar 2 limites a la vez, despejemos λ0 en función de Q de la Ec. 10.5
Q
⇒ λ0 =
2R sin(θ0 )

108
CAPÍTULO 10. CAMPO ELÉCTRICO

Y sustituimos en la Ec. 10.4


   
1 Q 1 sin(2θ0 ) 1 Q 1 θ0 sin(2θ0 )
Ex = − θ0 + =− 1+
4π0 R2 2 sin(θ0 ) 2 4π0 R2 2 sin(θ0 ) 2θ0

Sacamos el límite θ0 → 0
 
1 Q 1 θ0 sin(2θ0 ) 1 Q 1
lı́m Ex = lı́m − 1+ =− [1 + 1]
θ0 →0 θ0 →0 4π0 R2 2 sin(θ0 ) 2θ0 4π0 R2 2

1 Q
Ex = −
4π0 R2
Donde usamos sin(θ)/θ → 1 en θ → 0.
Recuperamos el límite de carga puntual.

[Sears] Problema 21.107


Dos varillas deslgadas de longitud L están a lo largo del eje x, una entre x = a/2 y x = a/2 + L, y la otra
entre x = −a/2 y x = −a/2 − L. Cada varilla tiene carga positiva Q distribuida de manera uniforme en
toda su longitud. a) Calcule el campo eléctrico producido por la segunda varilla en puntos a lo largo del
eje x positivo. b) Demuestre que la magnitud de la fuerza que ejerce una varilla sobre la otra es

Q2 (a + L)2
 
F = ln
4π0 L2 a (a + 2L)

c) Demuestre que si a  L, la magnitud de esta fuerza se reduce a F = Q2 /4π0 a2 . Sugerencia: Use la


expansión ln(1 + z) = z − z 2 /2 + z 3 /3 − · · · , válida para |z|  1. Considere todas las expansiones al menos
hasta el orden L2 /a2 . Interprete este resultado.

Solución

Figura 10.14: [Sears] Problema 21.107

No podemos nada más decir que F = q 2 /(4π0 r2 ) por que no son cargas puntuales/esferas. (a) Calculemos
primero el E2 de la segunda barra en función de x positivo.. Es una barra uniforme, así que cumple
λ = Q/L.

109
CAPÍTULO 10. CAMPO ELÉCTRICO

El E2 apunta hacia x+ en la región x > 0, entonces podemos despreocuparnos de que cambie de signo.
Entonces la magnitud E2 en la posición está dada por
ˆ
1 dq2
E2 (x) =
4π0 r2
El r lo podemos averiguar haciendo una resta vectorial. En general, la distancia entre los puntos r2 y r1
es |r2 − r1 |. Como estamos en una dimensión, hagamos r2 → x y r1 → x0 , donde x0 va a apuntar a un
diferencial de carga dq en la barra 2. La distancia r es
2
r = x − x0 ⇒ r2 = x − x0

Uno podría pensar que lo puedo poner como r2 = (x + x0 )2 , como se ve en la Fig. 10.14 ya que x0 está en
la región de x < 0, más que el dibujo hace parecer que deberíamos hacer una suma y no una resta para
obtener la distancia, pero mejor colocarlo así y dejarle el signo implicito de x0 a los límites de la integral,
ahí es donde se especifica donde recorre x0 , sea si está en el positivo o negativo.2
Entonces ˆ −a/2 ˆ
1 λ dx0 1 Q −a/2 dx0
E2 (x) = =
4π0 −a/2−L (x − x0 )2 4π0 L −a/2−L (x − x0 )2
Donde recuerden, los límites de integración indican donde va la carga que se está integrando está ubicada,
la posición x es irrelevante, tienen que ver donde recorre el x0 que es la variable que apunta hacia los dq
que se están integrando.
Realizando la integral, nos da
 
1 Q 1 1
E2 (x) = a − a

4π L x + x+ +L
2 2
(b) Conociendo el E2 (x), ahora planteamos cuanto es el diferencial de fuerza dF que hace la barra 2 sobre
un diferencial de carga dq1 , que es
Q
dF = E2 (x) dq1 = E2 (x) λdx = E2 (x) dx
L
De aquí integramos y nos fijamos de donde a donde está la carga dq1 distribuida para poner los límites
de la integral
ˆ a/2+L ˆ a/2+L
 
Q 1 Q 2
F = E2 (x) dx =  1 − 1  dx
L 4π L2 a a
a/2 0 a/2 x+ x+ +L
2 2
1 Q2 h  a  a i a/2+L
= ln x + − ln x + + L

4π0 L 2 2 2

a/2

1 Q2
= [ln(a + L) − ln(a + 2L) − ln(a) + ln(a + L)]
4π0 L2
Usando propiedades de logaritmos, se puede reducir a
1 Q2 (a + L)2
 
F = ln
4π0 L2 a(a + 2L)
2
Así uno evita pensar como: “x0 es negativo, y si pongo este otro negativo, bla bla... ” y se hace un enredo pensando en
el signo implícito que tienen las variables, mejor dejar que la integral se encargue de eso.

110
CAPÍTULO 10. CAMPO ELÉCTRICO

Esa es la fuerza entre las barras, que es lo que pide el problema.


(c) Empezemos a aproximar términos, si a  L ⇒ L/a  1. Es un cambio un poco trivial, pero el
término unido de a/L va a ser el término con el que vamos a expandir.
Aproximemos el argumento del ln. Usemos
1
≈ 1 − x + x2 + O(x3 )
1+x
Entonces
L 2
 
1+
(a + L)2 L 2 L2
    3 
a L L
=  ≈ 1+ 1−2 +4 2 +O
a(a + 2L) L a a a a3
1+2
a
Ahora expandimos el (1 + L/a)2 , distribuimos y mantenemos términos hasta segundo orden

L L2 L2
 3 
L2
   3
L L L
= 1+2 + 2 1−2 +4 2 +O =1+ 2 +O
a a a a a3 a a3

1 Q2 (a + L)2 1 Q2 L2
    3 
L
⇒ 2
ln ≈ 2
ln 1 + 2 + O
4π0 L a(a + 2L) 4π0 L a a3
Utilicemos la expansión ln(1 + z) = z − z 2 /2 + z 3 /3 − · · · y quitemos el O

1 Q2 L2 1 Q2 L2 1 Q2
   
⇒F ≈ ln 1 + ≈ =
4π0 L2 a2 4π0 L2 a2 4π0 a2

Este es el límite cuando las barras están tan alejadas que la forma en como está distribuida las cargas
deja de importar y se ven como cargas puntuales.

Problema de desafío
[Purcell] Problema 1.39
Cuatro cuerpos con cargas positivas, dos con cargas Q y dos con cargas q, son conectadas por cuerdas
no extensibles de igual longitud. En ausencia de fuerzas externas, asumen la configuración de equilibrio
que se muestra en la Fig. 10.15. Muestre que tan3 θ = q 2 /Q2 . (Capítulo de Campo Eléctrico) Usted
puede mostrar que esta relación se debe mantener si la fuerza total en cada cuerpo, the suma vectorial
de las tensiones de la cuerdas y la repulsión eléctrica, es cero. (Capítulo de Potencial) O usted podría
escribir la expresión de la energía potencial del ensamble y minimizarla.

Solución
Veamos que todas las tensiones de las diferentes cuerdas son iguales.
De la Fig. 10.16, si giramos el sistema alrededor del eje x, el sistema se sigue viendo igual. Entonces tiene
que cumplir que T1 = T3 y T2 = T4 . Además, si le damos vuelta alrededor del eje y, el sistema aún se
sigue viendo igual, entonces tiene que cumplir T1 = T2 y T3 = T4 . Con esto vemos que todas las tensiones
tiene que ser iguales, le voy a poner T nada más para la magnitud.
Voy a usar la notación Fij , que es la fuerza en la carga j provocada por la carga i, y para el vector de
tensión en una cuerda específica, va a ser Tij en dirección de i hacia j. Usemos que las cuerdas tienen

111
CAPÍTULO 10. CAMPO ELÉCTRICO

Figura 10.15: [Purcell] Problema 1.39

Figura 10.16: Tensiones.


[Purcell] Problema 1.39

longitud L. Voy a plantearlo, pero tengo que quitarlo en algún momento. Pongamole un número a cada
carga como se ven en las figuras 10.16 y 10.17
En la figura 10.17, la distancia entre los centros de las cargas q y q es 2L sin(θ) y la distancia entre Q y
Q es 2L cos(θ).

Figura 10.17: Longitudes entre cargas.


[Purcell] Problema 1.39

Hagamos sumatoria de fuerzas sobre la carga Q de la izquierda. Voy a poner el 1/4π0 afuera de todas

112
CAPÍTULO 10. CAMPO ELÉCTRICO

las fuerzas eléctricas por espacio. También voy a usar la notación cos(θ) = cθ y sin(θ) = sθ

Figura 10.18: Diagrama de fuerzas en Q.


[Purcell] Problema 1.39

X
Fi = F21 + F31 + F41 + T14 + T12 = 0
i

Q2
 
1 qQ   qQ     
−cθ î + sθ ĵ − ĵ + −cθ î − sθ ĵ + T cθ î − sθ ĵ + T cθ î + sθ ĵ =0
4π0 L2 4L2 c2 θ L2
Cancelando términos
Q2
 
1 2qQ
− + î + 2T cθî = 0
4π0 L2 4L2 c2 θ
Ya nos queda una sola componente, moviendo términos y dividiendo entre cos(θ)

Q2
 
1 2qQ
+ = 2T (10.6)
4π0 L2 4L2 cos3 (θ)

Hagamos lo mismo para la carga q de abajo, la 2 La sumatoria de fuerzas es

Figura 10.19: Diagrama de fuerzas en q.


[Purcell] Problema 1.39

X
Fi = F12 + F32 + F42 + T21 + T24 = 0
i

q2
 
1 qQ   qQ      
cθ î − sθ ĵ + −cθ î − sθ ĵ − ĵ + T −cθ î + sθ ĵ + T cθ î + sθ ĵ =0
4π0 L2 L2 4L2 s2 θ
Cancelando términos
q2
 
1 2qQsθ
− + ĵ + 2T sθĵ = 0
4π0 4L2 s2 θ L2

113
CAPÍTULO 10. CAMPO ELÉCTRICO

Dividiendo entre sin(θ) y moviendo términos

q2
 
1 2qQ
+ 2 = 2T (10.7)
4π0 4L2 sin3 (θ) L

Del R.H.S de las Ec. 10.6 y10.7 son iguales, entonces igualamos el L.H.S

Q2 q2
   
1 2qQ 1 2qQ
+ = + 2
4π0 L2 4L2 cos3 (θ) 4π0 4L2 sin3 (θ) L

Eliminando el 1/4π0 y 1/L2 que están a factor común

Q2 q2
2qQ + = 2qQ +
4 cos3 (θ) 4 sin3 (θ)

q2
⇒ tan3 (θ) =
Q2
Que es lo que pedía el problema. 

114
11 | La Ley de Gauss

III Parcial 2012 I Ciclo Pregunta 2, Ampliación Verano 2013 Problema 5, III Parcial 2017 II Ciclo
Problema 3
Una pirámide con su base cuadrada de 10 m2 y 4 m de altura se halla en un campo eléctrico externo
E = 50 ĵ N/C. Si la punta de la pirámide señala hacia ĵ, calcule el flujo eléctrico a través de una de las
cuatro superficies inclinadas de la pirámide.

Solución
Hagamolo de 2 maneras
Método 1
Este es el método rápido. La piramide no está cargada, la carga dentro es 0. Entonces podemos asegurar
que ˛
qin
ΦE = E · dA = =0
0
El flujo total es 0. El flujo total es la suma de los flujos en todas las caras. Sea Φb el flujo en la base y
Φl el flujo de las superficies inclinadas. Todas las superficies inclinadas van a tener el mismo flujo, todas
tienen la misma área, misma ángulo respecto a E y tenemos 4 caras.
X Φb
ΦE = Φi = Φb + 4φl = 0 ⇒ Φl = −
4
i

El flujo Φb es facil de calcular, el ángulo entre E y la cara es 180◦ , entonces

Φb = E · Ab = −EAb
EAb
Φl = (11.1)
4
Método 2
Hagamos el producto punto
ΦE = E · A = EA cos(θ) (11.2)
Sea ` el lado de la base cuadrada, con A = `2 . Sea d la “altura” de las caras inclinadas como se muestra
en la Fig 11.1. El área de una cara inclinada es
`d
A` = (11.3)
2

115
CAPÍTULO 11. LA LEY DE GAUSS

Figura 11.1: Dimensiones de la piramide.


III Parcial 2012 I Ciclo Pregunta 2

El cos(θ) se ve de la Fig. 11.1 usando al definición de cos


 
`
2
cos(θ) = (11.4)
d
Usando las Ec. 11.3 y 11.4 en la Ec. 11.2 tenemos
E`2
  
`d (`/2) EA`
ΦE = E = =
2 d 4 4
Obtenemos el mismo resultado de la Ec. 11.1 con ambos métodos.
Evaluando
(50)(10) N · m2
ΦE = = 125
4 C
[Serway] Problema 24-63, [Resnick] Problema 27-2 Hay una superficie cerrada de dimensiones a =
b = 0.400 m y c = 0.600 m como se muestra en la Fig. 11.2. El borde izquierdo de la superficie cerrada
 x = a. El campo eléctrico que pasa por la región no es uniforme y está dado por
se está en la posición
2
E = 3.00 + 2.00x î N/C, donde x está en metros. (a) Calcule el flujo eléctrico neto en la superficie.
(b) Cual es la carga neta dentro de la superficie?

Figura 11.2: [Serway] Problema 24-63

Solución

116
CAPÍTULO 11. LA LEY DE GAUSS

(a) Pongamole nombre a las caras. Sea A1 la superficie apuntando en −î ubicada en x = a, A2 la cara
que apunta en î ubicada en x = a + c. EL flujo neto sobre la superficie cerrada es la sumatoria de los
flujos que pasan por cada cara.
X6
Φ= Φi
i=1

Donde el Φi corresponde a la Ai superficie. Veamos el flujo primero sobre el resto de caras. No es necesario
numerarlas. Cada una de esas caras apuntas en ±ĵ o ±k̂. Hagamos el producto punto de esas caras. Primero
para las que apuntas en ±ĵ. Para i 6= 1, 2
 
E · dAi = (algo) î · ±ĵ = 0

Lo mismo va a pasar para las caras que apuntan en ±k̂. Solo las caras A1 y A2 van a tener un flujo
diferente de 0. Calculemos cada flujo. En cada integral
ˆ ¨   ¨
2
3.00 + 2.00x2 dydz
 
Φ1 = E · dA1 = 3.00 + 2.00x dydz î · −î = −

La integral recorre sobre y y z. La variable x es constante respecto a y y z, entonces podemos sacarla de


la integral, evaluando que x = a
¨
2
dydz = − 3.00 + 2.00a2 ba
 
Φ1 = − 3.00 + 2.00a
˜
Donde usamos que dydz = ba, que es el área de la cara de integración.
Hacemos lo mismo para la cara A2 , con x = a + c
ˆ ¨ ¨
2
3.00 + 2.00x2 dydz = 3.00 + 2.00(a + c)2 ab
  
Φ1 = E · dA2 = 3.00 + 2.00x dydz î · î =

Sumamos los flujos


6
X
Φi = − 3.00 + 2.00a2 ba + 3.00 + 2.00(a + c)2 ab + 0 + 0 + 0 + 0
 
Φ=
i

⇒ Φ = 2ab (a + c)2 − a2
 

Evaluando h i N · m2
Φ = 2(0.4)(0.4) (0.4 + 0.6)2 − 0.42 = 0.269
C
(b) Usamos la ley de Gauss para saber la carga dentro de la superficie cerrada

Qin
⇒= Qin = 0 Φ = 8.85 × 10−12 (0.269)

Φ=
0

⇒ Qin = 2.38 × 10−12 C

[Resnick] Ejercicio 27-7


Una carga puntual +q se halla a una distancia d/2 de la superficie cuadrda de lado d y está arriba del
centro del cuadrado como se indica en la Fig 11.3. Determine el flujo eléctrico que atraviesa el cuadrado.

117
CAPÍTULO 11. LA LEY DE GAUSS

Figura 11.3: [Resnick] Problema 27-7

(Sugerencia: imagine el cuadrado como la cara de un cubo de lado d.)

Solución
Hagamolo por 2 métodos.
Método 1 Usemos la sugerencia. Si está dentro de un cubo, sabemos que el flujo en toda la caja es
q
ΦE =
0
El cubo es simétrico, cada una de las caras va a tener el mismo flujo. Y la cara de interés es uno de los

Figura 11.4: [Resnick] Problema 27-7

lados del cubo. Dividimos el flujo entre 6 que el cubo tiene 6 caras. El flujo a través de la superficie es
1q
Φ= (11.5)
6 0
Método 2 Este método es mandarse a lo tractor y hacer explícita la integral1
ˆ
E · dA

Por comodidad, cambiemos las dimensiones de d → 2d, posicionemos la carga en r = 0 y la superficie


cuadrada arriba de la carga en z = d. Fue solo reescalar, trasladar y rotar. El vector unitario es
r xî + y ĵ + dk̂
r̂ = =
r (x + y 2 + d2 )1/2
2

1
Requiere Cálculo III.

118
CAPÍTULO 11. LA LEY DE GAUSS

El vector dA es
dA = dxdy k̂
Entonces
q r̂ · dA q 1  
E · dA = = x î + y ĵ + dk̂ · dxdy k̂
4π0 r2 4π0 (x2 + y 2 + d2 )3/2
qd dxdy
⇒ E · dA =
4π0 (x2 + y 2 + d2 )3/2
Integramos
ˆ ˆd ˆd
qd 1
Φ= E · dA = dx dy
4π0 (x2 + y 2 + d2 )3/2
−d −d

Integrando primero en y usando la integral


ˆ
du u
=
u2 +a 2
a2 (u2 + a2 )1/2

Tenemos
ˆ d
" # d ˆ d
qd y qd 2d
Φ= dx = dx

1/2
4π0 4π0 + 2d2 )1/2

−d 2 2 2 2 2
(x + d ) (x + y + d ) −d (x2 + d2 ) (x2
y=−d

Le preguntamos a Mathematica cuanto da esta integral, Mathematica dice2


ˆ d
dx π
=
−d (x2 + d2 ) (x2 + 2d2 )1/2 3d2

Por lo tanto, el flujo es


2qd2 π 1q
⇒ ΦE = =
4π0 3d2 6 0
Da lo mismo que la Ec. 11.5 con el método simple.

[Serway] Problema 24-53


Una gran placa conductora situada en el plano xy tiene una carga por unidad de superficie de σ. Una
segunda placa situada por encima de la primera en z = z0 y orientada paralela al plano xy lleva una carga
por unidad de área de −2σ. Encuentre el campo eléctrico para (a) z < 0, (b) 0 < z < z0 , y (c) z > z0 .

Solución
El campo eléctrico por una lámina es es E = σ/(20 ), hacia afuera de la lámina (o hacia la lámina si está
cargada negativa) como se muestra en la Fig 11.5
Con 2 láminas, el campo total en cada región lo obtenemos por superposición, los campos en cada sección
se muestran en la Fig 11.6
2
https://www.smbc-comics.com/comic/2013-01-20

119
CAPÍTULO 11. LA LEY DE GAUSS

Figura 11.5: Campo de una lámina


[Serway] Problema 24-53

Figura 11.6: Campos de las láminas


[Serway] Problema 24-53

Para z < 0  
σ σ σ
E= − k̂ = k̂
0 20 20
Para 0 < z < z0  
σ σ 3σ
E= + k̂ = k̂
0 20 2 0
Para z0 < z  
σ σ σ
E= − + k̂ = − k̂
0 20 20

[Knight] Problema 24-38


En la Fig 11.7 se muestra una esfera metálica sólida en el centro de una esfera metálica hueca. Cuanto es
la carga total (a) en el exterior de la esfera interna, (b) en el interior de la esfera hueca, (c) en el exterior
de la esfera hueca?
Solución
Todas las cargas están distribuidas en esferas y de manera uniforme. Por el teorema de cascarones, sabemos
que esas cargas no van a contribuir al campo eléctrico dentro de ellas. Sea las dimensiones a = 5 cm,
b =10 cm y c = 15 cm, Qa , Qb y Qc las cargas en las superficies respectivas. Los puntos donde se miden el
campo eléctrico r1 = 8 cm y r2 = 15 cm. (a) Para r1 solo contribuye la carga de la esfera interna, en la
superficie interna ni externa de la esfera hueca contribuyen. Haciendo una superficie gaussiana de radio

120
CAPÍTULO 11. LA LEY DE GAUSS

Figura 11.7: [Knight] Problema 24-38

r1 y aprovechando la simetría esférica, que E es constante en la superficie de integración y anti paralelo3


a cada dA ˛ ˆ ˆ
Qin
E · dA = −EdA = −E dA = −EA =

Qa
−4πr12 E = ⇒ Qa = −4π0 r12 E = −4π 8.85 × 10−12 (0.08)2 (15000)

0
⇒ Qa = −1.067 × 10−8 C
(b) Dentro de los conductores en equilibrio, el campo eléctrico es 0. Si hacemos una superficie gaussiana
con un radio b < r < c, el flujo es 0 por que E es 0 en toda la superficie, entonces la carga encerrada es 0.
Ya que la carga en los conductores solo se puede almacenar en las superficies, las cargas dentro de la
superficie gaussiana es Qa y Qb . Entonces
Qin = Qa + Qb = 0 ⇒ Qb = −Qa = 1.067 × 10−8 C
(c) Hacemos de nuevo hacer una superficie gaussiana de radio r2 y aprovechamos la simetría igual que la
parte (a). Las cargan encerradas son Qa , Qb y Qc .
Qin Qa + Qb + Qc Qc
EA = = =
0 0 0
Qc = 0 EA = 4π0 r22 E = 4π 8.85 × 10−12 (0.17)2 (15000)


Qc = 4.82 × 10−8 C

[Resnick] Problema 27-14, [Sears] Problema 22-61, [Serway] Problema 22-64


Una región esférica tiene una carga uniforme por unidad de volumen ρ. Sea r el vector que va del centro
de la esfera hasta un punto general P dentro de ella. (a) Demuiestre que el campo eléctrico en P está
dado por E = ρr/(30 ). (b) Una cavidad esférica se crea en ella, com se aprecia en la FIGURA . Usando
superposición, demuestre que en todos los puntos de la cavidad el campo eléctrico es E = ρa/(30 ), donde
a es el vector que coneccta el centro de la esfera al de la cavidad.
3
Aquí estoy usando la magnitud del campo en lugar del campo y dirección, como si ya hubiera evaluado el producto
punto E · dA = |E| |dA| cos(π) = −EdA, no pongo el negativo por la dirección del campo evaluando hasta el final, si no ya
mismo y y uso la magnitud que es positiva.

121
CAPÍTULO 11. LA LEY DE GAUSS

Figura 11.8: [Resnick] Problema 27-14

Solución
(a) Usamos la ley de Gauss, haciendo una superficie gaussiana esférica de radio r. Aprovechamos que E
es constante en la superficie de integración y es paralelo a cada dA y que ρ es constante.
˛ ˆ ˆ
qin
E · dA = EdA = E dA = EA =
0
El área es A = 4πr2 ˆ
1 1 1 1
E= qin = ρdV
0 A 4π0 r2
ˆ
1 1 r 2 1 1 4 3
= 2
ρ4π r0 dr = ρ πr
4π0 r 0 4π0 r2 3
ρr
⇒E=
30
Esta es la magnitud. Para escribirlo como vector, usamos que E tiene direcció radial r̂. El campo eléctrico
como vector es
ρr ρ (rr̂) ρr
E= r̂ = =
30 30 30
ρr
E= (11.6)
30
Donde usamos que el vector posición es r = rr̂
(b) Usemos superposición. La esfera con densidad ρ con una cavidad es igual que una esfera completa de
densidad ρ y una esfera de densidad −ρ donde iba la cavidad, como se muestra en la Fig. 11.9 Sea r+ el

Figura 11.9: Superposición.


[Resnick] Problema 27-14

vector que sale desde el centro de la esfera y r− el vector que sale desde el centro de la cavidad. El campo
eléctrico es la superposición es
E = E+ + E−
Cambiamos cada campo eléctrico de cada esfera con la Ec. 11.6
ρr+ (−ρ)r− ρ
E = E+ + E− = + = (r+ − r− ) (11.7)
30 30 30

122
CAPÍTULO 11. LA LEY DE GAUSS

El vector r+ − r− lo podemos cambiar usando como suman los vectores, como se muestra en la Fig 11.10.

r+ = a + r− ⇒ a = r+ − r− (11.8)

Usando la Ec. 11.8 en la Ec. 11.7 se obtiene

Figura 11.10: Superposición.


[Resnick] Problema 27-14

ρa
E=
30
Es un campo de magnitud constante E = ρ |a| /(30 ) que va en la dirección del vector a.

[Sears] Problema 22-65, [Knight] Problema 24-60, [Irodov] Problema 3-25


II Parcial 2010 I Ciclo Pregunta 5, Reposición III Parcial 2014 I Ciclo Pregunta 4, III Parcial 2017 I
Ciclo Problema 3
Una distribución de carga no uniforme, pero con simetría esférica, obedece a la siguiente densidad de
carga ρ(r):
ρ(r) = ρ0 (1 − r/R) para r ≤ R
(11.9)
ρ(r) = 0 para r ≥ R
(a) Encuentre la carga total que encierra esta distribución. (b) Encuentre el campo eléctrico para r < R.
(c) Encuentre el campo eléctrico para r > R

Solución
(a) Para encontrar la carga total, integramos la densidad. Usamos que dV = 4πr2 dr, ya que ρ es solo
función de r ˆ ˆ R 
r πρ0 R3
Q = ρ (r) dV = ρ0 1 − 4πr2 dr = (11.10)
0 R 3
(b) Ya que la distribución de carga es radial, por el teorema de cascarones sabemos que la carga fuera de
la esfera donde construimos la superficie gaussiana no contribuye al campo eléctrico. Solo ocupamos saber
la carga encerrada para encontrar el E. El campo es constante en la superficie de integración y paralelo
a todos los dA ˛ ˆ ˆ
qin
E · dA = EdA = E dA = EA = 4πr2 E =
0
1 1
⇒E= qin
4π0 r2
Donde qin se obtiene integrando la densidad hasta un radio r
ˆ r 
r0

2
qin = ρ0 1 − 4π r0 dr0
0 R

123
CAPÍTULO 11. LA LEY DE GAUSS

Entonces ˆ r
r0
 
1 1 2
E= ρ0 1 − 4π r0 dr0 (11.11)
4π0 r2 0 R
r2
 
ρ0 r
⇒E= −
0 3 4R
(c) Hacemos lo mismo de la parte (b). Solo quye hay una diferencia, la superficie gaussiana va a tener el
mismo radio r, pero los límites de integración de la densidad van de 0 → R, ya que afuera de la esfera no
hay carga. Podemos reusar la Ec 11.11 con los respectivos límites de integración. El campo eléctrico es
ˆ
1 1 R r0
 
2
E= 2
ρ 0 1 − 4π r0 dr0
4π0 r 0 R

ρ0 R3 1
E=
120 r2

Advertencia: Vi mucha gente cometendiendo un error. Diciendo que E = 0 donde la densidad ρ es 0.


A diferencia del siguiente problema, el [Serway] Problema 24-67, dice que tiene cierta densidad ρ(r) y
que es un cilindro de radio R, es facil pensar que la densidad ρ(r) aplica solo cuando se está dentro del
cilindro, en r < R.
A como se está escribiendo en la ecuación 11.9, decir que ρ = 0 para R < r es una manera sofisticada
de decir que no hay carga fuera de la esfera. O poniendolo un poco redundante, no hay esfera fuera de la
esfera. Agarrando un caso límite, imaginemos una carga puntual ubicada en r = 0. La densidad la carga
puntual es 0 en todo el espacio, excepto en el punto r = 0. Y aún así, sí tenemos un campo eléctrico fuera
de donde está la carga, que es
1 q
E=
4π0 r2
En todo el espacio hay campo eléctrico y la densidad es 0 para r 6= 0.

[Serway] Problema 24-67


Un cilíndro aislante de longitud infinita y de radio R tiene una densidad de carga volumétrica que varía
en función del radio de la forma  r
ρ = ρ0 a −
b
donde ρ0 , a y b son constantes positivas y r la distancia al eje del cilindro. Utilice la ley de Gauss para
determinar la magnitud del campo eléctrico a las siguientes distancias radiales (a) r < R y (b) r > R.

Solución
Esto tiene la misma receta que el problema pasado, el [Sears] Problema 22-65. La única diferencia es
la geometría, aquí es un cilindro en lugar de una esfera. Aplicando la ley de Gauss, el flujo en las tapas
del cilindro es 0 por que el dA es perpendicular a E como se ve en la Fig. 11.11
En la superficie curva, el E es siempre paralelo a cada dA y constante sobre la superficie de integración,
entonces tenemos ˛ ˆ
qin
E · dA = EdA = EA = 2πrLE =
0
1
⇒E= qin
2πrL0

124
CAPÍTULO 11. LA LEY DE GAUSS

Figura 11.11: Superficie gaussiana dentro del cilindro.


[Serway] Problema 24-67

El dV en coordenadas cilindricas es dV = rdθdrdz. Integrando sobre todo el ángulo y un cilindro de largo


L, el dV es dV = 2πLrdr.
ˆ ˆ r 
r0

1 1
E= ρ (r) d = ρ0 a − 2πLr0 dr0
2πrL0 2πrL0 0 b
ˆ
11 r r0
 
⇒E= ρ0 a − r0 dr0 (11.12)
0 r 0 b
ρ0 ar r2
 
E= −
0 2 3b
(b) Igual que el problema pasado, [Sears] Problema 22-65 parte (c), la superficie gaussiana sigue
teniendo un radio r, pero los límites de integración de la densidad solo va de 0 → R, ya que la densidad
es 0 fuera del cilindro. Reusamos la Ec. 11.12 solo cambiamos los límites
ˆ
11 R r0
 
E= ρ0 a − r0 dr0
0 r 0 b

ρ0 1 aR2 R3
 
⇒E= −
0 r 2 3b

Problema de desafío4
[Knight] Problema 24-57
Todos los ejemplos de la ley de Gauss han tenido superficies altamente simetrícas donde el flujo es cero o
EA. Aún así, hemos dicho que el flujo neto ΦE = Qin /0 es independiente de la superficie. Vale la pena
chequear esto. En la Fig 11.13 se muestra un cubo de lado L centrado a lo largo de un cable largo delgado
con densidad de carga λ. El flujo a través en una de las caras del cubo no es simplemente EA, porque, en
este caso, el campo eléctrico varia tanto en magnitud como en dirección. Pero se puede calcular el flujo
haciendo la integral de flujo.
4
Requiere Cálculo III.

125
CAPÍTULO 11. LA LEY DE GAUSS

Figura 11.12: Superficie gaussiana fuera del cilindro.


[Serway] Problema 24-67

(a) Considere una cara paralela al plano yz. Defina el área dA como una cinta de grosor dy y altura L
con el vector apuntando en la dirección. Esa tira esta localizada en la posición y. Use el campo eléctrico
conocido
λ r̂
E=
2π0 r
en la tira para calcular el flujo eléctrico dΦ para esta área pequeña. Su expresión debería estar escrita en
términos de y, lo cual es una variable y varias constantes. No debería quedar en términos de algún ángulo.
b) Ahora integre dΦ para encontrar el flujo en esta cara.
c) Finalmente, muestre que el flujo neto a través del cubo es ΦE = Qin /0

Figura 11.13: [Knight] Problema 24-57

Solución
Saltandonos un poco las instrucciones del libro, hagamos directo la integral de superficie para una sola
cara El sistema es simétrico, entonces el flujo total es 4 veces el de una cara. Sea Φc el flujo para una de
las caras del cubo. Tomando la cara paralela a yz, a una distancia x = L/2. El diferencial dA es

dA = dydz î

126
CAPÍTULO 11. LA LEY DE GAUSS

El vector unitario r̂ antes de evaluar en x = L/2 es


 
r xî + y ĵ
r̂ = =
r (x2 + y 2 )1/2

Haciendo el producto punto


λ r̂ · dA λ 1     λ x
E · dA = = 2
x î + y ĵ · dydz î = dydz
2π0 r 2π0 r 2π0 x + y 2
2

Integrando y evaluando en x = L/2


ˆ ˆ L/2 ˆ L/2
λ L/2
Φc = E · dA = dz dy
z=−L/2 y=−L/2 2π0 (L/2)2 + y 2

Evaluamos la integral en z de inmediato que la función a integrar no depende de z y sacamos las constantes
  L/2
y
ˆ L/2

2 2 arctan
λ L 1 λ L L/2
⇒ Φc = dy 2 2
=
2π0 2 L/2 (L/2) + y 2π0 2 L/2


y=−L/2
    
λ L/2 −L/2 λ h π  π i
= L arctan − arctan = L − −
2π0 L/2 L/2 2π0 4 4
λ
⇒ Φc = L
40
Está es para una cara. Para las otras 4 caras no paralelas al plano xy tenemos el mismo flujo. Las
caras paralelas al plano xy que están en z = ±L/2 el flujo va a dar 0 por que el vector dA y E son
perpendiculares.  
xî + y ĵ
r̂ · dA = · (±)dxdy k̂ = 0
(x2 + y 2 )1/2
Entonces el flujo total es
λL
ΦE = 4Φc =
0
Y cambiando λL = Qin
Qin
ΦE =
0
La Ley de Gauss se mantiene para superficies donde el E no es constante sobre los dA como en magnitud
ni dirección.
Q.E.D. 

127
12 | Potencial Eléctrico

[Serway] Problema 25-31


Dos esferas aislantes tienen radios de R1 = 0.300 cm y R2 = 0.500 cm , con masas de m1 = 0.100 kg
y m2 = 0.700 kg, y cargas uniformemente distribuidas de q1 = −2.00 µC y q2 = 3.00 µC. Cuando sus
centros están separados una distancia de d = 1.00 m, estas esferas se liberan partiendo del reposo. Cuales
serán sus velocidades cuando entren en colisión?

Solución
La única fuerza que tenemos es la fuerza eléctrica, pero esta fuerza es conservativa. La energía del sistema
de conserva. La energía cinética incial es Ki = 0. Para esferar distribuidas uniformes, la fuerza y el
potencial es igual al de cargas puntuales, la energía potencial inicial es
1 q1 q2
Ui =
4π0 d
La energía cinética final es

Figura 12.1: Estado inicial.


[Serway] Problema 25-31

1 1
Kf = m1 v12 + m2 v22
2 2
Cuando entren en colisión, la distancia entre sus centros va a ser R1 +R2 , ya que las esferas no se traslapan.
1 q1 q2
Uf =
4π0 (R1 + R2 )

Igualando las energías


Ki + Ui = Kf + Uf

128
CAPÍTULO 12. POTENCIAL ELÉCTRICO

Figura 12.2: Estado final cuando colisionan.


[Serway] Problema 25-31

1 q1 q2 1 q1 q2 1 1
⇒0+ = + m1 v12 + m2 v22 (12.1)
4π0 d 4π0 (R1 + R2 ) 2 2
Tenemos 2 variables, v1 y v2 y una ecuación, ocupamos otra ecuación. Usemos momentum. No hay fuerzas
externas, por lo tanto, el momentum del sistema se conserva.
El momentum inicial es pi = 0, ya que las 2 esferas están inicialmente en reposo.

pi = pf ⇒ 0 = m1 v1 + m2 v2
m1
⇒ v2 = − v1 (12.2)
m2
m1
⇒ v2 = − v1
m2
Elevando al cuadrado1  2
m1
v22 = v12 (12.3)
m2
Usando el resultado de la Ec 12.3 en la Ec. 12.1
 2
1 q1 q2 1 q1 q2 1 1 m1
= + m1 v12 + m2 v12
4π0 d 4π0 (R1 + R2 ) 2 2 m2

Despejando v1 se obtiene v
u 2  
q1 q2 1 1
v1 = u −
u
m21
 
t 4π0 d (R1 + R2 )
m1 +
m2
Evaluando, y usando el signo de q1 que es negativo, por que usamos un q general, que se asume positivo
hasta evaluar.
v
(−2.00 × 10−6 ) (3.00 × 10−6 )
 
u 1 1
v1 = u2 (9 × 109 ) −
u
" #
u (0.1)2 (1.00) (0.003 + 0.005)
t (0.1) +
(0.7)

m
⇒ v1 = 10.8
s
1
Son vectores, lo que estoy haciendo en realidad es producto punto de v2 · v2 = v22 del LHS, y (m1 /m2 )v1 · (m1 /m2 )v1 =
(m1 /m2 )2 v12 del RHS.

129
CAPÍTULO 12. POTENCIAL ELÉCTRICO

Con v1 conocido, evaluamos en la Ec. 12.2 (Evaluandolo escalar)


0.1 m
v2 = (10.8) = 1.54
0.7 s

[Sears] Problema 23-52


Una esfera pequeña con masa de m = 5.00 × 10−7 kg y carga de q = 3.00 µC se libera del reposo a una
distancia de z1 = 0.400 m arriba de una lámina aislante horizontal grande con una carga que tiene una
densida de carga superficial uniforme σ = 8.00 pC/m2 . Usando conceptos de energía, calcule la rapidez
de la esfera cuando se encuentra a z2 = 0.100 arriba de la lámina cargada.
Solución
Para usar conservación de la energía, ocupo saber cuanto es el potencial en función de z. Intentemos
obtener el potencial integrando las cargas.

Figura 12.3: [Sears] Problema 23-52

p
Usando la Fig. 12.3, la distancia del dq a un punto z arriba de la lámina es z 2 + ρ2 , donde ρ es el radio
polar. Integrando en coodernadas polares con dA = ρdρdθ obtenemos
ˆ ˆ
1 dq 1 σdA
V (z) = = p
4π0 r 4π0 z 2 + ρ2
ˆ 2π ˆ ∞
σ ρ σ 1/2 ∞
= dθ dρ p = z 2 + r2 =∞ (12.4)
4π0 20

θ=0 r=0 z 2 + ρ2 r=0

El potencial nos da ∞. Es por tener una lámina infinita, este resultado es correcto, pero no nos sirve.
Ocupamos otro método.

Figura 12.4: [Sears] Problema 23-52

130
CAPÍTULO 12. POTENCIAL ELÉCTRICO

Obtengamos por ley de Gauss el E de una lámina infinita. De la Fig 12.4, usando que E solo pasa por las
tapas del cilindro, tenemos ˛
qin σA σ
E·A= ⇒ 2EA = ⇒E=
0 0 20
En lugar de usar la referencia con V (∞) = 0, cambiemos la referencia en V (z = 0) = 0 y evaluamos la
diferencia de potencial con la integral
ˆ z ˆ z
σ
V (z) − V (0) = − E · ds = −
0 z=0 20
σz
⇒ V (z) = −
20
Planteamos conservación de la energía, con la cinética Ki = 0 porque parte del reposo. La potencial es la
potencial por la gravedad y potencial por campo eléctrico
1
Ki + UGrav + UElec = Kf + UGrav + UElec ⇒ qV (z1 ) + mgz1 = mvf2 + qV (z2 ) + mgz2
2
Despejamos vf y sea ∆z = z1 − z2
r
2
vf = (mg∆z + q (V (z1 ) − Vz2 )) (12.5)
m
La diferencia de potencial es
σz1 σz2 σ∆z
V (z1 ) − V (z2 ) = − +− =−
20 20 20
s  

⇒ vf = 2 g − ∆z (12.6)
20 m
Evaluamos con los datos
s 
(3 × 10−6 ) (8 × 10−12 )

vf = 2 (9.8) − (0.4 − 0.1)
2 (8.85 × 10−12 ) (5 × 10−7 )
m
⇒ vf = 2.06
s
Extra: Aun podemos usar la integral y obtener la diferencia de potencial que ocupamos “restando 2
infinitos”. Antes de evaluar en el infinito, usemos el límite con R → ∞ donde R va a ser la variable con
que sacamos el límite.2 Reutilicemos rápido el resultado de la Ec. 12.4
ˆ 2π ˆ R ˆ 2π ˆ R
σ ρ σ ρ
V (z1 ) − V (z2 ) = lı́m dθ dρ p 2 − dθ dρ p 2
R→∞ 4π0 θ=0 r=0 z1 + ρ2 4π0 θ=0 r=0 z2 + ρ 2
ˆ R ˆ R
σ ρ σ ρ
= lı́m dρ p 2 − dρ p 2
R→∞ 20 r=0 z1 + ρ2 20 r=0 z2 + ρ2
 
σ 2
 R
2 1/2 2
 R
2 1/2
= lı́m z1 + ρ − z2 + ρ
R→∞ 20

ρ=0 ρ=0

2
También se podría ver como el límite de como se ve la diferencia de potencial entre z2 y z1 de un disco de radio R y
densidad σ y lo tendemos a infinito el radio.

131
CAPÍTULO 12. POTENCIAL ELÉCTRICO

σ h 2 1/2 1/2 i
= lı́m z1 + R 2 − z1 − z22 + R2 + z2
R→∞ 20
"  1/2 1/2 #
z12
 2
σ z1
= lı́m R +1 −R +1 − z1 + z 2
R→∞ 20 R2 R2
Los parentesis tienden a 1
 
 2  1
*
1/2  2 *1

1/2

σ 
 z1   z2   
= lı́m R + 1 − R + 1 − z 1 + z 2

R→∞ 20  R2 R2
  
 

−σ
lı́m [R − R − z1 + z2 ] = (z1 − z2 ) = V (z1 ) − V (z2 )
R→∞ 20
Este resultado lo podríamos usar en la Ec. 12.5 y obtener la Ec 12.6, solo que es más complicado así.

[Serway] Problema 25-47, [Knight] Problema 25-73


Ampliación Verano 2013 Pregunta 6, Ampliación 2017 I Ciclo Pregunta 5
Un alambre que tiene densidad lineal de carga λ se dobla como se muestra en la Fig 12.5. Calcule el
potencial eléctrico en el punto O.

Figura 12.5: [Serway] Problema 25-47

Solución
Dividamos el problema en 3 partes como se muestra en la Fig. 12.6, donde el potencial de toda la figura
va a ser
V = V1 + V2 + V3

Figura 12.6: Secciones del problema.


[Serway] Problema 25-47

Calculemos la parte 2, la sección circular. Todos los dq2 están a una distancia R del centro.
1 dq2 1 λd` 1 λRdθ 1
dV2 = = = = λdθ
4π0 R 4π0 R 4π0 R 4π0

132
CAPÍTULO 12. POTENCIAL ELÉCTRICO

ˆ π
1 λ
V2 = λdθ =
0 4π0 40
Para las partes 1 y 3, el potencial que hace la parte 1 es igual a la parte 3, las 2 son secciones con densidad
λ del mismo largo a la misma distancia desde donde se obtiene el potencial. Entonces V1 = V3 . Calculemos
V3 Ese segmento de cable mide 2R y el punto más cercano del segmento está a una distancia R. Entonces
el final del cable está a una distancia 3R. Estos son los límites de integración, de R → 3R. La distancia
al dq3 es la distancia x.
1 dq3 1 λdx
dV3 = =
4π0 r 4π0 x
ˆ 3R
1 λdx
V3 = (12.7)
R 4π0 x
1
⇒ V3 = λ ln (3)
4π0
Con V2 , V3 y V1 que es igual a V3 , sumamos los potenciales de todas las secciones
1 λ 1
V = V1 + V2 + V3 = λ ln (3) + + λ ln (3)
4π0 40 4π0
λ 2
V = + λ ln (3)
40 4π0
Advertencia: Usamos que V1 = V3 para ahorrarnos un cálculo. Hagamos algo que parece bien, pero
está mal, obtengamos el V1 calculandolo. Reusamos la Ec. 12.7, solo que los límites de integración van de
−3R → −R. ˆ −R    
1 λdx 1 −R 1 1
V1 = = ln = ln
−3R 4π0 x 4π0 −3R 4π0 3
1 1
ln 3−1 = −

V1 = ln (3) = −V3
4π0 4π0
Obtuvimos el negativo, V1 = −V3 . Estoy integrando sobre cargas positivas, el potencial entre cargas
positivas no se cancela, no es un campo eléctrico que tienen que sumar como vector. Y obtendríamos mal
el potencial total V que sería V = V1 + V2 + V3 = V2 .
Donde está el error? El error está en usar x en el denominador como la coordenada para cualquier x. En
la fórmula de potencial para una carga puntual
1 q
V =
4π0 r
El r es una distancia, las distancias están definidas positvas. Cuando se integró de la región −3R → −R,
nada más poner x da valores negativos. Para correjir esto y no irse de pollos, ponemos un valor absoluto
para asegurarnos que las distancias sean positivas. x → |x|. En la región negativa, tenemos |x| → −x.
Usemos la Ec. 12.7 de manera correcta, no cambiando los límites de integración, si no el x → |x|
ˆ −R ˆ −R    
1 λdx 1 λdx 1 −R 1 1
V1 = =− =− ln =− ln
−3R 4π0 |x| −3R 4π0 x 4π0 −3R 4π0 3
1
V1 = ln (3) = V3
4π0
Ya obtuvimos el resultado correcto.
Moraleja: Hay que cuidar cuando r recorre intervalos negativos, que nos daría malos resultados.

133
CAPÍTULO 12. POTENCIAL ELÉCTRICO

[Sears] Problema 23-86


III Parcial 2017 I Ciclo Pregunta 5
El potencial eléctrico V en cierta región del espacio está dado por

V (x, y, z) = A x2 − 3y 2 + z 2

(12.8)

Donde A es una constante. (a) Obtenga una expresión para el campo eléctrico E en cualquier punto de
esta región, (b) Se mide el trabajo realizado por el campo cuando una carga de prueba de 1.5 µC se
mueve del punto (x, y, z) = (0, 0, 0.250 m) al origen y resulta ser 6 × 10−5 J, determine A, (c) determine
el campo eléctrico en el punto (0, 0, 0.250 m).

Solución
(a) Obtenemos el campo eléctrico con el gradiente, E = −∇V

∂V ∂ 
A(x2 − 3y 2 + z 2 ) = 2Ax

=
∂x ∂x
∂V ∂ 
A(x2 − 3y 2 + z 2 ) = −6Ay

=
∂y ∂y
∂V ∂ 
A(x2 − 3y 2 + z 2 ) = 2Az

=
∂z ∂z
∂V ∂V ∂V
E = −∇V = − î − ĵ − k̂
∂x ∂y ∂z
E = −2Axî + 6Ay ĵ − 2Ay k̂ (12.9)
(b) El trabajo por el campo eléctrico es
ˆ f ˆ f
Wi→f = F · ds = q E · ds
i i

Tenemos 2 opciones, parametrizamos la trayectoria y el campo eléctrico e integramos3 , o aprovechamos


que este campo eléctrico conservativo por que viene de una función potencial4 .
ˆ f  ˆ f 
Wi→f = q E · ds = −q − E · ds = −q [Vf − Vi ]
i i

Evaluamos. Sea zi = 0.250 m. Evaluamos la diferencia de potencial

Wi→f = −qA (0 + 0 + 0) − 0 + 0 + z02 = Aqz02


 

Despejamos A
6 × 10−5

Wi→f V
A= = 2 = 640 m2
qz02 −6
(1.5 × 10 ) (0.25)
3
En este caso es facil que es una línea recta, no una curva de funciones feas.
4
Este caso también es obvio ver que viene de una función potencial por que nos la dieron, pero si no sabemos, si sacamos
el rotacional y obtenemos ∇ × E = 0, sabemos que viene de una función potencial y aprovechamos no hacer la integral de
línea.

134
CAPÍTULO 12. POTENCIAL ELÉCTRICO

Las unidades de A se pueden ver de la Ec. 12.8. Del LHS tenemos el potencial, que tiene unidades de
[V], y del RHS tenemos [A][m2 ], despejamos [V] = [A][m2 ] ⇒ [A] = [V][m−2 ] (c) Usamos la Ec. 12.9,
evaluamos en el punto (0, 0, 0.250 m) y sabiendo el valor de A
h i V
E = −(640) 0î + 0ĵ + 2(0.250)k̂ = −320k̂
m

III Parcial 2013 I Ciclo Pregunta 5, Reposición III Parcial 2014 I Ciclo Pregunta 5, III Parcial Verano
2014 III Ciclo Pregunta 5
Un disco de radio R tiene una distribución de carga superficial dada por σ = σ0 r2 /R2 , donde σ0 es
una constante y r la distancia desde el centro del disco, determine: (a) la carga total del disco. (b) El
potencial eléctrico a una distancia z del centro del disco en su eje perpendicular y que pasa por el centro.

Solución
(a) La carga total la obtenemos integrando la densidad de carga. Usamos coordenadas polares, el dA es
dA = rdrdθ ˆ ˆ 2π ˆ R
r2 π
Q = σ(r)dA = dθ dr r 2 σ0 = R2 σ0
θ=0 r=0 R 2
p
(b) Viendo la Fig 12.7, la distancia r al dq lo obtenemos con pitagoras, que es r = z 2 + ρ2 , donde ρ es
el radio polar

Figura 12.7: Distancias del problema.


III Parcial 2013 I Ciclo Pregunta 5

Obtenemos el dV usando que el dA = ρdρdθ


1 dq 1 σ (r) dA 1 ρ2 ρdρdθ
dV = = p = σ0 2 p
4π0 r 4π0 z 2 + ρ2 4π0 R z 2 + ρ2
Integramos ˆ ˆ
2π R
1 σ0 ρ3
V = dθ dρ p
4π0 R2 θ=0 ρ=0 z 2 + ρ2
Según Mathematica, la integral es
ˆ
x3 1 2 p
dx √ = x − 2z 2 x2 + z 2
z 2 + x2 3

135
CAPÍTULO 12. POTENCIAL ELÉCTRICO

El potencial es
 R
1 σ0 1 2 2
p
2 2
V = ρ − 2z ρ +z
20 R2 3 ρ=0

1 σ0 h
2 2
 p
2 + z 2 − 2z 3
i
⇒V = R − 2z R
20 R2

[Resnick] Problema 24-4


El campo eléctrico dentro de una esfera no conductora de radio R, que contiene una densidad de carga
uniforme, sigue una dirección radial y tiene la magnitud
qr
E=
4π0 R3
donde q es la carga total de la esfera y r la distancia respecto al centro de la esfera. (a) Determine el
potencial V dentro de la esfera, suponiendo que V = 0 en r = 0. (b) Cuál es la diferencia de potencial
elétrico entre un punto de la superficie y el centro de la esfera? Si q es positiva, qué punto se halla en el
potencial más alto? (c) Demuestre que el potencial a una a una distancia r del centro, donde r < R está
dado por
3R2 − r2
V =q
8π0 R3
donde el cero del potencial se toma con r = ∞. Por qué este resultado no es igual al de la parte (a)?

Solución
El campo eléctrico para dentro de la esfera nos lo están dando5 , el campo afuera para una distribución
radial es igual al de una carga puntual.
 qr
 para r < R
 4π0 R3


E(r) =
 q 1
para r > R


4π0 r2

El campo eléctrico va a tener esta forma independiente de donde agarre la referencia del potencial 0.
(a) Usemos los subíndices V0 (r) para la referencia V0 (r = 0) = 0 y V∞ (r) para la referencia V∞ (r = ∞) =
0.
La diferencia de potencial la obtenemos usando el campo eléctrico
ˆ B
VB − VA = − E · ds
A

Tomando a B en r y A en r = 0 en la referencia V0 (r = 0) = 0
ˆ r
qr0
V0 (r) − V0 (0) = V0 (r) = − dr0
0 4π0 R3
5
Que se probó en el problema [Resnick] Problema 27-14 de la sección de la Ley de Gauss, solo que lo expresé con ρ en
lugar de la carga total, pero el cambio se hace inmediato.

136
CAPÍTULO 12. POTENCIAL ELÉCTRICO

q r2
V0 (r) = − (12.10)
4π0 2R3
(b) Evaluando la Ec. en r = R
q 1 q 1
V0 (R) − V0 (0) = − −0=− (12.11)
4π0 2R 4π0 2R
Tomando q > 0, el V0 (R) − V0 (0) < 0 que es negativo, ⇒ V0 (R) < V0 (0). El potencial en R es menor,
porque el potencial disminuye si se va en la dirección del campo eléctrico.
(c)Ahora tomemos la referencia V∞ (r = ∞) = 0. Calculamos el potencial
ˆ r
V∞ (r) − V∞ (r = ∞) = − E · ds

La integral la tenemos que partir en 2 porque el E es diferente dependiendo de r. Una parte de ∞ → R


y la otra de R → r ˆ R ˆ r
V∞ (r) = − E · ds − E · ds
∞ R
ˆ ˆ
R r
qr0 r 2 − R2
 
q0 1 0 q 1 q 1
=− dr − dr = −
∞ 4π0 (r0 )2 R 4π0 R 3 4π0 R 4π0 R3 2
q 3R2 − r2 q 3R2 − r2
⇒ V∞ (r) = = (12.12)
4π0 2R3 8π0 R3
Que es lo que pedía el problema. La función V (r) no da igual para ambos casos porque la referencia de
donde es 0 se están tomando en diferentes lugares. Aún así, debería obtener los mismos resultados para
la forma del gráfico del potencial, diferente de potencial en diferentes puntos, campos eléctricos.
Hagamos cosas extra al problema.
Extra: Saquemos la diferencia de potencial usando la Ec. 12.12

q 3R2 − R2 q 3R2
V∞ (r = R) − V∞ (r = 0) = −
8π0 R3 8π0 R3
q 1
V∞ (r = R) − V∞ (r = 0) = − (12.13)
8π0 2R
El ∆V de la Ec. 12.13 con la referencia V∞ (r = ∞) es el mismo que se obtiene en la Ec. 12.11. La
referencia no va a cambiar en ∆V entre 2 puntos.
Obtengamos el potencial para r > R en ambas referencias. Para la referencia V∞ es igual al de una carga
puntual. Para el V0 (r) hacemos la integral, que tenemos que dividir en 2 regiones porque el campo eléctrico
cambia ˆ r
V0 (r) − V0 (0) = − E · ds
0
ˆ ˆ
R
qr0 r  
0 q
0 1 q 1 q 1 1
=− dr − dr 2 =− − − +
0 4π0 R3 R
0
4π0 (r ) 4π0 2R 4π0 r R
q 1 q 31
V0 (r) = −
4π0 r 4π0 2 R

137
CAPÍTULO 12. POTENCIAL ELÉCTRICO

El potencial V0 para todo r es


q r2


 − para r < R
4π0 2R3


V0 (r) =

 q 1 q 31
− para r > R


4π0 r 4π0 2 R
El potencial V∞ para todo r, reacomodando un poco la Ec. 12.12 es
q r2

q 31


 −
 4π0 2 R 4π0 2R3 para r < R
V∞ (r) =

 q 1
para r > R


4π0 r
Las 2 funciones para r < R es una parábola negativa, donde el coeficiente de r2 es q/(4π0 ) · 3/(2R) y
las 2 funciones para r > R dependen de r−1 y el coficiente de r−1 es q/(4π0 ). Las expresiones para V (r)
dependen de la misma manera en r, solo diferen de la constante
q 31
V∞ (r) − V0 (r) =
4π0 2 R
para todo r. Solo cambia la constante de referencia, pero la dependencia de r es la misma.6

[Resnick] Problema 28-10


III Parcial 2015 I Ciclo Problema 5
Una cantidad total de carga positiva Q se esparce sobre un anillo no conductor circular y plano de radio
interno a y radio externo b. La carga se distribuye de modo que la densidad de carga por unidad de
área está dada por σ = k/r3 , donde r es la distancia del centro del anillo a un punto cualquiera de él.
Demuestre (con V = 0 en el infinito) el potencial en el centro está dado por
 
Q a+b
V =
8π0 ab

(a) Relacionemos Q y k. La carga Q se obtiene integrando la densidad de carga σ. Usamos coordenadas


polares, con dA = rdrdθ
ˆ ˆ 2π ˆ b ˆ b  
kr 1 1 1 b−a
Q = σ (r) dA = dθ dr 3 = 2πk dr 2 = 2πk − = 2πk
θ=0 r=a r r=a r a b ab
Q ab
⇒k= (12.14)
2π b − a
Con el valor de k, obtengamos ahora el potencial en el centro. En este problema en particular, la distancia
de las cargas hasta el punto de interés es el mismo que el radio polar.7 El potenciual en el centro es
ˆ ˆ
1 dq 1 σ (r) dA
V = =
4π0 r 4π0 r
6
Por lo tanto, las 2 funciones tambien van a tener el mismo E. Cuando se hace E = −∇V , las derivadas quitan las
constantes, que es por lo único que difiere V∞ y V0 , entonces no afecta.
7
El r de V = (1/4π0 )q/r es igual al r polar. Aquí r = r aunque suene tonto resaltarlo, pero como siempre, esto lo
digo porque he visto gente comentiendo estos tipos de errores, que no ayuda que nos quedamos sin letras para notación y
confunden los r usados en diferentes contextos. Vease el III Parcial 2013 I Ciclo Pregunta 5, que el r del potencial no es
el mismo que el polar

138
CAPÍTULO 12. POTENCIAL ELÉCTRICO

ˆ 2π ˆ b ˆ b
1 k r 1 1
V = dθ = dr
4π0 θ=0 r=a r3 r 20 a r3
Intengrando y cambiando el k con la Ec. 12.14

Q ab b2 − a2
 
Q ab 1 1 1
− =
4π0 b − a 2 a2 b2 8π0 b − a a2 b2

Usando b2 − a2 = (b − a)(b + a) para simplificar a la respuesta que quiere el problema

Q ab (b − a)(b + a)
V =
8π0 b − a a2 b2
Cancelando términos  
Q a+b
⇒V =
8π0 ab
Que era lo que se buscaba.

Problema de desafío
[Purcell] Problema 1-39
Cuatro cuerpos con cargas positivas, dos con cargas Q y dos con cargas q, son conectadas por cuerdas no
extensibles de igual longitud. En ausencia de fuerzas externas, asumen la configuració de equilibrio que
se muestra en la figura. Muestre que tan3 θ = q 2 /Q2 . (Capítulo de Campo Eléctrico) Usted puede
mostrar que esta relación se debe mantener si la fuerza total en cada cuerpo, the suma vectorial de las
tensiones de la cuerdas y la repulsión electrica, es cero. (Capítulo de Potencial) O usted podría escribir
la expresión de la energía potencial del ensamble y minimizarla.

Figura 12.8: [Purcell] Problema 1.39

Solución
Consideremos la energía potencial del sistema. El sistema se va a acomodar de tal manera que va a estar
en un mínimo de energía. Como la energía depennde del ángulo, tenemos que encontrar un θ0 tal que
dU/dθ = 0 en θ0 . Y como derivamos en el proceso, los términos constantes de energía potencial se van
y solo quedan los términos que dependen de θ. La energía potencial entre una carga q y Q es constante
porque siempre es la distancia entre ellas es L, no cambia. Las energías que pueden variar con el ángulo
son la energía potencial entre q y q, la otra es U entre Q y Q.

139
CAPÍTULO 12. POTENCIAL ELÉCTRICO

Figura 12.9: Distancias de las cargas.


[Purcell] Problema 1.39

De la Fig 12.9, veamos que la distancia entre q y q es rqq = 2L sin(θ) y la distancia entre Q y Q es
rQQ = 2L cos(θ). La enería potencial del sistema es

1 q2 1 Q2
U (θ) = + + U0
4π0 2L sin(θ) 4π0 2L cos(θ)

Donde U0 es la energía de las cargas que son constantes. Sacamos a factor común y derivamos
 
dU 1 1 cos(θ) 2 sin(θ) 2
= − 2 q + Q =0
dθ 4π0 2L sin (θ) cos2 (θ)

El término entre los parentesis cuadrados es 0

cos(θ) 2 sin(θ) 2 cos(θ) 2 sin(θ) 2


− 2 q + 2
Q =0⇒ 2 q = 2 (θ)
Q
sin (θ) cos (θ) sin (θ) cos

Movemos términos
sin3 (θ) q2 3 q2
= ⇒ tan (θ) =
cos3 (θ) Q2 Q2
Que es lo que el problema pide.

140
13 | Capacitancia

[Serway] Problema 26-58


Dos grandes placas metálicas paralelas, cada una de área A, están orientadas en sentido horizontal y
están separadas a una distancia 3d. Están unidas mediante un alambre a tierra e inicialmente las placas
no tienen carga. Ahora se inserta entre las placas una tercera placa idéntica con carga Q, paralelamente
a las anteriores y localizada a una distancia d de la placa superior, como se observa en la Fig. 13.1. (a)
Cuál es la carga inducida que aparece en cada una de las dos placas originales? (b) Cuál es la diferencia
de potencial que aparece entre la placa intermedia y cada una de las demás placas?

Figura 13.1: [Serway] Problema 26-58

Solución
(a) Sea la parte de arriba 1 y la de abajo 2 como se muestra en al Fig. 13.2. La carga Q se va a distribuir
arriba y abajo de la placa central, con densidades σ1 y σ2 tal que

Q = A (σ1 + σ2 ) (13.1)

Figura 13.2: [Serway] Problema 26-58

141
CAPÍTULO 13. CAPACITANCIA

Ya que la carga se conserva. Ocupamos otra ecuación que tenemos 2 variables y una ecuación. Estás cargas
van a inducir cargas en las otras placas, de −σ1 arriba y −σ2 . Entre cada sección, se genera un campo
E constante de la forma Ei = σi /0 . Podemos relaciones estos campos eléctricos notando que las placas
de arriba y abajo están al mismo potencial. De la placa cargada en el centro a cualquiera de las otras
placas hay una diferencia de potencial V , aún no sabemos el valor de V , pero sabemos que es el mismo
por ambos caminos. Para campos eléctricos constantes tenemos V = Ei di . Entonces relacionamos
σ1 σ2
V = V1 = V2 ⇒ E1 d1 = E2 d2 ⇒ d= 2d
0 0
σ1 = 2σ2 (13.2)
Con las Ec. 13.1 y 13.2, ya tenemos 2 variables y 2 ecuaciones. Despejando σ1 y σ2 , obtenemos
2Q
σ1 =
3A
1Q
σ2 =
3A
Multiplicando por A cada σi , obtenemos las cargas inducidas en cara cada, que son
2
Q1 = Q
3
1
Q2 = Q
3
(b) La diferencia de potencial la obtenemos con cualquiera de los V = Ei di , evaluemos con 1
σ1 2 Qd
V = E1 d1 = d=
0 3 A0

[Serway] Problema 26-55


Cuatro placas metálicas paralelas P1 , P2 , P3 y P4 , cada una con una superficie de 7.50 cm2 , están separadas
sucesivamente por una distancia d = 1.19 mm, como se muestra en la Fig 13.3. La placa P1 está conectada
a la terminal negativa de una batería y P2 a la terminal positiva. La batería mantiene una diferencia de
potencial de V = 12.0 V. (a) Si P3 se conecta a la terminal negativa, cuál es la capacitancia del sistema
de placas P1 P2 P3 ? (b) Cuál es la carga de P2 ? (c) Si se conecta P4 a la terminal positiva de la batería,
cuál será la capacitancia del sistema de cuatro placas P1 P2 P3 P4 ? (d) Cual es la carga de P4 ?
Solución
Podemos obtener las capacitancias, las partes (a) y (c) de 2 maneras
Método 1
(a) Todas las placas están a la misma distancia d de la otra placa más cercana. De las placas 2 → 1 y del
2 → 3 tienen la misma diferencial de potencial y la misma distancia d. Entre placas cercanas podemos
asumir que el campo eléctrico es constante, entonces
V
V = Ed ⇒ E =
d
El campo eléctrico es igual entre todas las placas, ya que el V /d es igual para todas. El campo eléctrico
entre las placas es E = σ/0 , y como todas las placas tiene el mismo E, todas las placas tiene la misma

142
CAPÍTULO 13. CAPACITANCIA

Figura 13.3: [Serway] Problema 26-55

Figura 13.4: Cargas inducidas en las placas.


[Serway] Problema 26-55

σ. Además, solo las partes “internas” de las placas van a tener una carga inducida. Esto resumido se ve
en la Fig. 13.4
La carga almacenada en placa 2 es
Q1 = 2σA
Cambiamos σ con E = σ/0 y el E con E = V /d
2V A0
Q1 = 2σA = 2E0 A = (13.3)
d
Obteniendo la proporción C1 = Q/V
Q 2A0
C1 = = (13.4)
V d
Evaluamos con los datos
2(7.5 × 104 )(8.85 × 10−12 )
C1 = = 1.15 × 10−11 F = 11.2 pF
1.19 × 10−3
(b) La carga la encontramos como paso intermedio de la parte (a), que es la Ec. 13.3
2V A0 2(12)(7.5 × 104 )(8.85 × 10−12 )
Q1 = = = 1.34 × 10−10 C = 134 pC
d 1.19 × 10−3

143
CAPÍTULO 13. CAPACITANCIA

También podríamos averiguar Q1 para evitar tanto número en la calculadora y aprovechar el resultado
resultados, ya sabemos la capacitancia C1 , usamos la definición de capacitancia y obtenemos
Q1
C1 = ⇒ Q1 = C1 V = (11.2) (12) pC = 134 pC (13.5)
V
(c) Cuando se conecta la placa 4 al sistema, todos los E siguen siendo iguales ya que la relación E = V /d
es la misma, por lo tanto, también los σ en cada cara de las placas. Solo que ahora se induce cargas en
las superficies entre 3-4, como se muestra en la Fig. 13.5 Siguiendo el mismo procedimiento de la parte

Figura 13.5: Cargas inducidas en las placas con la placa 4.


[Serway] Problema 26-55

(a), solo que en lugar de tener 2 caras con superficies inducidas, tenemos 3. Va a dar lo mismo que la Ec.
13.3, solo que con 3 en lugar de 2
3V A0
Q2 =
d
Dividiendo para obtener la proporción Q2 /V

Q2 3A0 3(7.5 × 104 )(8.85 × 10−12 )


C2 = = = = 1.67 × 10−11 F = 16.7 pF (13.6)
V d 1.19 × 10−3
(d) La placa 4 tiene solo carga solo tiene carga inducida en 1 cara, la carga en P4 es

V A0 (12)(7.5 × 104 )(8.85 × 10−12 )


QP 4 = σA = = = 6.69 × 10−11 C = 66.9 pC
d 1.19 × 10−3

Método 2
(a) En lugar de obtener la capacitancia obteniendo la relación C = Q/V , podemos usar que entre la placa
2 y 1 se forma un capacitor y la placa 2 y 3 se forma otro como se muestra en a la izquierda Fig 13.6.
Tenemos 2 capacitores en paralelo, cada uno de área A y distancia d, cada capacitor es
A0
C=
d
Los 2 capacitores están en paralelo, entonces la capacitancia equivalente es
2A0
C1 = C + C =
d

144
CAPÍTULO 13. CAPACITANCIA

Que es lo mismo que la Ec. 13.4, y podemos obtener la carga como se obtuvo en la parte (b) sabiendo
la capacitancia del sistema. (c) Tenemos los mismos capacitores entre 2-1, y 2-3, y se forma un nuevo
capacitor en paralelo con las placas 4-3, de área A y separación d. Tenemos 3 capacitores el paralelo como
se muestra en a la derecha de la Fig. 13.6, la capacitancia equivalente es
3A0
C2 = C + C + C =
d
Obtenemos el mismo resultado de la Ec. 13.6

Figura 13.6: Esquema del sistema.


[Serway] Problema 26-55

[Sears] Problema 24-59


En la Fig. 13.7, C1 = 3.00 µF y Vab = 150 V. La carga en el capacitor C1 es de 150 µC y la carga en C3
es de 450 µC. Cuáles son los valores de las capacitancias de C2 y C3 ?

Figura 13.7: [Sears] Problema 24-59

Solución
Los capacitores C1 y C2 al estar en paralelo tienen la misma diferencia de potencial. Podemos averiguar
el V1 ya que sabemos q1 y C1 .
Q1 150 × 10−6
V1 = = = 50 V
C1 3.00 × 10−6
EL voltaje Vab es la suma de los voltajes de los componentes eléctricos, Vab = V1 +V3 , de aquí averiguamos
el voltaje en C3
V3 = Vab − V1 = 150 − 50 = 100 V
Y sabemos cuanto es la carga en C3 , entonces
Q3 150 × 10−6
=
C3 = = 4.50 µF
V3 100
Sabemos cuanto es V2 , es el mismo voltaje que V1 , V2 = 50 V. Podemos averiguar la carga que está en C2 .
Suponiendo que estaba descargado1 todo el sistema antes se conectara a Vab , la carga en el nodo interno
1
Sería raro que estuviera cargado y luego se conectara a Vab , aunque sí es posible y cambia la carga “en el nodo interno”
que conecta a los capacitores, pero eso usualmente no pasa.

145
CAPÍTULO 13. CAPACITANCIA

que conecta a los capacitores, la sección encerrada que se muestra en la Fig. 13.8, la carga adentro es 0.
Dentro del circulo está la placa cargada negativa de C1 y C2 y la positiva de C3 .

Figura 13.8: [Sears] Problema 24-59

Entonces cumple que

Qdentro = −Q1 − Q2 + Q3 = 0 ⇒ Q2 = −Q1 + Q3 = −150 + 450 = 300 µC

Donde Q2 tiene signo positivo aunque haya tomando la placa negativa. Ya que estoy usando las cargas
en el capacitor como positivas (La carga en la placa positiva) 2 , le pongo un menos en −Q1 y Q2 para
tomar el signo correcto en las placas negativas.
Con el valor de Q2 y V2 , averiguamos C2 con

Q2 300 × 10−6
C2 = = = 6.00 µF
V2 50

[Resnick] Problema 30-9


IV Parcial Verano 2013 Pregunta 1
Una batería produce una diferencia de potencial de ∆V de 12 V. (a) Determine la carga en cada capacitor
cuando el interruptor S está abierto y (b) cuando (más tarde) el interruptor S está cerrado. Suponga que
C1 = 1.0 µF, C2 = 2.0 µF, C3 = 3.0 µF y C4 = 4.0 µF

Figura 13.9: [Resnick] Problema 30-12

Solución
2
Aunque legitimamente los capacitores tienen carga neta 0.

146
CAPÍTULO 13. CAPACITANCIA

Cuando el interruptor está abierto, C1 y C3 quedan en serie, C2 y C4 quedan en serie. No ocupamos


sacar la capacitancia equivalente de todo el sistema para saber la carga en cada capacitor. Los capacitores
equivalentes de C13 y C24 es
 
1 1 1 1 1 1 4
= + = + ⇒ C13 = µF
C13 C1 C3 1.0 3.0 µF 3
 
1 1 1 1 1 1 4
= + = + ⇒ C24 = µF
C24 C2 C4 2.0 4.0 µF 3
Los capacitores equivalentes C13 y C24 tiene un voltaje V , las cargas en cada capacitor equivalente es
 
3
Q13 = C13 V = (12) µC = 9 µC
4
 
4
Q24 = C24 V = (12) µC = 16 µC
3
Ahora nos devolvemos al circuito original. El capacitor C13 es el equivalente en serie, los capacitores en
serie tienen la misma carga que el capacitor equivalente

Q13 = Q1 = Q3 = 9 µC

(b) Cuando es interruptor se cierra, C1 y C2 quedan en paralelo, C3 y C4 quedan en paralelo. A diferencia


de la parte (a), que los equivalentes C13 y C24 ya están a un voltaje conocido V , aquí no sabemos. Tene-
mos que sacar el equivalente total e irnos en “reversa” averiguando los datos necesarios. La capacitancia
equivlente C12 es
C12 = C1 + C2 = 1 + 2 µF = 3 µF
C34 = C3 + C4 = 3 + 4 µF = 7 µF
Los equivalentes C12 y C34 están en serie. La capacitancia equivalente es
1 1 1 1 1 1 21
= + = + ⇒ Ceq = µF
Ceq C12 C34 3 7 µF 10

La carga que entra al todo el sistema es


 
21
Q = Ceq V = 12 µC = 25.2 µC
10

De aquí empezamos a irnos en reversa al circuito original, mantiendo los valores necesarios como se
muestra en la Fig. 13.10
Ya que Ceq es el equivalente de C12 y C34 en serie, los equivalentes C12 y C34 van a tener la carga Q. Los
voltajes en cada uno de los equivalentes es
Q 25.2 µC
V12 = = = 8.4 V
C12 3 µF
Q 25.2 µC
V34 = = = 3.6 V
C34 7 µF

147
CAPÍTULO 13. CAPACITANCIA

Figura 13.10: Valores importantes del circuito.


[Resnick] Problema 30-12

Como C12 es el equivalente de C1 y C2 en paralelo, van a tener el mismo voltaje que C12 , V1 = V2 =
V12 = 8.4 V. Igual para 3 y 4, el voltaje de 3 y 4 es V3 = V4 = V34 = 3.6 V.
Con los voltajes de todos los capacitores, podemos averiguar las cargas en cada capacitor.

Q1 = C1 V1 = (1)(8.4) µC = 8.4 µC

Q2 = C2 V2 = (2)(8.4) µC = 16.8 µC
Q3 = C3 V3 = (3)(3.6) µC = 10.8 µC
Q4 = C4 V4 = (4)(3.6) µC = 14.4 µC

Para comprobar que lo hicimos bien, la región cuadrada de la PONER FIGURA tiene que sumar 0,
sumemos las cargas que se encuentran en esa región

Qneto = −Q1 − Q2 + Q3 + Q4 = −8.4 − 16.8 + 10.8 + 14.4 = 0 C

Sí nos dio 0.
[Resnick] Ejercicio 30-27
Un capacitor cilíndrico tiene radios a y b como se muestra en la Fig 13.12. Demuestre que √ la mitad de
la energía potencial eléctrica almacenada se halla dentro de un cilindro cuyo radio es r0 = ab. Suponga
que es un cilindro de largo L mucho mayor a los radios a y b.
Solución
Encontremos primero toda la energía contenida. Se puede hacer de 2 maneras.

148
CAPÍTULO 13. CAPACITANCIA

Figura 13.11: Región de carga neta 0.


[Resnick] Problema 30-12

Figura 13.12: Vista trasversal del cilindro.


[Resnick] Ejercicio 30-27

Método 1 Se puede probar usando la Ley de Gauss que el E es


q
E=
2πL0
La densidad de energía del campo eléctrico es u = (1/2)0 E 2 . Integramos sobre el volumen, usando
coordenadas cilindricas, donde dV = rdrdθdz
ˆ ˆ ˆ  2
1 1 q
U = u dV = 0 E 2 dV = 0 dV
2 2 2πL0
ˆ L ˆ 2π ˆ b
q2 q2 r
= 2 2 = 2 2 dz 2
8π L 0 8π L 0 0 0 a r
ˆ b
q2 dr
⇒U = (13.7)
4πL0 a r
q2
 
b
⇒U = ln (13.8)
4πL0 a
Método 2 Podemos usar energía almacenada en un capacitor
1 q2
U= (13.9)
2C
La capacitancia del un capacitor cilíndrico es C = 2π0 L/ ln(b/a). Sustituyendo en la Ec. 13.8 tenemos
q2 q2
 
1 b
U=  = ln
2 2π0 L 4πL0 a
ln(b/a)

149
CAPÍTULO 13. CAPACITANCIA

Da lo mismo que la Ec. 13.7 Con la energía total, encontremos la mitad de la energía. Reusemos la Ec.
13.7, solo que expresamos con diferentes límites de integración, que van de a → r0 , y sabemos cuanto es
la energía total, que está en la Ec. 13.8
ˆ r0
1 q2 dr
⇒ U=
2 4πL0 a r

1 q2 q2
 
b r 
0
ln = ln
2 4πL0 a 4πL0 a
 
1 b  r0 
⇒ ln = ln
2 a a
Despejando r0 , obtenemos √
r0 = ab

IV Parcial 2017 II Ciclo Problema 1


Un capacitor hecho por una esfera conductora de radio a y una esfera hueca de radio interno b están
cargados con una carga ±q como se muestra en la Fig. 13.13(a) Usando la densidad de energía eléctrica,
encuentre la energía almacenada en el capacitor. (b) Encuentre el radio r0 al cual desde r = a hasta
r = r0 se encuentra almacenada la mitad de la energía eléctrica.

Figura 13.13: Capacitor esférico.


IV Parcial 2017 II Ciclo Problema 1

Solución
Este problema se resuelve igual pasado, el [Resnick] Ejercicio 30-27, la diferencia es que tenemos una
esfera un lugar de un cilindro. El campo eléctrico es E = q/(4π0 r2 ), el dV es dV = 4πr2 dr, entonces
ˆ ˆ b 2
q2 1

1 1
U= 0 E 2 dV = 0 4πr2 dr
2 a 2 4π0 r
ˆ b
q2 dr
⇒U = (13.10)
8π0 a r2
q2
 
1 1
⇒U = − (13.11)
8π0 a b

150
CAPÍTULO 13. CAPACITANCIA

Para comprobar el resultado de la Ec. 13.11, podemos obtener este resultado usando la relación de la
energía almacenada en un capcitor, que es U = Q2 /(2C), la capacitancia de este sistema es
4π0
C= 
1 1

a b
La energía almacenada es

1 q2 q2 q2
 
1 1 1
U= =  = − (13.12)
2C 2 4π0 8π0 a b
1/a − 1/b
Que la lo mismo que la Ec. 13.11 que se obtuvo integrando la densidad de energía.
(b) Reusamos la Ec. 13.10, solo que cambiamos los límites de integración de a → r0 , donde r0 es el radio
a cual se encuentra almacenada la mitad de la energía eléctrica y usamos la energía total obtenida en la
parte (a) (La Ec. 13.11). ˆ r0
1 q2 dr
U=
2 8π0 a r2
1 q2 q2
   
1 1 1 1
⇒ − = −
2 8π0 a b 8π0 a r0
 
1 1 1 1 1
− = −
2 a b a r0
Despejando r0 , obtenemos  
1 1 
r0 =
2 1 1
 
+
a b
IV Parcial 2017 II Ciclo Problema 5
Se tiene una red de capacitores como se muestra en la Fig. 13.14 conectados a una V0 desconocida, pero
se conocen los valores de C1 , C2 , C3 y C4 y se conoce el valor de la carga Q2 en el capacitor C2 . Determine
en términos de Q2 , C1 , C2 , C3 y C4 : (a) La capacitancia equivalente del sistema. (b) La carga Q4 en el
capacitor C4 . (c) El voltaje V1 del capacitor C1 .

Figura 13.14: IV Parcial 2017 II Ciclo Problema 5

Solución

151
CAPÍTULO 13. CAPACITANCIA

(a) Los capacitores C2 y C3 están en serie, el C23 equivalente entre ellos es


1 1 1 C2 C3
= + ⇒ C23 = (13.13)
C23 C2 C3 C2 + C3
Los capacitores C23 y C4 están en paralelo, la equivalente entre ellos es

C234 = C23 + C4

El capacitor equivalente C234 está en serie con C1 . La capacitancia equivalente de todo el sistema es
1 1 1 C1 C234
= + ⇒ Ceq =
Ceq C C234 C1 + C234

(b) El capacitor C3 tiene la misma carga que C2 por estar en serie. El capacitor C23 va a tener la misma
carga Q2 de C2 . De aquí podemos obtener el voltaje de C4 que es el mismo de C23 . Usamos la Ec. 13.13
que obtuvimos C23 .  
Q 1 1
V4 = V23 = = Q2 +
C23 C2 C3
La carga Q4 es  
1 1
Q4 = V4 C4 = Q2 C4 +
C2 C3
(c) En el nodo interno que conecta a C1 , C2 y C4 , la carga suma 0. Sumando las cargas dentro de la
sección de la Fig. 13.15 tienen que sumar 0. Tomando que todos los Qi tienen signo positivo, tenemos que
  
1 1
−Q1 + Q2 + Q4 = 0 ⇒ Q1 = Q2 + Q4 = Q2 1 + C4 +
C2 C3
El voltaje de C1 es   
Q1 Q2 1 1
V1 = = 1 + C4 +
C1 C1 C2 C3

Figura 13.15: Nodo con carga neta 0.


IV Parcial 2017 II Ciclo Problema 5

Modificado del [Sears] Problema 24-21


IV Parcial 2017 I Ciclo Problema 4
Problema 4 Para el sistema de capacitores de la Fig. 13.16 se mantiene una diferencia de potencial de
VA − VB = 25 V a través de los nodos A y B. Determine:

152
CAPÍTULO 13. CAPACITANCIA

Figura 13.16: [Sears] Problema 24-21

(a) La capacitancia equivalente entre A y B. (b) Cuanta carga se almacena en el sistema. (c) La energía
almacenada en el capacitor de 10.0 nF. (d) La diferencia de potencial a través del capacitor de 30.0 nF.
Solución
(a) Sea C 0 la capacitancia equivalente de los capacitores en el medio 18.0, 30.0 y 10.0 nF. La capacitancia
equivalente de los capacitores en serie es
 
1 1 1 1 1
= + + ⇒ C 0 = 5.29 nF
C0 18.0 30.0 10.0 nF
La capacitancia equivalente de todo el sistema es

Ceq = 7.50 + C 0 + 6.50 mboxnF = 7.50 + 5.29 + 6.50 = 19.29 nF

(b) La carga almacenada en todo el sistema es

Q = Ceq Vab = (19.29)(25) nC = 482.25nC

(c) Para la parte (c) y (d) ocupamos la carga en los capacitores del centro. Esta no va a ser la carga
total Q del sistema. Podemos calcular la carga Q0 que el equivalente tiene una diferencia de potencial de
25 V, y por estar en serie, es la misma carga en los 3 capacitores del centro

Q0 = C 0 Vab = (5.29)(25) = 132 nF

La energía almacenada en el capacitor de 10.0 nF es


2
1 (Q0 )2 1 132 × 10−9
U10 = = = 8.712 × 10−7 J = 871.2 nJ
2 C 2 (10.0 × 10−9 )
(d) La diferencia de potencial en el capacitor de 30.0 nF es
Q0 132
V30 = = V = 4.4 V
C 30

Problema de desafío3
3
El problema es más “matematicoso” que físico, pero el resultado sí es relevante.

153
CAPÍTULO 13. CAPACITANCIA

Se tiene N capacitores en serie de valores conocidos C1 , C2 , . . . , CN . Pruebe que la capacitancia equivalente


Ceq es menor al menor de los capacitores. Ceq < min{C1 , C2 , . . . , CN }.
Esto sirve para chequear que se haya calculado bien la capacitancia equivalente, si es mayor a alguno de
los capacitores, o más directo, si la Ceq es mayor al menor de los capacitores, el cálculo de Ceq está mal
hecho.
Solución
La capacitancia equivalente de los N capacitores en serie es
N
1 X 1 1 1 1
= = + +·+
Ceq Ci C1 C2 CN
i

Juntemos todo con un solo denominador. Escribiendo todo con el mismo denominador, el primer término,
va a tener C2 C3 . . . CN , sin el C1 , para el segundo va a tener C1 C3 . . . CN , sin el C2 , así para todos los Ci
1 C 2 C 3 . . . CN C1 C3 . . . CN C1 C2 . . . Ck−1 Ck+1 . . . CN −1 CN C1 C2 . . . CN −1
= + + ··· + + ···
Ceq C1 C2 · · · CN C1 C2 · · · CN C1 C2 · · · Ck · · · CN C1 C2 · · · CN

1 C2 C3 . . . CN + · · · + C1 C2 . . . Ck−1 Ck+1 . . . CN + · · · + C1 C2 · · · CN −1
⇒ = (13.14)
Ceq C1 C2 · · · CN
Invertimos la Ec. 13.14
C1 C2 · · · CN
Ceq = (13.15)
C2 C3 . . . CN + · · · + C1 C2 . . . Ck−1 Ck+1 . . . CN + · · · + C1 C2 · · · CN −1

Dividimos entre Ck la Ec. 13.15 arbitrario y tenemos


Ceq C1 C2 . . . Ck−1 Ck+1 . . . CN
= (13.16)
Ck C2 C3 . . . CN + · · · + C1 C2 . . . Ck−1 Ck+1 . . . CN + · · · + C1 C2 · · · CN −1

El término del RHS de la Ec. 13.16 es menor a 1, el término C1 C2 . . . Ck−1 Ck+1 . . . CN que está en
el numerador también está abajo el denominador. Todas las capacitancias son positivas, entonces el
denominador va a ser más grande que el numerador. Viendolo con notación más facil, sean a, b, c... numeros
positivos. Entonces vamos a tener que
a
a < a + b + c + ··· ⇒ <1
a + b + c···
Es lo mismo en el RHS de la Ec. 13.16, es menor a 1. Entonces
Ceq C1 C2 . . . Ck−1 Ck+1 . . . CN
= <1
Ck C2 C3 . . . CN + · · · + C1 C2 . . . Ck−1 Ck+1 . . . CN + · · · + C1 C2 · · · CN −1

Ceq
⇒ < 1 ⇒ Ceq < Ck
Ck
Para cualquier k. En particular, sea Cm el menor capacitor del sistema, tenemos Ceq < Cm . La capacitancia
equivalente es menor que el menor de los capacitores en serie. Q.E.D. 

154
14 | Corriente y Resistencia

[Sears] Problema 25-15


Un filamento cilíndrico de tungsteno de L = 15.0 cm de largo y d = 1.00 mm de diámetro va a utilizarse
en una máquina cuya temperatura de operación variará entre la temperatura ambiente (Tmin = 20◦ C)
y Tmax = 120◦ C. Conducirá una corriente de 12.5 A en todas las temperaturas. (a) Cuál será el campo
eléctrico máximo en este filamento? (b) Cuál será su resistencia con ese campo? (c) Cuál será la máxima
caída de potencial a todo lo largo del filamento?
Datos adicionales: ρW = 5.25 × 10−8 Ω · m y αW = 0.0045 (◦ C)−1

Solución
El campo eléctrico es E = ρJ. Ocupamos maximizar E. La densidad de corriente, suponiendo que se
distribuye de manera uniforme es
I I I
J= = 2 =
A πr π(d/2)2
Vemos que J es una constante. Podemos variar ρ para maximizar E.
I
E = ρJ = ρ0 (1 + α∆T )
π(d/2)2
Maximizamos el E haciendo el ∆T lo más grande posible, que sería hacer Tmax − Tmin . Evaluamos
12.5 V
E = 5.25 × 10−8 [1 + (0.0045)(120 − 20)]

2 = 1.21 m
π (0.001/2)
(b) La resistencia también va a ser máxima cuando ρ es máxima.

5.25 × 10−8 [1 + (0.0045)(120 − 20)] (0.15)



ρL ρ0 (1 + α∆T ) L
R= = = = 0.0145 Ω
A π(d/2)2 π (0.001/2)2
(c) Esto lo podemos evaluar de 2 maneras. La primera es usando la relación V = IR, usando el R máximo

V = IR = (12.5)(0.0145) = 0.181 V

También se puede con V = EL, para campos eléctricos constantes

V = EL = (1.21)(0.015) = 0.0181V

[Resnick] Problema 29-14, [Sears] Problema 25-65, [Serway] Problema 28-85


Ampliación Verano 2013 Pregunta 7, Ampliación 2014 Verano Pregunta 7

155
CAPÍTULO 14. CORRIENTE Y RESISTENCIA

Un resistor presenta la forma de un cono truncado circular recto como se muestra en la Fig. 14.1. Los
radios terminales son a y b, y la longitud es L. Si la variación gradual es pequeña, cabe suponer que
la densidad de corriente es uniforme en cualquier sección transversal, (a) Calcule la resistencia de este
objeto. (b) Demuestre que su respuesta se reduce a ρL/A en el caso especial de una variación gradual
cero (a = b).

Figura 14.1: [Resnick] Problema 29-14

Solución
Ya que no es una figura con área constante, no podemos usar la relación R = ρL/A, si no que tenemos
que integrar ˆ
ρ
R= dx
A(x)
donde la integral recorre en la dirección donde avanzaría la corriente. En este caso es de la cara de radio
a hasta la cara de radio b. Ocupamos escribir el área en función de esa dirección Ponemos la distancia
que hay desde el eje hasta la superficie del cono en función de x como se muestra en la Fig. 14.2, donde
la variable x es la distancia que se ha avanzado en el eje, que va de x = 0 hasta x = L. La función r(x)

Figura 14.2: Radio en función de x.


[Resnick] Problema 29-14

tiene la forma r(x) = mx + b0 . La b0 de la pendiente es en r(x = 0) ⇒ b0 = a. La pendiente es


r(x = L) − r(x = 0) b−a
m= =
L−0 L
La función es  
b−a
r(x) = a + x
L
Ya podemos integrar
ˆ ˆ L ˆ L
ρ ρ ρ 1
R= dx = 2
dx =  2 dx
A πr (x) π
 
0 0 b−a
a+ x
L

156
CAPÍTULO 14. CORRIENTE Y RESISTENCIA

Haciendo la integral1 , obtenemos


   
ρ L 1 1 ρ L b−a
R= − =
πb−a a b πb−a ab

ρL
⇒R=
πab
(b) Hacemos el límite donde b → a, la resistencia en el límite es

ρL ρL
R= 2
=
πa A
Donde A = πa2 . Se reduce al caso de un cilindro normal.

Modificado del [Sears] Problema 25-66, [Resnick] Problema 29-15, [Serway] Problema 27-83
Se tiene un material ómhmico de resistividad ρ en forma esférica, radio interno a y radio externo b como
se muestra en la Fig. 14.3 A este se le conecta una fuente de voltaje V0 en el radio interno y en el externo.
(a) El campo eléctrico tiene la forma E = B/r2 r̂, donde B es una constante. Encuentre el valor B.(b)
Encuentre la resistencia del cilindro entre los puntos conectados a la fuente. (c) Encuentre la densidad de
corriente J. (d) Encuentre la corriente I.

Figura 14.3: Esfera óhmico

Solución
Esto se puede hacer 2 máneras distintas. No es necesario seguir el orden de las preguntas.
Método 1
(b) Encontremos la resistencia. Ocupamos hacer la integral
ˆ
ρ
R= dx
A
Donde el dx va en la dirección de la corriente. Aquí la corriente va en la dirección radial, entonces dx = dr.
El área en función de r es A(r) = 4πr2 .
ˆ ˆ b  
ρ ρ ρ 1 1
R= dr = 2
dr = − (14.1)
A a 4πr 4π a b
ˆ
1 dx 1
=−
(a + bx)2 b(a + bx)

157
CAPÍTULO 14. CORRIENTE Y RESISTENCIA

(d) El radio interno al externo están conectado a un voltaje V0 y ya calculamos la resistencia.


V0 4πV0
I= =   (14.2)
R 1 1
ρ −
a b

(c) Podemos asumir que la densidad de corriente solo depende de r, ya que el sistema tiene simetría
esférica. Con la corriente I calculada, dividimos entre el área de la esfera de radio r para obtener J
I 4πV0 V0
J= =   =  
A 1 1 2 1 1 2
ρ − 4πr ρ − r
a b a b
Ya que va en la dirección radial, el J es
V0 1
J = Jr̂ =   2 r̂ (14.3)
1 1 r
ρ −
a b

(a) Usamos la ley de Ohm y obtenemos


V0 1
E = ρJ =  r̂ (14.4)
1 1 r2


a b
El valor de la constante B es
V0
B=  (14.5)
1 1

a b

Método 2 Con esta manera vamos a ir al revés


(a) Podemos encontrar la constante B con la relación2 de V0 y E
ˆ ˆ b  
B 1 1
V0 = E · ds = 2
dr = B −
a r a b
Despejando B obtenemos
V0
B= 
1 1

a b
y el campo eléctrico es
V0 1
E=   r̂
1 1 r2

a b
Que dan lo mismo que las Ec. 14.4 y 14.5.
(c) Usando la ley de Ohm obtenemos
1
E = ρJ ⇒ J = E
ρ
2
Usualmente hay un menos, pero ya se que de lado es el potencial positivo y la dirección de E, pongo los límites de
integración para mantener todo positivo.

158
CAPÍTULO 14. CORRIENTE Y RESISTENCIA

V0 1
J=   2 r̂
1 1 r
ρ −
a b
Nos da lo mismo que la Ec. 14.3.
(d) La corriente la encontramos integrando
ˆ
I= J · dA

Integramos sobre una esfera de radio r, los dA son paralelos a los J y constante sobre la superficie de
integración, obtenemos ˆ
V0 1
I = JdA = JA =   2 4πr2
1 1 r
ρ −
a b
4πV0
⇒I=  
1 1
ρ −
a b
Nos da lo mismo que la Ec. 14.2.
(b) Con V0 y la corriente I, averiguamos la resitencia por su definición
 
V0 V0 ρ 1 1
R= = = −
I 4π a b
 4πV0 
  
 1 1 
ρ −
a b
Obtenemos lo mismo de la Ec. 14.1.

IV Parcial 2017 II Ciclo Pregunta 2


Se tiene un material ómhmico de resistividad ρ en forma de cilindro hueco de largo L, radio interno a y
radio externo b como se muestra en la Fig. 14.4 A este se le conecta una fuente de voltaje V0 en el radio
interno y en el externo.
(a) El campo eléctrico tiene la forma E = B/r r̂, donde B es una constante. Encuentre el valor B.(b)
Encuentre la resistencia del cilindro entre los puntos conectados a la fuente. (c) Encuentre la densidad de
corriente J. (d) Encuentre la corriente I.

Figura 14.4: Cilindro óhmico

Solución

159
CAPÍTULO 14. CORRIENTE Y RESISTENCIA

Igual que el problema pasado, el [Sears] Problema 25-66 Modificado, se puede hacer de las 2 maneras,
cada una con el orden respectivo. Lo único que cambia es la geometría. Ahora tenemos un cilindro un
lugar de una esfera.
Método 1
(b) Encontramos la resistencia integrando
ˆ
ρ
R= dx
A
Donde el dx va en la dirección de la corriente. La corriente es radial, entonces dx = dr, y el área A es el
área de un cilindro de radio r, que es A = 2πrL.
ˆ ˆ b  
ρ ρ ρ b
⇒R= dx = dr = ln (14.6)
A a 2πrL 2πL a

(d) Ya que tenemos V0 y R, la corriente la obtenemos con

V0 2πLV0
I= = (14.7)
R ρ ln(b/a)

textbf(c) Podemos asumir que J va en la dirección radial y depende solo de r por la simetría del sistema.
Usamos un cilindro de área A = 2πrL y obtenemos
I 2πLV0 1 V0 1
J= = =
A ρ ln(b/a) 2πrL ρ ln(b/a) r

Ya tenemos la magnitud de J, lo escribimos de forma vectorial


V0 1
J = Jr̂ = r̂ (14.8)
ρ ln(b/a) r

(a) Usando la Ley de Ohm, obtenemos el E

V0 1
E = ρJ = r̂ (14.9)
ln(b/a) r

Comparando, la constante B es
V0
B= (14.10)
ln(b/a)

Método 2
(a) La relación entre el potencial y el campo eléctrico es
ˆ ˆ b  
B b
V0 = E · ds = dr = B ln
a r a

V0
⇒B=
ln(b/a)
El campo E es
V0 1
E= r̂
ln(b/a) r

160
CAPÍTULO 14. CORRIENTE Y RESISTENCIA

Nos da lo mismo que las Ec. 14.9 y 14.10.


(c) Usamos la ley de Ohm y obtenemos

1 V0 1
E = ρJ ⇒ J = E= r̂
ρ ρ ln(b/a) r

Nos da lo mismo que la Ec. 14.8.


(d) Obtenemos la corriente I integrando J en un cilindro de radio r. Aprovechamos que todos los dA son
paralelos a J y J es constante sobre la superficie.
ˆ ˆ ˆ
V0 1
I = J · dA = JdA = J dA = JA = 2πrL
ρ ln(b/a) r

2πLV0
⇒I=
ρ ln(b/a)
Nos da lo mismo que la Ec. 14.7.
(b) Con V0 y I, encontramos R con su definición
 
V0 V0 ρ b
R= = = ln
I 2πLV0 2πL a
ρ ln(b/a)

Nos da lo mismo que la Ec. 14.6

[Resnick] Problema 31-5


IV Parcial 2017 I Ciclo Pregunta 3
Las barras conductores A y B están conectados en serie. Cada barra tiene una longitud igual de L = 42.6 m,
un área transversal de A = 91.0 × 10−4 m2 y resistencias de 76.2 µΩ y 35.0 µΩ respectivamente. Una
diferencia de potencial de V = 630 V se aplica en sus puntos terminales. Determine: (a) Las resistividades
de cada barra. (b) La densidad de corriente en cada barra. (c) La intensidad de campo eléctrico en cada
barra. (d) La diferencia de potencial en cada barra.

Solución
(a) La relación entre resistencia y resistividad de cada barra es

ρi Li Ri Ai
Ri = ⇒ ρi =
Ai Li
Evaluando para cada barra

(76.2 × 10−6 )(91.0 × 10−4 )


ρA = = 1.63 × 10−8 Ω · m
42.6
(35.0 × 10−6 )(91.0 × 10−4 )
ρB = = 7.48 × 10−9 Ω · m
42.6
(b) Tenemos 2 resistencias en serie, la resistencia total es RA + RB . La corriente que pasa por las
resistencias es
V 630
I= = = 5.67 × 106 A
RA + RB (76.2 + 35.0) × 10−6

161
CAPÍTULO 14. CORRIENTE Y RESISTENCIA

Ambas resistencias tienen la misma área y la misma corriente, entonces tienen la misma densidad de
corriente
I 5.67 × 106 A
J = JA = JB = = = 6.23 × 108 2
A 91.0 × 10−4 m
(c) El campo eléctrico en cada resistencia es Ei = ρi J

V
EA = (1.63 × 10−8 )(6.23 × 108 ) = 10.15
m
V
EB = (7.47 × 10−9 )(6.23 × 108 ) = 4.65
m
(d) La diferencia de potencial lo podemos obtener de 2 maneras. La primera usando al relación Vi = Ei Li

VA = (10.15)(42.6) = 432 V (14.11)

VB = (4.65)(42.6) = 198 V (14.12)


También lo podemos obtener la diferencia de potencial con Vi = Ii Ri

VA = (5.67 × 106 )(76.2 × 10−6 ) = 432 V

VB = (5.67 × 106 )(35.0 × 10−6 ) = 198 V


Lo cual da lo mismo que las Ec. 14.11 y 14.12.
Para verificar el resultado, la diferencias de potenciales VA + VB deberían sumar V

VA + VB = 432 + 198 = 630 V = V

Sí da lo que debería.

[Knight] Problema 28-74


En la Fig. 14.5 se muestra un cable de dos segmentos con el mismo diametro con conductividades σ1 y
σ2 . Cuando una corriente I pasa a través del cable, se acumula carga en la frontera entre los segmentos.
Obtenga una expresión para la densidad de carga η en la frontera. Sus resultado puede quedar en términos
de I, σ1 , σ2 y el área transversal del cable A.

Figura 14.5: [Knight] Problema 28-74

Solución
La densidad de corriente va a ser la misma en los 2 medios ya que tienen la misma I y la misma A.
Usemos la ley de Ohm para obtener los campo eléctricos en cada medio
I I
J = σk Ek = ⇒ Ek =
A σk A

162
CAPÍTULO 14. CORRIENTE Y RESISTENCIA

Ahora usamos la Ley de Gauss. Hacemos una superficie gaussiana cilindrica en la frontera. El campo
eléctrico va en la dirección de la corriente. El campo E1 entra a la superficie y E2 sale de la superficie.
˛
Qin
E · dA =
0
ηA I I η
⇒ E2 A − E1 A = ⇒ − =
0 σ1 A σ2 A 0
 
0 I 1 1
⇒η= −
A σ2 σ1

[Sears] Problema 25-87


La resistividad de un semiconductor se puede modificar si se agregan cantidades diferentes de impurezas.
Una varilla de material semiconductor de longitud L y de área de sección transversal A se localiza sobre
el eje x, entre x = 0 y x = L. El material cumple la ley de Ohm, y su resistividad varía a lo largo de
la varilla según la expresión ρ = ρ0 exp(−x/L). El extremo de la varilla en x = 0 está a un potencial V0
mayor que el extremo en x = L. (a) Calcule la resistencia total de la varilla y la corriente en ella. (b)
Encuentre la magnitud del campo eléctrico E(x) en la varilla en función de x. (c) Determine el potencial
eléctrico V (x) en la varilla en función de x.

Solución
(a) No podemos decir que R = ρL/A ya que la resistividad cambia con la posición. Encontramos R
integrando ˆ
ρ(x)
R= dx
A
La barra va de x = 0 hasta x = L. Calculemos la resistecia desde x0 = 0 hasta x0 = x
ˆ x  0
ρ0 x ρ0 L h  x i
R(x) = exp − dx0 = 1 − exp − (14.13)
0 A L A L

La resistencia total de la barra es evaluando en x = L


ρ0 L 
1 − e−1

R=
A
La corriente entonces es
V0 AV0
I= =
R ρ0 L [1 − e−1 ]
(b) La densidad de corriente es
I V0
J= =
A ρ0 L [1 − e−1 ]
La resistividad varía con la posición a lo largo del cable, pero en la dirección radial del cable, entonces la
densidad de corriente va a ser la misma en todo el cable. Usando le Ley de Ohm obtenemos
 x V0
E(x) = ρ(x)J = ρ0 exp −
L ρ0 L [1 − e−1 ]

V0 exp[−x/L]
⇒ E(x) =
L 1 − e−1

163
CAPÍTULO 14. CORRIENTE Y RESISTENCIA

(c) Esto lo podemos averiguar de 2 maneras


Método 1 Evaluamos la diferencia de potencial con V0 − V (x) = IR(x). Usamos la Ec.14.13

V0 A ρL h  x i
V (x) = V0 − IR(x) = V0 − 1 − exp −
ρ0 L (1 − e− 1) A L

Simplificando, obtenemos
exp(−x/L) − e−1
 
V (x) = V0 (14.14)
1 − e−1
Método 2 Integramos el campo eléctrico
ˆ x ˆ x
V0 exp[−x0 /L]
V (x) − V0 = − E · ds ⇒ V (x) = V0 − dx0
0 0 L 1 − e−1

Integrando y simplificando
1 − exp(−x/L)
⇒ V (x) = V0 − V0
1 − e−1
exp(−x/L) − e−1
 
⇒ V0
1 − e−1
Da lo mismo que la Ec. 14.14

164
15 | Circuitos DC

[Sadiku] Problema 2-12


Para el circuito en la Fig. 15.1, encuentre v1 , v2 y v3

Figura 15.1: [Sadiku] Problema 2-12

Solución
Usemos la regla de voltaje de Kirchhoff recorriendo las mallas como se muestra en la Fig. 15.2
Por la trayectoria 1 obtenemos

−40 − 50 + 20 + v1 = 0 ⇒ v1 = 70 V

Por la trayectoria 2 obtenemos

−40 − 50 + 30 + v3 = 0 ⇒ v3 = 60 V

Por la trayectoria 3 obtenemos


30 − v2 − 20 = 0 ⇒ 10 V
Podríamos haber usado la trayectoria 4 sabiendo cualquiera de los v1 , v2 ó v3 para haberiguar el que
faltaba. Pero aún así, hagamos la suma de voltajes por esa trayectoria

−v1 + v2 + v3 = −70 + 60 + 10 = 0 V

Sí nos da 0.

165
CAPÍTULO 15. CIRCUITOS DC

Figura 15.2: Recorrido de las mallas.


[Sadiku] Problema 2-12

[Sadiku] Problema 2-13


Para el circuito de la Fig. 15.3, usa la regla de corriente de Kirchhoff para encontrar de I1 a I4

Figura 15.3: [Sadiku] Problema 2.13

Solución
Los nodos se enumeran con números azules en la Fig. 15.3. Hagamos la regla de corrientes de Kirchhoff
en cada nodo. Usemos que las corrientes entrando al nodo son positivas y las que salen son negativas.
En el nodo 1 obtenemos
−3 − 7 − I2 = 0 ⇒ I2 = −10 A
Aunque la corriente I2 nos haya dado negativa, no le cambiemos la flecha de dirección. Dejemola ah’i con
el signo y cuando hagamos cálculos, respetamos el signo.
En el nodo 1 obtenemos

I1 + I2 − 2 = 0 ⇒ I1 = −I2 + 2 = −(−10) + 2 = 12 A

166
CAPÍTULO 15. CIRCUITOS DC

Donde aquí usamos el signo negativo de I2 en lugar de darle vuelta. Pueden comprobar que son consistentes
todos los cálculos al final con respecto al cálculo de las corrientes.
En el nodo 2 obtenemos
2 − I4 − 4 = 0 ⇒ I = −2 A
Podemos averiguar la última corriente desconocida I3 con los nodos 4 ó 5. Usemos el 4.

7 + I4 − I3 = 0 ⇒ I3 = 7 + I4 = 7 + (−2) = 5 A

Comprobemos que la suma de corrientes en el nodo 5 da 0

−I1 3 + I3 + 4 = −12 + 3 + 5 + 4 = 0 A

IV Parcial 2017 II Ciclo Problema 4


Se tiene una resistencia Rc usada para calentar agua como se muestra en la Fig. 15.4. (a) En términos
de V0 , Rc y R0 , cuanta es la potencia disipada en la resistencia Rc ? (b) Cuanto voltaje V0 tiene que
suministrar la fuente para evaporar una cantidad m de agua con calor latente de vaporización L en un
tiempo T ?
Suponga que toda la potencia disipada en la resistencia es transferida al agua. Nota: Q = mL

Figura 15.4: Calentador del Problema 4

Solución
(a) La potencia en el resistor Rc es P = I 2 Rc . Los resistores Rc y R0 están en serie. La corriente que
pasa a través de ellos es
V0
I=
Rc + R0
Entonces la potencia es
V02
P = I 2 Rc = Rc
(Rc + R0 )2
(b) La energía entregada en un tiempo T por la resistencia es E = P T . Esta energía es la energía usada
en la evaporación E = mL. Igualamos y despejamos V0

V02
P T = mL ⇒ Rc T = mL
(Rc + R0 )2
r
mL
⇒ V0 = (Rc + R0 )
Rc T

167
CAPÍTULO 15. CIRCUITOS DC

[Serway] Problema 28-31


Ampliación 2015 II Ciclo Pregunta 6, Ampliación 2017 II Ciclo Pregunta 7
Para el circuito de la Fig. 15.5, determine: (a) Encuentre la corriente en cada uno de los resistores. (b)
La potencia disipada en la resistencia R1 . (c) Encuentre la diferencia de potencial VA − VB .

Figura 15.5: [Serway] Problema 28-31

Solución
Escojemos las direcciones de las corrientes I1 , I2 e I3 como se muestra en la Fig. 15.6 con las flechas azules.
Ponemos los signos en el orden +− en las resistencias en la dirección de cada corriente, ya que la corriente
va del mayor potencial al menor potencial en las resistencias. Podemos ver en el nodo A obtenemos

Figura 15.6: Corrientes y mallas usadas.


[Serway] Problema 28-31

I1 + I3 = I2 (15.1)

Aplicamos la regla de voltaje de Kirchhoff usando las mallas que se muestran con las flechas naranjas en
la Fig. 15.6 y las recorremos en esa dirección, obtenemos las ecuaciones

I1 R1 − E1 + E2 + I2 R2 = 0 (15.2)

168
CAPÍTULO 15. CIRCUITOS DC

− I2 R2 − E2 − R3 I3 + E3 = 0 (15.3)
Reacomodando las Ec. 15.1, 15.2 y 15.3 obtenemos
    
1 −1 1 I1 0
R1 R2 0  I2  = E1 − E2 
0 R2 R3 I3 E3 − E2

Evaluando con los datos1     


1 −1 1 I1 0
2.00 3.00 0  I2  = 10.0
0 3.00 4.00 I3 20.0
La solución del sistema es    
I1 0.385
I2  = 3.077 mA
I3 2.692
(b) La potencia en R1 es

P = I12 R1 = (0.385 × 10−3 )2 (2000) = 2.96 × 104 mW = 0.296 mW

(c) Podemos calcularlo por 3 caminos diferentes. Vayamos por la izquierda

VA − VB = E1 − I1 R1 = 70.0 − (0.385)(2.00) = 69.23 V

Por el centro
VA − VB = E2 + I2 R2 = 60 + (3.077)(3.00) = 69.23 V
Por la derecha
VA − VB = −I3 R3 + E2 = −(2.692)(4.00) + 80.0 = 69.23 V
Por cualquier trayectoria se obtiene lo mismo.

[Sears] Problema 24-49


En el circuito de la Fig. 15.7 , todos los capacitores están descargados al principio, la batería no tiene
resistencia interna, y el amperímetro es ideal. Calcule la lectura del amperímetro (a) inmediatamente
después de haber cerrado el interruptor S y (b) mucho tiempo después de que se cerró el interruptor.
Solución
Antes de empezar el problema, veamos como se comportan en el caso límite, sin carga y completamente
cargado. En el caso que no hay carga, no hay una diferencia de potencial en el capacitor, los nodos de
conección van a estar al mismo potencial. Si ponemos una resistencia en paralelo en nodos que están al
mismo potencial, la corriente no va a pasar por la resistencia. Es como si la resistencia no estuviera como
se muestra en la Fig. 15.8. La resistencia queda en corto.
El otro caso límite es cuando está completamente cargado. No va a pasar corriente por el capacitor. Es
como si estuviera en abierto esa sección del circuito donde está el capacitor, como se muestra en la Fig.
15.9. Si va a haber una diferencia de potencial que va a estar determinada por el resto del circuito.
(a) Apenas se cierra el interruptor, los capacitores están descargados. Las resistencias en paralelo a ellos
quedan en corto. El circuito en t = 0 se comporta como el circuito de la derecha de la Fig. 15.10 La
1
Voy a poner las resistencias en kΩ, las corrientes van a dar en mA

169
CAPÍTULO 15. CIRCUITOS DC

Figura 15.7: [Sears] Problema 24-49

Figura 15.8: Capacitor descargado.


[Sears] Problema 24-49

Figura 15.9: Capacitor cargado.


[Sears] Problema 24-49

Figura 15.10: Circuito en t = 0.


[Sears] Problema 24-49

resistencia equivalente de las resistencias de 50 y 25 Ω es


 
1 1 1 1 (25)(50)
= + ⇒ R|| = Ω = 16.667 Ω
R|| 50 25 Ω (50 + 25)

170
CAPÍTULO 15. CIRCUITOS DC

Ahora todas las resistencias quedan en serie y podemos calcular la corriente inicial.
V 100 100
Ii = = = = 0.937 A = 937 mA
Req 75.0 + R|| + 15 75.0 + 16.67 + 15

(b) Mucho tiempo después (t → ∞), los capacitores están completamente cargados y no pasan corriente
en ellos. Para efectos de como lo ve el circuito, los capacitores se ven como si estuvieran en abierto como
se muestra a la derecha en la Fig. 15.11

Figura 15.11: Circuito en t → ∞.


[Sears] Problema 24-49

Vemos que todas las resistencias se ven como si estuvieran en serie. Podemos obtener la corriente com
V 100
If = = = 0.606 A = 606 mA
Req 25.0 + 75.0 + 25.0 + 25.0 + 15.0

[Irodov] Problema 3-184


Encuentre la diferencia de potencial Va − Vb entre las placas del capacitor C en el circuito mostrado. si
la fuente tiene une EMF E1 = 4.0 V y E2 = 1.0 V y las resistencias son iguales a R1 = 10 Ω, R2 = 20 Ω
y R3 = 30 Ω. Las resistencias de las fuentes son despreciables y que el capacitor está completamente
cargado.

Figura 15.12: [Irodov] Problema 3-184

Solución

171
CAPÍTULO 15. CIRCUITOS DC

Igual que el [Sears] Problema 24-49, el capacitor se va a ver como abierto y la diferencia de potencial está
determinada por el resto del circuito. Para hacer Kirchhoff lo podemos quitar. Asumamos una dirección
para cada corriente Ik en cada resistor Rk y ponemos los +− respectivos, donde cada corriente va del
+ → −, como se muestra en la Fig. 15.13

Figura 15.13: Corrientes y trayectorias para Kirchhoff.


[Irodov] Problema 3-184

Applicando regla de corrientes de Kirchhoff en el nodo que conecta a R1 , R2 y E2 obtenemos

I2 = I1 + I3

Ahora haciendo regla de voltaje de Kirchhoff en las mallas que se muestran con las flechas naranjas,
usando que I2 = I1 + I3 , obtenemos

− E1 + I1 R1 + (I1 + I3 ) R2 = 0 (15.4)

− E1 + I1 R1 + (I1 + I3 ) R2 = 0 (15.5)
Reordenando las Ec. 15.4 y 15.5 obtenemos
    
R1 + R2 R2 I1 E1
=
R2 R2 + R3 I3 −E2

Evaluando con los datos     


30 20 I1 4.0
=
20 50 I3 −1.0
La solución al sistema de ecuaciones es
   
I1 0.2
= A
I3 −0.1

Ahora podemos usar una malla cerrada que pase por el capacitor para saber VA − VB = VAB . Usemos las
2 trayectorias en verde que se muestran en la Fig. 15.14

172
CAPÍTULO 15. CIRCUITOS DC

Figura 15.14: Trayectorias para calcular VA − VB


[Irodov] Problema 3-184

Por la trayectoria 1 obtenemos


VAB + I1 R1 − E2 = 0 ⇒ VAB = −I1 R1 + E2 = −(0.2)(10) + 1.0 = −1.0 V
Por la trayectoria 2 obtenemos
I3 R3 + VAB + E1 = 0 ⇒ VAB = −I3 R3 − VAB = −(−0.1)(30) − 4.0 = −1.0 V
Comentario: Lo hice de la manera larga por “cuestiones didácticas” para que vean las cosas que se
pueden hacen, pero podríamos hacer Kirchhoff de una vez y obtener una matriz 4 × 4 en lugar de una
más simple de 2 × 2. Hagamos Kirchhoff usando las trayectorias de las Fig. 15.15 y no usemos quitemos
a I2 como hicimos antes en el sistema de ecuaciones. Las ecuaciones que obtenemos son
I1 − I2 + I3 = 0
I3 R3 + E2 + I2 R2 = 0
VAB + I1 R1 − E2 = 0
−E1 + I1 R1 + I2 R2 = 0
Acomodando las ecuaciones obtenemos
    
1 −1 1 0 I1 0
  I2  = −E2 
0 R2 R3 0    

R1 0 0 1   I3   E2 
R1 R2 0 0 VAB E1
Evaluando con los datos     
1 −1 1 0 I1 0
  I2  = −1.0
0 20 30 0    

10 0 0 1   I3   1.0 
10 20 0 0 VAB 4.0

173
CAPÍTULO 15. CIRCUITOS DC

Figura 15.15: Trayectorias para Kirchhoff sin pensar mucho


[Irodov] Problema 3-184

La solución del sistema es    


I1 0.2 A
 I2   0.1 A 
 I3  = −0.1 A
   

VAB −1 V
Obtenemos de una vez las corrientes y el resultado VAB que se buscaba.

[Sears] Problema 26-64


Cuál debe ser la EMF E para que la corriente a través del resistor de 7.00 Ω sea 1.80 A? Cada fuente de
EMF tiene una resitencia interna despreciable. Las unidades de la Fig. 15.16 son V para la fuente y Ω
para las resistencias

Figura 15.16: [Sears] Problema 26-64

Solución

174
CAPÍTULO 15. CIRCUITOS DC

El voltaje en el resistor de 7.00 Ω es

V = IR = (1.80)(7.00) = 12.6 V

Sea las corrientes I1 e I2 en las direcciones como se muestra en la Fig. 15.17 Aplicaquemos la regla de

Figura 15.17: Corrientes.


[Sears] Problema 26-64

voltajes de Kirchhoff por la trayectoria de afuera.


24 − 12.6
I1 (3.00) − 24.0 + 12.6 = 0 ⇒ I1 = = 3.80 A
3.00
Aplicamos la regla de corrientes de Kirchhoff en el nodo de arriba

I1 + I2 − 1.80 = 0 ⇒ I2 = 1.80 − I1 = 1.80 − 3.80 = −2.00 A

Ahora realizamos la regla de voltaje de Kirchhoff en la malla derecha

I2 (2.00) − E + 12.6 = 0 ⇒ E = I2 (2.00) + 12.6 = (−2.00)(2.00) + 12.6

⇒ E = 8.6 V

[Serway] Problema 28-75


Ampliación 2017 I Ciclo Pregunta 6
El interruptor del circuito de la Fig. 15.18 ha estado cerrado durante un tiempo suficientemente largo
para que el capacitor se cargue por completo. Determine (a) la corriente en estado estacionario en cada
resistor y (b) la carga Qmax del capacitor. (c) Ahora el interruptor se abre en t = 0. Escriba una ecuación
para la corriente en R2 como una función del tiempo y (d) Determine el intervalo de tiempo necesario
para que la carga del capacitor se reduzca a un quinto de su valor inicial.
Use los valores V0 = 9.0 V, R1 = 12.0 kΩ, R2 = 15.0 kΩ, R3 = 3.00 kΩ y C = 10 µF
Solución
(a) En estado estacionario, el capacitor se encuentra completamente cargado. Si está completamente
cargado, no pasa corriente a través del capacitor. La resistencia R3 está en serie con el capacitor. Por lo
tanto I3 = 0. Además, toda la corriente que pasa por R1 se va a ir a R2 . Las resistencias R1 y R2 están
en serie para estos efectos. Obtenemos la corriente con
V0 9.0
I1 = I2 = = mA = 0.333 mA
R1 + R2 (12.0 + 15.0)

175
CAPÍTULO 15. CIRCUITOS DC

Figura 15.18: [Serway] Problema 28-75

(b) El voltaje del capacitor es el mismo que el de la resistencia R2

R2
VC = IR2 = V0
R1 + R2
De aquí podemos despejar la carga Qmax

R2 (15)
Qmax = CVC = CV0 = (10)(9.0) = 50 µC
R1 + R2 12 + 15
(c) Cuando se abre el interruptor, el capacitor queda en serie con R2 y R3 . La constante de tiempo es
τ = (R1 + R2 ) C. La carga en el capacitor en función del t es
 
t
Q(t) = Qmax exp − (15.6)
τ

Derivamos Q(t) respecto al tiempo para obtener la corriente


   
dQ(t) Qmax t Qmax t
I(t) = =− exp − =− exp −
dt τ τ (R2 + R3 )C (R2 + R3 )C

Donde la dirección de esta corriente es entrado a la placa positiva (La de arriba) del capacitor. Es la
misma que la corriente en R2 , solo que la dirección entra positivo abajo de R2 .
Evaluando
   
(50) t t
I(t) = − exp − mA = −0.277 exp − mA
(15.0 + 3.00)(10) [(15.0 + 3.00)(10) × 10−3 ] 0.18

(d) Usamos la Ec. 15.6 y buscamos el t0 tal que Q(t0 ) = Qmax /5


   
Qmax t0 1 t0
Q(t0 ) = = Qmax exp − ⇒ = exp −
5 τ 5 τ
 
t0 1
⇒ − = ln ⇒ t0 = τ ln(5) = (R1 + R2 )C ln(5)
τ 5
Evaluando
t0 = 0.18 ln(5) = 0.289

[Sears] Problema 26-86

176
CAPÍTULO 15. CIRCUITOS DC

Un circuito RC tiene una constante de tiempo RC. (a) Si el circuito está descargándose, cuánto tiempo
tomará que la energía almacenada se reduzca a 1/e de su valor inicial? (b) Si se está cargando, cuánto
tiempo se necesita para que la energía almacenada alcance un 1/e de su valor máximo?

Solución
(a) Cuando el capactor se está cargargando, la carga en el capacitor es
 
Q(t) = Q0 1 − e−t/RC

La energía almacenada en el capacitor cuando se está cargado es

1 Q2 (t) 1 Q0  2  2
U (t) = = 1 − e−t/RC = U0 1 − e−t/RC
2 C 2 C
Queremos el U (t0 ) tal que U (t0 ) = U0 /e. Evaluamos y despejamos
U0  2
U (t0 ) = = U0 1 − e−t0 /RC ⇒ e−1/2 = 1 − e−t0 /RC
e
t0  
e−t0 /RC = 1 − e−1/2 ⇒ − = ln 1 − e−1/2
RC
 
−1/2
t0 = −RC ln 1 − e

El resultado parece no tener sentido que parece un tiempo negativo. El ln 1 − e1/2 es negativo. Negativo


por negativo da positivo. Sí está bien.


 
t0 = −RC ln 1 − e−1/2 = 0.9327RC

(b) La carga en el capacitor en descarga en este circuito es

Q(t) = Qi e−t/RC

Entonces la energía es
1 Q2 (t) 1 Q2i −2t/RC
U (t) = = e = U0 e−2t/RC
2 C 2 C
Donde U0 = Qi /(2C). Queremos el U (t0 ) que es U (t0 ) = U0 /e, igualamos
U0
U (t0 ) = = U0 e−2t0 /RC ⇒ e−1 = e−2t0 /RC
e
Los exponentes son iguales
2t0 RC
−1 = − ⇒ t0 =
RC 2

[Resnick] Problema 31-18, [Sears] Problema 26-89, [Serway] Problema 29-68


IV Parcial 2017 I Ciclo Pregunta 5
Un circuito RC está conectado a una fuente con un voltaje de E. El capacitor se encuentra descargado en
t = 0. En t = 0 se empieza a cargar el capacitor hasta t → ∞. Determine:
(a) La energía disipada en la fuente. (b) La energía almacenada en el capacitor. (c) La energía disipada
en la resistencia. (d) Como se relacionan estas cantidades? (e) La fracción de la energía suministrada

177
CAPÍTULO 15. CIRCUITOS DC

por la fuente que se almacenó en el capacitor.

Solución
Es un circuito en carga, la corriente en el tiempo es
E −t/RC
I(t) = e
R
´
Calculamos la energía disipada integrando E = P dt, donde la potencia en el tiempo es P (t) = V (t)I(t)
para cada elemento.
(a) Para la fuente de voltaje la energía disipada es
ˆ ∞ ˆ ∞
E E2
EE = VE (t)IE (t) dt = E e−t/RC dt = RC = E 2 C
0 0 R R

(b) En la resistencia es
ˆ ∞ ˆ ∞ ˆ ∞
E 2 −2t/RC E 2 RC E 2C
ER = VR (t)IR (t) dt = RI 2 (t) dt = R 2
e dt = =
0 0 0 R R 2 2

(c) La energía en el capacitor es una función de estádo. Inicialmente tenía 0 y al términar, el capacitor
va a tener voltaje E. La energía final es
E 2C
EC = (15.7)
2
Por deporte, integremos la potencia. Sabemos cuanto es la carga en el capacitor, que es

QC (t) = CEe−t/RC

Usamos la definición de capacitancia para obtener el voltaje


1
VC (t) = Q(t) = Ee−t/RC
C
Ahora integramos la potencia
ˆ ∞ ˆ ∞ ˆ ∞
−t/RC E −t/RC E2
EC = VC (t)IC (t) dt = Ee e dt = e−2t/RC dt
0 0 R R 0

E 2C
⇒ EC =
2
Da lo mismo de la Ec. 15.7.
(d) Toda la energía suministrada en la fuente es la energía que se disipo en la resistencia más la energía
almacenada en el capacitor.
EE = ER + EC
(e) La fracción es
EC E 2 C/2 1
f= = 2 =
EE E C 2
La mitad de la energía suministrada se almacenó en el capacitor.

178
CAPÍTULO 15. CIRCUITOS DC

IV Parcial 2017 II Ciclo Problema 3


Para el circuito de la Fig. 15.19, encuentre la corriente I3 con ambos métodos:
(a) Obteniendo resistencias equivalentes y reduciendo el circuito. (b) Aplicando las leyes de Kirchhoff.
(Sin reducir con resistencias equivalentes.) Use V0 = 110 V, R1 = 10 Ω, R2 = 4 Ω, R3 = 5 Ω.

Figura 15.19: IV Parcial 2017 II Ciclo Problema 3

Solución
(a) Sea R23 la resistencia equivalente entre R2 y R3 .

1 1 1 R2 R3
= + ⇒ R23 = (15.8)
R23 R2 R3 R2 + R3
La resistencia equivalente R23 está en serie con R1 , entonces la corriente que pasa por la fuente y R1 es
V0
I=
R1 + R23
El voltaje en las resistencias R2 y R3 es
V0
V3 = V23 = IR23 = R23 (15.9)
R1 + R23
Con el voltaje en R3 , ya podemos averiguar I3
V3 1 V0 1 V0 1 V0
I3 = = R23 =  =    
R3 R3 (R1 + R23 ) R3 R1 R3 1 1
+1 R1 + +1
R23 R2 R3

Evaluando con los datos


1 110
I3 =     =4A
5 1 1
10 + +1
5 4
Extra: Usando el Divisor de Voltajes 2 , podemos ahorrarnos pasos. Usemos la resistencia equivalente
R23 calculada Ec. 15.8, el voltaje es
R23
V23 = V0
R1 + R23
2
Uno de los problemas más adelante.

179
CAPÍTULO 15. CIRCUITOS DC

Y sabemos que V3 tiene que ser igual a V23 por que viene de resistencias en paralelo. Obtenemos el mismo
resultado de la Ec. 15.9 y ahorramos pasos intermedios.
(b) Sea la corriente I2 que pasa por la resistencia R2 . Haciendo la regla de corrientes de Kirchhoff en el
nodo que une a R1 , R2 y R3 obtenemos
I1 = I2 + I3
Realizamos las mallas como se muestra en la Fig. 15.20.

Figura 15.20: Mallas para hacer Kirchhoff.


IV Parcial 2017 II Ciclo Problema 3

− V0 + (I2 + I3 ) R1 + I2 R2 = 0 (15.10)

− I2 R2 + I3 R3 = 0 (15.11)
Reacomodando las Ec. 15.10 y 15.11 tenemos
    
R1 + R2 R1 I2 V0
=
−R2 R3 I3 0

Evaluando con los datos     


14 10 I2 110
=
−4 5 I3 0
La solución al sistema de ecuaciones es    
I2 5
= A
I3 4
La corriente es I3 = 4 A.

[Knight] Problema 28-78


Usted llego a las finales de las Olimpiadas de Ciencias! Como una de sus tareas, a usted se le da m =
1.0 g de aluminio y se le pide hacer un cable, usando todo el aluminio,que tiene que disipar P = 7.5 W
cuando se le conecta a una bateria de V = 1.5 V. Cual largo y diámetro va a escojer para el cable?
Datos: Resistividad del aluminio ρe = 2.8 × 10−8 Ω m, densidad del aluminio ρm = 2700 kg/m3
Solución
Podemos averiguar la resistencia que se ocupa con

V2 V2
P = ⇒R=
R P

180
CAPÍTULO 15. CIRCUITOS DC

Cambiando R = ρL/A
ρL V2
= (15.12)
A P
Ocupamos otra ecuación que tenemos 2 variables desconocidas. Sabemos cuanto es la masa de aluminio
que tenemos, entonces
m = ρm V = ρm LA (15.13)
Ya tenemos 2 Ec y 2 variables. Despejando L y A de 15.12 y 15.13 obtenemos
s s
ρe mP (2.8 × 10−8 )(0.001)(7.5)
A= = = 1.859 × 10−7 m2
ρm V 2 (2700)(1.5)2
s s
mV 2 (0.001)(1.5)2
L= = = 1.99 m
ρe ρm P (2.8 × 10−8 )(2700)(7.5)
Nos están pidiendo el diámetro, entonces despejamos el diámetro del área
r r
πd2 A 1.859 × 10−7
A= ⇒d=2 =2
4 π π

⇒ d = 4.86 × 10−4 m = 0.486 mm

[Knight] Problema 28-50


Un bombillo está en serie con un resistor de R0 = 2.0 Ω. Este bombillo disipa P = 10 W cuando a este
circuito en serie es conectado a una batería de V = 9.0 V. Cual es la corriente a través del bombillo? Hay
2 respuestas posibles, de ambas respuestas.

Solución
Sea Rb la resistencia del bombillo. La corriente por el bombillo es
V0 V0
I= = (15.14)
Req R0 + Rb

La potencia en el bombillo es
V02
P = I 2 Rb = R0 (15.15)
(R0 + Rb )2
Despejamos Rb de la Ec. 15.15 y obtenemos 2 soluciones

−2P R0 + V 2 ± −4P R0 V 2 + V 4
Rb =
2P
Evaluando para los 2 signos
p
−2(10)(2.0) + (9.0)2 ± −4(10)(2.0)(9.0)2 + (9.0)4
Rb =
2(10)

Rb = 1.6, 2.5 Ω

181
CAPÍTULO 15. CIRCUITOS DC

Evaluando para los valores posibles de la resistencia en la Ec. 15.14 obtenemos


9.0
I1 = = 2.5 A
2.0 + 1.6
9.0
I2 = = 2.0 A
2.0 + 2.5

[Sears] Problema 25-83


IV Parcial 2017 I Ciclo Problema 2
Considere el circuito de la siguiente figura. La fuente E tiene una resistencia despreciable. Los resistores
tienen resistencias de R1 = 6.00 Ω y R2 = 4.00 Ω. El capacitor tiene una capacitancia de C = 9.00 µF.
Cuando el capacitor se encuentra completamente cargado, la magnitud de la carga almacenada es de
Q = 36.0 µC. Determine: (a) La corriente en R1 . (b) El valor de la FEM E.

Figura 15.21: [Sears] Problema 25-83

Solución
El voltaje en el capacitor es
Q 36.0
VC = = = 4.0 V
C 9.00
El voltaje en la resistencia R2 es el mismo del capacitor que están en paralelo. La corriente en R2 es
VC 4.0
IR2 = = = 1.0 A
R2 4.0
Por el capacitor cargado no pasa corriente. La corriente IR2 es la misma de IR1 .

I = IR2 = IR1 = 1.0 A

(b) Con la regla de voltajes de Kirchhoff, haciendo una malla recorriendo E, R1 y R2 obtenemos

−E + IR1 + IR2 = 0 ⇒ E = I (R1 + R2 ) = (1.0)(4.00 + 6.00)

⇒ E = 10.0 V

[Irodov] Problema 3-118


Encuentre la diferencia de potencial en cada capacitor de la Fig. 15.22 de la placa izquierda a la derecha
en cada capacitor.
Solución

182
CAPÍTULO 15. CIRCUITOS DC

Figura 15.22: [Irodov] Problema 3-118

Figura 15.23: [Irodov] Problema 3-118

Pongamos la placa positiva de C2 del lado izquierdo, que va a estar cargado con q. Ya que la carga se
conserva, en la placa izquierda de C1 va a estar cargada con carga −q, como se muestra en la Fig. 15.23.
Recorremos la malla usando la regla de voltaje de Kirchhoff y obtenemos
q q
−E1 + + E2 + =0
C2 C1
Despejamos q  
C1 C2
⇒q= (E1 − E2 )
C1 + C2
La diferencia de potencial, de la placa izquierda a derecha en C2 es
 
q C1
V2 = = (E1 − E2 )
C2 C1 + C2

La diferencia de potencial de la placa derecha a izquierda en C1 es


 
0 q C2
V1 = = (E1 − E2 )
C1 C1 + C2

Entonces de izquierda a derecha es


 
C2
V1 = −V10 = (E2 − E1 )
C1 + C2

Problema: El Divisor de Voltaje

183
CAPÍTULO 15. CIRCUITOS DC

El divisor de voltaje es una manera rápida para saber cual es el voltaje a través de una resistencia.
Suponga que se tiene R1 , R2 , . . . , RN resistores en serie. Demuestre que el voltaje en el resistor Rk es
Rk
Vk = V0
R1 + R2 + · · · + RN

En particular, para 2 resistencias, el potencial en la resistencia 1 (ó la 2) es

R1 R2
V1 = V0 V2 = V0
R1 + R2 R1 + R2

Solución
La resistencia equivalente de todos los resistores en serie es Req = R1 + R2 + RN . La corriente que pasa
por todas las resistencias es
V0 V0
I= =
Req R1 + R2 · · · + RN
El voltaje en la Rk resistencia es
Rk
Vk = IRk = V0
R1 + R2 · · · + RN
Es un poco corto el procedimiento, pero puede ayudar a hacer el problema más rápido. Además ayuda
a analizar voltajes en circuitos, vease la Fig. 15.24. Si sabemos que el nodo de la izquierda está a un
potencial V0 respecto a tierra3 , sea lo que sea que esté pasando en ese nodo, con solo saber que está a un
potencial V0 , podemos aplicar el divisor de tensiones y obtener rápido el voltaje de R2 , que es
R2
V2 = V0
R1 + R2
Para una aplicación, vease el problema IV Parcial 2017 II Ciclo Problema 3.

Figura 15.24: Caso aplicado del divisor.

3
El nodo de tierra es el nodo donde se define el potencial V = 0.

184
Bibliografía

[Cengel] Cengel, Y. A., & Boles, M. A. Thermodynamics: An Engineering Approach. 8th ed, Mc Graw
Hill Education, 2015
[Irodov] Irodov, I. Problems in General Physics, Moscow: Mir Publisher, 1981
[Knight] Knight, R. Physics for Scientist and Engineers A Strategic Approach, 4th ed, Pearson, 2017
[Purcell] Purcell, E. M. & Morin, D. J., Electricity and Magnetism, 3rd ed, Cambridge University Press,
2013
[Resnick] Resnick, R., Halliday, D., & Krane, K. S. Física Vol. 1, 5ta ed, Grupo Editorial Patria, 2009
[Sadiku] Alexander, C. & Sadiku, M., Fundamentos de Circuitos eléctricos, 5ta ed, Mc Graw Hill, 2013.
[Sears] Young, H., Freedman, A., Ford, L., Sears, & F. Zemansky, M. Física Universitaria Vol. 1, 13va
ed, Pearson, 2013
[Serway] Serway, R., & Jewett, J. Física para ciencias e ingenierías Vol. 1, 9na ed, Cengage Learning,
2015

185

Vous aimerez peut-être aussi